Você está na página 1de 327

Mestrado Profissional

em Matemtica em Rede Nacional

Iniciao Matemtica

Autores:
Krerley Oliveira

Adn J. Corcho

Unidade I:
Captulos I e II

Dedicamos este livro as nossas esposas e lhos, que compreenderam


os sbados sacricados em funo de escrev-lo e a nossos pais, por
tudo o que eles representam.

Tente! E no diga que a vitria est perdida. Se de batalhas que se


vive a vida. Tente outra vez! (Raul Seixas)

vi

Sumrio
Prefcio

xi

1 Primeiros Passos

1.1

Organizando as Ideias

. . . . . . . . . . . . . . . . . .

1.2

Verdadeiro ou Falso?

. . . . . . . . . . . . . . . . . . .

1.3

Teoremas e Demonstraes . . . . . . . . . . . . . . . .

1.3.1

Mtodos de Demonstrao . . . . . . . . . . . .

1.4

Algumas Dicas para Resolver Problemas

1.5

Solues dos Problemas da Seo 1.4

1.6

Exerccios

. . . . . . . .

15

. . . . . . . . . .

18

. . . . . . . . . . . . . . . . . . . . . . . . .

26

2 Equaes e Inequaes
2.1

2.2

. . . . . . . . . . . . . . .

33

Problemas Resolvidos . . . . . . . . . . . . . . .

37

Sistemas de Equaes do Primeiro Grau

. . . . . . . .

42

Problemas Resolvidos . . . . . . . . . . . . . . .

46

Equao do Segundo Grau . . . . . . . . . . . . . . . .

49

2.3.1

Completando Quadrados . . . . . . . . . . . . .

50

2.3.2

Relao entre Coecientes e Razes

. . . . . . .

55

2.3.3

Equaes Biquadradas

. . . . . . . . . . . . . .

59

2.3.4

O Mtodo de Viti

. . . . . . . . . . . . . . . .

60

2.2.1
2.3

31

Equaes do Primeiro Grau


2.1.1

10

vii

viii

SUMRIO

2.4

Inequaes . . . . . . . . . . . . . . . . . . . . . . . . .

62

2.5

Inequao do Primeiro Grau . . . . . . . . . . . . . . .

63

2.6

Inequao do Segundo Grau

69

2.6.1
2.7

2.8

. . . . . . . . . . . . . . .

Mximos e Mnimos das Funes Quadrticas

75

Miscelnea . . . . . . . . . . . . . . . . . . . . . . . . .

77

2.7.1

Equaes Modulares

77

2.7.2

Um Sistema de Equaes No lineares

Exerccios

. . . . . . . . . . . . . . .
. . . . .

80

. . . . . . . . . . . . . . . . . . . . . . . . .

81

3 Divisibilidade

89

3.1

Conceitos Fundamentais e Diviso Euclidiana

. . . . .

90

3.2

Bases Numricas

. . . . . . . . . . . . . . . . . . . . .

99

3.3

Mximo Divisor Comum e Mnimo Mltiplo Comum

. 106

3.3.1

Mximo Divisor Comum . . . . . . . . . . . . . 106

3.3.2

Algoritmo de Euclides

3.3.3

Mnimo Mltiplo Comum

3.3.4

Equaes Diofantinas Lineares . . . . . . . . . . 120

. . . . . . . . . . . . . . 111
. . . . . . . . . . . . 115

3.4

Nmeros Primos e Compostos . . . . . . . . . . . . . . 123

3.5

Procurando Primos . . . . . . . . . . . . . . . . . . . . 127

3.6

3.5.1

O Crivo de Eratstenes . . . . . . . . . . . . . . 127

3.5.2

Primos de Mersenne

3.5.3

O Teorema Fundamental da Aritmtica . . . . . 133

Exerccios

. . . . . . . . . . . . . . . 129

. . . . . . . . . . . . . . . . . . . . . . . . . 139

4 O Princpio da Casa dos Pombos

143

4.1

Primeiros Exemplos . . . . . . . . . . . . . . . . . . . . 145

4.2

Uma Verso mais Geral . . . . . . . . . . . . . . . . . . 146

4.3

Aplicaes na Teoria dos Nmeros . . . . . . . . . . . . 149

4.4

Aplicaes Geomtricas . . . . . . . . . . . . . . . . . . 151

SUMRIO

ix

4.5

Miscelnea . . . . . . . . . . . . . . . . . . . . . . . . . 153

4.6

Exerccios

. . . . . . . . . . . . . . . . . . . . . . . . . 157

5 Contagem

161

5.1

Princpio Aditivo da Contagem

. . . . . . . . . . . . . 162

5.2

Princpio Multiplicativo de Contagem . . . . . . . . . . 170

5.3

Uso Simultneo dos Princpios Aditivo e Multiplicativo

5.4

Permutaes Simples . . . . . . . . . . . . . . . . . . . 181

5.5

Arranjos Simples

5.6

Combinaes Simples . . . . . . . . . . . . . . . . . . . 188

5.7

O Binmio de Newton

5.8

Contagem e Probabilidades

5.9

Exerccios Propostos

178

. . . . . . . . . . . . . . . . . . . . . 184

. . . . . . . . . . . . . . . . . . 193
. . . . . . . . . . . . . . . 195

. . . . . . . . . . . . . . . . . . . 197

6 Induo Matemtica

203

6.1

Formulao Matemtica

. . . . . . . . . . . . . . . . . 204

6.2

Aplicaes . . . . . . . . . . . . . . . . . . . . . . . . . 206
6.2.1

Demonstrando Identidades . . . . . . . . . . . . 206

6.2.2

Demonstrando Desigualdades

6.2.3

Induo e Problemas de Divisibilidade

. . . . . . . . . . 210
. . . . . 212

6.3

Induo na Geometria

6.4

Miscelnea . . . . . . . . . . . . . . . . . . . . . . . . . 220
6.4.1

. . . . . . . . . . . . . . . . . . 215

Cuidados ao Usar o Princpio da Induo . . . . 222

6.5

Induo e Recorrncias . . . . . . . . . . . . . . . . . . 222

6.6

Exerccios

. . . . . . . . . . . . . . . . . . . . . . . . . 229

7 Desigualdades

233

7.1

Desigualdade Triangular

. . . . . . . . . . . . . . . . . 234

7.2

Desigualdade das Mdias . . . . . . . . . . . . . . . . . 238

SUMRIO

7.3

Desigualdade de Cauchy-Schwarz

. . . . . . . . . . . . 245

7.4

Desigualdade de Jensen . . . . . . . . . . . . . . . . . . 246

7.5

Exerccios

. . . . . . . . . . . . . . . . . . . . . . . . . 250

8 Polinmios

255

8.1

Operaes com Polinmios . . . . . . . . . . . . . . . . 255

8.2

Algoritmo de Euclides

. . . . . . . . . . . . . . . . . . 263

8.3 Sempre Existem Razes de um Polinmio?


8.3.1
8.4

. . . . . . . . 268

Nmeros Complexos e Razes de Polinmios

Exerccios

. . 269

. . . . . . . . . . . . . . . . . . . . . . . . . 272

A Apndice: Funes

279

Referncias

285

Prefcio
Imaginao mais importante que onhe imento.
Albert Einstein
Leo, vo tem uma religio? Assim, uma religio, omo judasmo,
ou ristianismo, ou Matemti a...?
Alon Peres, 6 anos, lho do Matemti o Yuval Peres

Neste livro pretendemos oferecer ao leitor uma introduo Matemtica Elementar.

Juntando as experincias didticas vividas pelos

autores individualmente no Brasil e em Cuba, e mais alguns anos


juntos como treinadores de projetos de introduo Matemtica no
estado de Alagoas, esperamos tornar para o leitor a Matemtica mais
interessante, mostrando um pouco do imenso brilho e beleza que ela
esconde.
O livro foi escrito em captulos, cada um deles detalhando um
tema central e trazendo alguns teoremas fundamentais. Com muitos
exemplos e aplicaes dos conceitos introduzidos, pretendemos mostrar ao leitor a importncia do assunto abordado. A organizao dos
exemplos tenta seguir uma linha em ordem crescente de diculdade e,
para o melhor aproveitamento do livro, o trabalho com os exerccios
parte fundamental. Ler o enunciado e resolver o maior nmero pos-

xi

xii

Prefcio

svel de exerccios imperativo.

Como j disse o Prof.

Elon Lima,

Matemtica no se aprende passivamente.


Os exemplos e aplicaes dos conceitos, bem como os teoremas,
devem ser lidos com cuidado e muita ateno.

Para os estudantes

que desejem treinar para olimpadas de Matemtica, sugerimos que


formem grupos de estudo para trabalhar os temas individualmente,
sob a orientao de um professor. Acreditamos que o texto pode ser
utilizado em uma disciplina elementar num curso de licenciatura ou
bacharelado em Matemtica.
O primeiro captulo para introduzir o leitor no esprito do livro
e dar uma amostra do tipo de problemas e material que seguir nos
demais captulos. So propostos alguns problemas, muitos deles com
solues, e discutimos alguns mtodos importantes para uso no dia a
dia dos estudantes. Nesta discusso inclumos o estudo de proposies
matemticas, provas por contraposio, o mtodo de reduo ao absurdo e algumas outras regras bsicas e cuidados que devemos ter ao
resolver problemas em Matemtica.
Em seguida, estudamos as equaes do primeiro e do segundo grau.
Estudamos os mtodos de resoluo dessas equaes, sistemas de equaes, relaes entre razes e coecientes, bem como alguns problemas
interessantes que podem ser solucionados via essas equaes. Em seguida, estudamos inequaes do primeiro e do segundo grau.
O captulo seguinte trata do conceito de divisibilidade . Tentamos
introduzir o leitor nos principais aspectos bsicos, incluindo-se a divisibilidade com resto, mximo divisor comum e mnimo mltiplo comum,
nmeros primos e compostos, e um pouco de equaes diofantinas lineares.
Um captulo til para o estudante que deseja participar de Olim-

Prefcio

xiii

padas de Matemtica o que trata do princpio da casa dos pombos .


Este captulo um belo exemplo de como algo aparentemente ingnuo
pode gerar consequncias interessantes.

Alguns dos exemplos esto

conectados com os captulos anteriores e aparentemente aplicam o


princpio de modo inusitado, em problemas de geometria, teoria dos
nmeros e em reas diversas.
No captulo de contagem, comeamos com noes teis sobre conjuntos e princpios bsicos para contar os elementos de um conjunto.
Nesse captulo, estamos mais preocupados com as aplicaes imediatas
do assunto, sugerindo alguns problemas para o estudante iniciante.
Seguimos discutindo os tipos de agrupamento de elementos e suas
consequncias. Obtemos o binmio de Newton e introduzimos a noo de probabilidade de um conjunto, resolvendo alguns problemas
relacionados.
Em seguida, estudante se depara com uma arma poderosa do matemtico. O mtodo da induo nita estudado procurando conectar
esta noo com os captulos anteriores, reobtendo com o auxlio do
mtodo da induo algumas coisas que j foram deduzidas por outros
mtodos. Vrios exemplos e problemas so resolvidos, alguns deles de
modo surpreendente e inesperado.
No prximo captulo, introduzimos algumas desigualdades populares para o uso do estudante. Algumas dessas desigualdades so muito
importantes no estudo mais profundo da Matemtica e no aparecem em cursos introdutrios, apesar de suas provas e aplicaes serem
elementares.

Todas as desigualdades aparecem com demonstraes,

em muito dos casos utilizando-se lgebra elementar e o mtodo de


induo nita. So apresentados vrios exemplos que mostram a utilidade dessas desigualdades em alguns problemas prticos. Para xar

xiv

Prefcio

o conhecimento, propomos vrios exerccios complementares. Alguns


deles, cuja soluo mais elaborada, so sugeridos.

No ltimo ca-

ptulo, estudamos um pouco as propriedades gerais dos polinmios.


Para complementar a formao do leitor menos experiente, inclumos
um apndice sobre funes.
Somos gratos a muitas pessoas que colaboraram com a elaborao
deste livro com sugestes e correes em verses iniciais. Entre eles,
citamos: Carlos Gustavo Moreira, Ali Tahzibi, Feliciano Vitrio, Eduardo Teixeira, Chico Potiguar e vrios de nossos alunos de Iniciao
Cientca e mestrado, que por vrias ocasies deram sugestes para
a melhoria do texto. Um agradecimento especial vai para Fernando
Echaiz, que nos ajudou ativamente nas notas do Captulo 5 que originaram este texto. Finalmente, agradecemos aos revisores pela leitura
cuidadosa e ao comit editorial da SBM, na pessoa da profa. Helena
Lopes, pelo excelente trabalho de editorao.

Macei, Abril de 2010

Krerley Oliveira
Adn J. Corcho

1
Primeiros Passos
Redu tio ad absurdum,

que Eu lides gostava tanto, uma das mais

nas armas do matemti o. muito mais no que um movimento


de xadrez: o jogador de xadrez pode ofere er o sa rif io de uma
pea, mas o matemti o ofere e o jogo inteiro.
G. H. Hardy

Neste captulo, discutiremos algumas ideias gerais e convenes


que serviro como base para os diferentes mtodos de resoluo de
problemas que trataremos nos captulos seguintes. Alguns dos exemplos que abordamos sero teis para orientar quanto ao cuidado que
devemos ter quando discutimos problemas em Matemtica.

1.1

Organizando as Ideias

Para resolver problemas matemticos precisamos ter bem claro o que


devemos provar e o que estamos assumindo como verdade.

sobre

isso que falaremos agora. Comearemos observando as seguintes armaes:

Primeiros Passos

(a) A soma de dois nmeros pares sempre um nmero par.

(b) Todo brasileiro carioca.

(c) A terra um planeta.

(d) Se

b,
(e) Se

o comprimento da diagonal de um retngulo de lados

ento

c 2 = a2 + b 2 .

a < 1,

ento

a2 > a.

Todas as armaes acima se encaixam no conceito de proposio,


que damos a seguir.
Uma proposio ou sentena uma frase armativa em forma de
orao, com sujeito, verbo e predicado, que ou falsa ou verdadeira,
sem dar lugar a uma terceira alternativa.
Por exemplo, as proposies (a) e (c) so claramente verdadeiras;
mais adiante nos convenceremos da veracidade da proposio (d). Por
outro lado, as proposies (b) e (e) so falsas. Com efeito, para constatar a veracidade da sentena (b) teramos que checar o registro de
nascimento de cada brasileiro e vericar se nasceu no Rio de Janeiro,
mas isto falso pois o conhecido escritor Graciliano Ramos um
brasileiro nascido em Alagoas. Analogamente, para convencer-nos de
que a proposio (e) falsa basta tomar

1/4

no maior do que

1/2

a = 1/2 e checar que (1/2)2 =

como a sentena arma. Em ambos os

casos temos vericado que as proposies (b) e (e) so falsas apresentando casos particulares onde as mesmas deixam de valer. Estes
casos particulares so chamados de contraexemplos e so muito teis
para vericar a falsidade de algumas proposies.
Notemos que as proposies (d) e (e) so do tipo:

1.1

Organizando as Ideias

Se
onde

P,

ento

Q,

tambm so sentenas. Por exemplo, na proposio (e)

temos que:

P: c

o comprimento da diagonal de um retngulo de lados

b,

Q: c2 = a2 + b2 ,
ou seja, estamos assumindo que
mos vericar se

verdade e usando este fato deve-

verdade ou no.

Uma proposio condicional ou implicativa uma nova proposio


formada a partir de duas proposies
Se

P,

ento

Q

ou

P

Q,

implica

onde para o ltimo caso usamos a notao:


a proposio

que escrita na forma:

de hiptese e a proposio

Q,

P = Q. Chamaremos
Q de tese. A hiptese

tambm chamada de proposio antecedente e a tese, de proposio

consequente.
Por exemplo, na proposio condicional (f ) a hiptese :
tese :

a<1

e a

a > a.

A partir de uma de uma proposio condicional podem-se gerar


novas proposies que so de especial interesse para os matemticos.
Vamos chamar o modo em que apresentamos uma proposio de forma

positiva. Por exemplo, quando enunciamos a proposio


Se como laranja, ento gosto de frutas,
assumimos esta armao como sua forma positiva. Vamos descrever
agora como podemos obter novas proposies a partir desta.

Primeiros Passos

Forma recproca de uma proposio condicional:

para cons-

truirmos a forma recproca, temos que trocar na forma positiva a hiptese pela proposio consequente e vice-versa.

Vejamos em nosso

exemplo:

Forma da proposio
Positiva
Recproca

Hiptese

Tese

como laranja

gosto de frutas

gosto de frutas

como laranja

Assim, a recproca de proposio de nosso exemplo ento:


Se gosto de frutas, ento como laranja

Forma contrapositiva de uma proposio condicional:

Para

obtermos a forma contrapositiva a partir da forma positiva de uma


proposio condicional podemos fazer primeiro sua forma recproca e
em seguida negamos as sentenas antecedente e consequente da recproca ou, tambm, podemos primeiro negar as sentenas antecedente
e consequente da forma positiva e imediatamente fazer a forma recproca desta ltima. A forma contrapositiva tambm conhecida como
forma contrarrecproca . Usando novamente nosso exemplo temos que:

Forma da Proposio
Positiva
Recproca
Contrapositiva

Hiptese

Tese

como laranja

gosto de frutas

gosto de frutas

como laranja

no gosto de frutas

no como laranja

Portanto, a forma contrapositiva escreve-se assim:

1.2

Verdadeiro ou Falso?

Se no gosto de fruta, ento no como laranja

Em particular, a forma contrapositiva de uma proposio poder ser,


eventualmente, uma forma indireta muito ecaz de vericar resultados
em Matemtica.

1.2

Verdadeiro ou Falso?

Uma das coisas que distingue a Matemtica das demais cincias naturais o fato de que um tema de Matemtica discutido utilizando-se
a lgica pura e, por conta disso, uma proposio em Matemtica, uma
vez comprovada sua veracidade, aceita como verdade irrefutvel e
permanecer assim atravs dos sculos. Por exemplo, at hoje usamos
o teorema de Tales do mesmo modo que foi usado antes de Cristo e
este fato continuar valendo eternamente.
Vamos ilustrar melhor essa diferena com um exemplo em Geograa.

Hoje, todos ns sabemos que a Terra tem aproximadamente

o formato de uma laranja, um pouco achatada nos polos.

Porm,

na poca de Pitgoras, um dos grandes temores dos navegadores era


encontrar o m do mundo. No pensamento de alguns destes aventureiros, a Terra tinha o formato de um cubo, e uma vez chegando em
um dos seus extremos, o navio despencaria no vazio. Esse um dos
muitos exemplos de como a concepo da natureza mudou ao longo
do tempo, transformando uma concepo verdadeira num perodo da
humanidade em algo completamente falso em outra poca.

Porm,

para nossa felicidade, isso no acontece na Matemtica. Uma proposio matemtica ou verdadeira ou falsa e permanecer assim para
sempre.

Primeiros Passos

Mas como saber se uma proposio verdadeira ou falsa? A primeira coisa que devemos fazer tomar muito cuidado. As aparncias
enganam ou, como diziam nossos avs, nem tudo que reluz ouro.
O leitor, avisado disso, pense agora na seguinte pergunta:

Pergunta 1:

Qual a chance de que pelo menos duas pessoas num

nibus com 44 passageiros faam aniversrio no mesmo dia do ano?


Como j avisamos, o leitor deve ter cuidado ao responder pergunta acima, pois podemos nos enganar muito facilmente. Por exemplo, podemos formular o seguinte argumento errado: o ano tem 365
dias e, como estou escolhendo um grupo de 44 (nmero muito pequeno
com respeito a 365) pessoas ao acaso, claro que podemos responder
pergunta com a seguinte armao:

Resposta intuitiva:

A chance de que num grupo de 44 pessoas pelo

menos duas delas faam aniversrio no mesmo dia do ano pequena.


primeira vista a resposta dada pode at parecer verdadeira, mas
com uma anlise mais cuidadosa veremos que completamente falsa.
Na verdade, a chance de que pelo menos duas pessoas do nibus faam
aniversrio no mesmo dia do ano de cerca de 93%!
Quem no acreditar nisto pode fazer duas coisas: primeiro, ir a
sua sala de aula ou no seu nibus escolar, que deve ter pelo menos
44 pessoas, e fazer o experimento ao vivo. Muito provavelmente voc
deve conseguir duas pessoas que fazem aniversrio no mesmo dia do
ano. Se voc verica que existem duas pessoas que fazem aniversrio
no mesmo dia do ano, no por acaso, pois a chance de isso acontecer
muito alta. Mas, cuidado! Isso no uma prova matemtica para
este fato. Para provar que este fato verdadeiro voc deve vericar
que se escolhermos ao acaso um grupo de 44 pessoas ento com aproxi-

1.2

Verdadeiro ou Falso?

madamente 93% de chance, pelo menos duas delas fazem aniversrio


no mesmo dia do ano!
Porm, se voc faz o experimento e no encontra duas pessoas que
fazem aniversrio no mesmo dia do ano (voc seria muito azarado!),
no se desespere. Lembre-se de que se trata de algo que acontece com

chance de 93% e que pode no acontecer quando fazemos um teste.


Em qualquer um dos casos, para ter a certeza de que a proposio
verdadeira o leitor deve demonstr-la.

Faremos isso no nal do

Captulo
Vamos analisar agora outro fato aparentemente bvio.

Pergunta 2:

Num campeonato de futebol onde cada time joga a

mesma quantidade de jogos, cada vitria vale trs pontos, o empate


vale um ponto e a derrota nenhum ponto.

Em caso de empate, o

critrio de desempate entre as equipes era o seguinte:

A melhor equipe aquela que tem mais vitrias.

Os organizadores decidiram passar a adotar o critrio a seguir:

A melhor equipe aquela que tem mais derrotas.

Voc acha que este ltimo critrio adotado justo?


Com respeito a esta pergunta, o leitor deve ter respondido do seguinte modo:

Resposta:

Um time que perdeu mais pior que um que perdeu me-

nos; portanto, a mudana de critrio totalmente injusta. Acertamos


a sua resposta?
Na verdade, no houve mudana nenhuma de critrio, ou seja,
ambos os critrios nos conduzem ao mesmo ganhador.

Primeiros Passos

Para ver isso rapidamente, lembre-se de que se a equipe


mais que a equipe

conseguisse empatar com a equipe


Sejam

perdeu

e ainda assim empataram, ento ela deve ter

ganho mais, para que no m do campeonato a equipe

mente.

d1 , e1 , v1

B.

ainda assim

Vamos mostrar isso precisa-

o nmero de derrotas, empates e vitrias,

respectivamente, da equipe

A.

Do mesmo modo, sejam

d2 , e2 , v2

nmero de derrotas, empates e vitrias, respectivamente, da equipe

B.
B,

Suponhamos que a equipe


ou seja, que

v1 > v2 .

obteve mais vitrias do que a equipe

Como cada equipe jogou o mesmo nmero de

jogos, temos que

d1 + e1 + v1 = d2 + e2 + v2 .

(1.1)

Por outro lado, note que o nmero de pontos obtidos pela equipe

e1 + 3v1 .
B igual

Do mesmo modo, o nmero de pontos obtidos pela equipe


a

e2 + 3v2 .

Como as duas empataram, temos que:

e1 + 3v1 = e2 + 3v2 .
Ou ainda,

3(v1 v2 ) = e2 e1
Como

ou

v2 v1 =

v1 v2 > 0, temos que e2 e1 > 0.

e2 e1
.
3

Reescrevendo a equao (1.1),

temos que:

d1 d2 = e2 e1 + (v2 v1 ) = e2 e1

e2 e1
2
= (e2 e1 ).
3
3

d1 d2 > 0, pois e2 e1 > 0. Isso signica que


derrotas que B ; logo, qualquer um dos dois critrios de

Logo, temos que

teve mais

desempate usado nos leva equipe vencedora.

1.3

Teoremas e Demonstraes

Assim, como estes dois exemplos mostram, ao depararmos com um


problema em Matemtica, devemos ter cuidado ao tirar concluses
apressadas para evitar que cometamos algum engano. Pode acontecer
que uma situao que claramente falsa para um observador menos
atento, se mostre verdadeira quando fazemos uma anlise mais criteriosa.

1.3

Teoremas e Demonstraes

Agora denimos o que entendemos por demonstrao matemtica de


uma proposio.
Uma demonstrao em Matemtica o processo de raciocnio lgico e dedutivo para checar a veracidade de uma proposio condicional. Nesse processo so usados argumentos vlidos, ou seja, aqueles
que concluam armaes verdadeiras a partir de fatos que tambm
so verdadeiros.
Como exemplo de demonstrao citamos a argumentao usada
para mostrar na segunda pergunta da seo anterior que os critrios
de desempate eram similares.
Sempre que, via uma demonstrao, comprovemos a veracidade de
uma proposio passamos ento a chamar esta de teorema. Assim, um
teorema qualquer armao que possa ser vericada mediante uma
demonstrao.
Alguns teoremas se apresentam na forma de uma proposio con-

P , ento Q ou implicativa
sentena P chamada de hiptese

dicional, isto , uma sentena do tipo Se


da forma  P
e a sentena

= Q. Nesse caso, a


Q denominada de tese.

nos implica a veracidade da tese.

Ou seja, a validade da hiptese

10

Primeiros Passos

Um exemplo de teorema o famoso teorema de Pitgoras , cujo


enunciado diz o seguinte:

Teorema 1.1

(Teorema de Pitgoras)

Num tringulo retngulo a

soma dos quadrados dos catetos igual ao quadrado da hipotenusa.

Notemos que o teorema de Pitgoras no est enunciado na forma


condicional, mas pode ser reescrito nessa forma como:

Teorema 1.2 (Teorema de Pitgoras) . Se T


de catetos

e hipotenusa

Observao 1.3.

c,

ento

um tringulo retngulo
2

c = a + b2 .

Em geral, mais comum usar a palavra teorema

apenas para certas proposies que so de grande importncia matemtica, chamando-se simplesmente de proposio ao resto das proposies verdadeiras que admitem uma demonstrao. Para uma discusso mais detalhada, recomendamos [8].

1.3.1 Mtodos de Demonstrao


Quando realizamos uma demonstrao no existe um caminho nico.
Dependendo do problema em questo podemos usar mtodos diferentes. A seguir ilustramos os seguintes trs mtodos:

Demonstrao direta.

Demonstrao por contraposio.

Demonstrao por reduo ao absurdo.

1.3

Teoremas e Demonstraes

11

Demonstrao Direta
A demonstrao direta aquela em que assumimos a hiptese como
verdadeira e atravs de uma srie de argumentos verdadeiros e dedues lgicas conclumos a veracidade da tese.

Figura 1.1: Figura auxiliar para a demonstrao do teorema de Pitgoras


Um exemplo de demonstrao direta a que daremos a seguir,
para o teorema de Pitgoras enunciado anteriormente no Teorema
1.1. Com efeito, usando a gura acima temos que a rea do quadrado
de lado

a+b

a soma das quatro reas dos tringulos retngulos

congruentes pelo critrio lado-ngulo-lado (de catetos


rea do quadriltero

Q,

b)

mais a

o qual um quadrado visto que cada um dos

seus lados coincide com a hipotenusa

dos tringulos retngulos de

b e, alm disso, cada um dos seus ngulos internos


= 180 ( + ) = 180 90 = 90 (veja a Figura 1.1).
catetos

Portanto,

(a + b)2 = 4

ab
+ c2 ,
2

de onde

a2 + 2ab + b2 = 2ab + c2 ,
e consequentemente

a2 + b 2 = c 2 ,

mede

12

Primeiros Passos

como queramos.

Demonstrao por Contraposio


Este mtodo baseado no fato de que a veracidade de forma positiva
de uma proposio equivalente veracidade de sua forma contrapositiva, podendo ser esta ltima, eventualmente, mais fcil de se provar.
Por exemplo, a armao

Se sou alagoano, ento sou brasileiro


equivalente armao

Se no sou brasileiro, ento no sou alagoano


Por exemplo, provemos a seguinte proposio:

Proposio 1.4.

Hiptese:

Tese:

Se

N2

N2

par, ento

par.

par.

par.

Desaamos o leitor a tentar mostrar esta proposio partindo da hiptese e tentando concluir a tese. Note que podemos vericar que nossa
proposio verdadeira para vrios valores de

N2

como na tabela a

seguir, mas isso no uma prova matemtica da nossa proposio.

N2
N

16

36

64

100

144

10

12

1.3

Teoremas e Demonstraes

13

Mesmo vericando para um bilho de valores de


restariam nmeros para serem vericados.

N 2,

sempre nos

Como nossas tentativas

de provar a forma positiva dessa proposio esto sendo frustradas,


apelaremos para mostrar a forma contrapositiva da mesma, isto :

Proposio 1.5.

Se

no par, ento

N2

no par.

Neste caso, temos:

Hiptese:

Tese:

N2

no par.

no par.

N
tem que ser mpar, ou seja, existe p, nmero inteiro, tal que N = 2p+1.

Demonstrao. Como estamos assumindo que

no par, logo

Logo,

N 2 = (2p + 1)(2p + 1)
= 4p2 + 2p + 2p + 1
= 4p2 + 4p + 1
= 2(2p2 + 2p) + 1
= 2q + 1,
onde

q = 2p2 + 2p.

Logo,

N 2 = 2q + 1

mpar e conclumos assim

nossa prova.

Demonstrao por Reduo ao Absurdo


Este mtodo uma das ferramentas mais poderosas da Matemtica.
O nome provm do latim reductio ad absurdum e tambm conhecido
como mtodo do terceiro excludo devido ao mesmo estar baseado na

14

Primeiros Passos

lei do terceiro excludo que diz o seguinte: uma armao que no


pode ser falsa, dever ser consequentemente verdadeira.
De um modo geral, o roteiro que segue uma demonstrao por
reduo ao absurdo o seguinte:

Assumimos a validade da hiptese.

Supomos que nossa tese falsa.

Usando as duas informaes anteriores conclumos, atravs de


argumentos verdadeiros, uma armao falsa; como tal fato no
poder ocorrer, ento nossa tese dever ser verdadeira.

Vamos mostrar como o mtodo funciona na prtica provando a


seguinte proposio:

Proposio 1.6.

Seja

um nmero positivo, ento

x + 1/x 2.

Destaquemos primeiramente a nossa hiptese e a nossa tese.

Hiptese:

Tese:

um nmero positivo.

x + 1/x 2.

Demonstrao. Seja
falsa, isto ,

x+

1
x

x um nmero positivo e suponhamos que a


< 2. Usando que x > 0 e multiplicando por

tese
este

a desigualdade anterior, obtemos que

x2 + 1 < 2x.
x2 2x + 1 < 0 equivalente a (x 1)2 < 0, j que
x2 2x + 1 = (x 1)2 , o que impossvel. Portanto, x + 1/x 2,
Da segue-se que

como desejvamos.

1.4

Algumas Dicas para Resolver Problemas

1.4

15

Algumas Dicas para Resolver Problemas

Nesta seo, damos algumas regras gerais que consideramos importante ter em mente na hora de resolver um problema de Matemtica.
Aplicaremos estas regras a alguns problemas interessantes para ilustrar a sua importncia. Elas so:

R1) Ler bem o enunciado do problema e utilizar todas as informaes


disponveis.
R2) Fazer casos particulares ou casos mais simples de problemas similares, para adquirir familiaridade com o problema.
R3) Mudar a representao do problema, transformando-o em um
problema equivalente.
R4) Usar a imaginao pesquisando caminhos alternativos.

Extra-

polar os limites!

A seguir propomos vrios problemas onde as regras anteriores so


muito teis.

O leitor deve tentar resolv-los; mas se no conseguir

achar soluo depois de muito tentar poder ento passar para a prxima seo onde os solucionamos.

Problema 1.7.

Ao encontrar uma velha amiga (A), durante uma

viagem de trem, um matemtico (M) tem a seguinte conversa:

(M)

 Como vo os trs lhos da senhora?

(A)

 Vo bem, obrigada!

16

Primeiros Passos

(M)

 Qual a idade deles mesmo?

(A)

 Vou lhe dar uma dica. O produto das idades deles 36.

(M)

 S com essa dica impossvel!

(A)

 A soma das idades deles igual ao nmero de janelas deste


vago.

(M)

 Ainda no sei!

(A)

 O mais velho toca piano!

(M)

Agora eu sei!

Voc capaz de descobrir as idades dos trs lhos da senhora?

Problema 1.8.

Numa cesta encontram-se 9 moedas idnticas, sendo

que 8 delas tm o mesmo peso e uma moeda mais leve que as demais.
Usando duas vezes uma balana de dois pratos, encontrar a moeda
mais leve.

Problema 1.9.

Numa pequena ilha existem 5 pessoas de olhos azuis

e 5 pessoas de olhos verdes. Existe um grande tabu nesta ilha que o


seguinte: se uma pessoa descobre que possui olhos azuis ela se suicida
meia-noite do dia em que descobriu, pulando do alto da prefeitura.
Por conta disso, ningum conversa sobre o assunto, olha para espelhos
ou v seu reexo na gua. Todos se cruzam diariamente e conhecem
os olhos de seus amigos. Numa manh, um estrangeiro chegou ilha
e reuniu as 10 pessoas para o seguinte pronunciamento:
Nesta ilha, existe uma pessoa de olhos azuis.
Pergunta-se:

1.4

Algumas Dicas para Resolver Problemas

17

(a) O que aconteceu com os habitantes da ilha?


(b) Que informao nova o estrangeiro trouxe?

Problema 1.10.

Um viajante deseja se hospedar durante 31 dias num

hotel. Entretanto, percebe que est sem dinheiro e que a nica coisa
que possui uma corrente com 31 elos de ouro. Para pagar sua conta,
ele acertou com o gerente pagar um elo por dia, sem atrasar ou adiantar o pagamento, durante os 31 dias. O gerente pode dar troco em
elos. Depois ele deseja recuperar a corrente e por isso ele quer pagar
a conta cortando a corrente no menor nmero de pedaos. Quantos
cortes voc conseguiria dar e pagar a conta?

Problema 1.11.

Sabendo que em cada jogada o movimento do cavalo

consiste em se deslocar duas casas na horizontal e uma na vertical


ou duas na vertical e uma na horizontal, decidir se possvel sair
da congurao apresentada no tabuleiro (a) e chegar congurao
apresentada no tabuleiro (b) da Figura 1.2 sem que em algum momento
existam dois cavalos na mesma casa.

(a)

(b)

Figura 1.2: Cavalos de xadrez

18

Problema 1.12.

Mostre que podemos cobrir os

Primeiros Passos

9 pontos no reticulado

da Figura 1.3 traando 4 segmentos de reta sem tirar o lpis do papel.

Figura 1.3: Reticulado de 9 pontos


Sugerimos seguir as dicas abaixo para obter sucesso na soluo dos
problemas:

Para os problemas 1.7 e 1.8 use a primeira regra.

Para os problemas 1.9 e 1.10 use a segunda regra. Por exemplo,


no problema 1.9 fazer primeiro o caso: uma pessoa com olhos
azuis e uma com olhos verdes e depois fazer o caso: duas pessoas
de olhos azuis e duas de olhos verdes; generalize.

Para os problema 1.11 use a terceira regra.

Para o problema 1.12 use a quarta regra.

1.5

Solues dos Problemas da Seo 1.4

A seguir apresentamos solues para os problemas enunciados na seo


anterior.

Soluo do Problema 1.7.

muito importante neste problema tirar

o mximo de informao das dicas da senhora. Vamos primeira dica:


o produto das idades 36.

1.5

Solues dos Problemas da Seo 1.4

19

0 6 x 6 y 6 z 6 36.
possibilidades para os nmeros x,

Suponhamos que as idades dos lhos sejam


Como

xyz = 36,

temos as seguintes

z:
x y
1
1
1
1
1
2
2
3

xyz

1 36
2 18
3 12
4 9
6 6
2 9
3 6
3 4

36
36
36
36
36
36
36
36

A segunda dica dada pela senhora a soma das idades.

Assim,

vamos agora calcular todas as possveis somas de acordo com as fatoraes de 36 dadas na tabela anterior:

x y
1
1
1
1

x+y+z

1 36
2 18
3 12
4 9

1 6

2 2
2 3
3 3

9
6
4

38
21
16
14
13
13

11
10

Sabemos que aps a segunda dica, o matemtico ainda no conseguiu deduzir as idades das crianas.

20

Primeiros Passos

Por que ele no conseguiu? Imagine que o nmero da casa fosse


14. Ora, de acordo com nossa tabela, s existe um terno de nmeros
cujo produto 36 e a soma 14, que o terno (1,4,9). Assim, se o
nmero da casa fosse 14 o matemtico teria dado a resposta aps a
segunda dica. Como ele cou em dvida, olhando a tabela 2, chegamos
concluso de que o nmero da casa s pode ser igual a 13.
Lembremos a ltima dica: o mais velho toca piano. No incio essa
dica parecia intil, mas agora ela fundamental para resolvermos o
problema. De fato, como o mais velho toca piano, isso signica que
existe um mais velho, o que descarta o caso (1,6,6). Assim, as idades
so 2, 2, e 9.

Soluo do Problema 1.8.

Este o tipo de problema que a primeira

vista pode parecer difcil, mas que quando usamos todas as informaes do seu enunciado se torna fcil. A ideia dividir as moedas em

A, B
A e B e deixaremos o grupo C

C.

trs grupos de trs moedas cada, que chamaremos grupos

Colocaremos na balana os grupos

fora.

Podem acontecer duas coisas:

(a) Os pratos cam equilibrados.


(b) Os pratos cam desequilibrados.

A e B tm o mesmo peso. Logo,


a moeda mais leve deve estar no grupo C . No caso (b), um dos grupos
No caso (a), temos que os grupos

cou mais leve, o que signica que a moeda mais leve est neste grupo.
Assim, utilizando a balana apenas uma vez conseguiremos descobrir
qual o grupo em que a moeda mais leve est. Digamos que este grupo
seja o grupo

A.

Para achar a moeda mais leve, procedemos de modo

semelhante ao que zemos anteriormente: separamos as trs moedas

1.5

Solues dos Problemas da Seo 1.4

do grupo

21

colocando uma em cada prato e deixando a terceira de

fora. Podem acontecer duas coisas:

(a) Os pratos cam desequilibrados e assim a moeda mais leve est


no prato mais leve.
(b) Os pratos cam equilibrados, logo a moeda mais leve foi a que
cou fora.

No nal, usamos a balana exatamente duas vezes.

Soluo do Problema 1.9.

Como em muitos problemas de Mate-

mtica, abordar casos mais simples do problema pode ajudar bastante


na soluo.

Assim, vamos imaginar o seguinte caso mais simples:

na ilha existe somente uma pessoa de olhos azuis e a outra de olhos


verdes. Pensando neste caso, a pessoa que tinha olhos azuis s via as
que tinham olhos verdes.

Quando o estrangeiro armou que existia

uma pessoa de olhos azuis, ela descobriu que tinha olhos azuis, pois as
outras pessoas tinham olhos verdes. Assim, meia-noite ela subiu na
prefeitura e pulou. Com isso, a pessoa que tinha olhos verdes descobriu
que tinha olhos verdes, pois se ela tivesse olhos azuis sua companheira
no se suicidaria no dia anterior.
Vamos agora dar um passo crucial na soluo do nosso problema
original, considerando o caso onde existem duas pessoas de olhos azuis
e duas pessoas de olhos verdes na ilha. Vamos chamar as pessoas de
olhos azuis de

e as pessoas de olhos verdes de

D.

No dia

em que o estrangeiro fez o seu pronunciamento, nada aconteceu, pois


as pessoas

via a pessoa

pessoa

D
B

viam as pessoas

com olhos azuis e a pessoa

com olhos azuis e vice-versa. J no segundo dia, a

teve o seguinte pensamento:

22

Primeiros Passos

Se eu tivesse olhos verdes, a pessoa B teria descoberto que


tinha olhos azuis ontem, pois ela veria trs pessoas de olhos
verdes. Como ela no se suicidou ontem, eu tenho olhos
azuis.
Pensando da mesma forma, a pessoa

descobriu que tambm tinha

olhos azuis. Por isso, meia-noite do segundo dia, as pessoas

se suicidaram.
O que aconteceu depois? As pessoas

ainda tinham a dvida

da cor de seus olhos. Para chegar concluso de que seus olhos so


verdes, no terceiro dia, a pessoa

pensou assim:

Bem, se eu tivesse olhos azuis, as pessoas

veriam

cada uma duas pessoas com olho azul. Logo, elas no teriam se suicidado no segundo dia, pois no conseguiriam
deduzir a cor de seus olhos.

Logo, tenho olhos verdes.

Ufa!
Do mesmo modo, a pessoa

conseguiu descobrir a cor de seus olhos.

Analisando de modo semelhante, conseguiremos deduzir que no


problema original as cinco pessoas de olhos azuis descobriro que possuem olhos azuis e juntas se suicidaro no quinto dia aps o pronunciamento do estrangeiro.
Agora vamos descobrir a resposta da segunda pergunta do enunciado:

que informao nova o estrangeiro trouxe?

Aparentemente

nada de novo foi acrescentado pela frase do estrangeiro, pois cada


pessoa estava vendo alguma pessoa com olhos azuis. Mas isso no
verdade.
Para ver isso e descobrir qual a nova informao que o estrangeiro
trouxe, vamos voltar ao caso de somente duas pessoas na ilha, uma

1.5

Solues dos Problemas da Seo 1.4

23

de olhos azuis e outra de olhos verdes. Neste caso, a pessoa de olhos


azuis somente v uma pessoa de olhos verdes. Com a informao de
que existe uma pessoa de olhos azuis ela pode descobrir a cor de seus
olhos.

Note que a pessoa de olhos verdes j sabia que existia pelo

menos uma pessoa de olhos azuis.

Mas ela no sabia que a pessoa

de olhos azuis tinha conhecimento de que na ilha existia algum com


olhos azuis. Essa a nova informao que o estrangeiro trouxe.

Soluo do Problema 1.10.

Uma primeira soluo cortar a cor-

rente 30 vezes, separando todos os elos. Porm, essa no a melhor soluo, como veremos a seguir. Vamos iniciar nossa anlise observando
que para pagar o primeiro dia precisamos dar um corte na corrente.
Assim, o gerente receber um elo. O pulo do gato do problema vem
agora: para pagar o 2

dia, vamos cortar a corrente de modo a separar

dois elos de uma vez. Assim, daremos dois elos ao gerente e ele devolver um elo de troco. Com este elo pagaremos o terceiro dia. Note
que pagamos trs dias fazendo dois cortes na corrente, como mostra a
tabela:

Elos

Gerente

Viajante

1, 2

28

Note que o nmero 2 denota o pedao que contm 2 elos.


pagar o 4

Para

dia, cortaremos a corrente de modo a obter um pedao

com quatro elos.

Entregamos ao gerente este pedao e recebemos

de troco um elo solto e um pedao com dois elos.


pagamos o 5

dia. Assim, no 5

dia teremos os seguintes grupos de

elos:

Elos

Com o elo solto,

Gerente

Viajante

1, 4

2, 24

24

Assim, pagamos o 6

Primeiros Passos

dia com o pedao que contm dois elos e

receberemos o elo solto de troco. Finalmente pagaremos o 7

dia com

o elo solto. Note que foi possvel pagar 7 dias com apenas trs cortes na
corrente. A continuao do procedimento est quase revelada. Para
pagar o 8

dia, cortaremos um pedao com oito elos.

Daremos este

pedao e receberemos de troco 7 elos, sendo um elo solto, um pedao


com 4 e um pedao com dois elos. Repetindo o procedimento anterior,
pagaremos os 7 dias seguintes, pagando at o 15

dia sem precisar de

cortes adicionais. Ou seja, para pagar os 15 primeiros dias, precisamos


de 4 cortes na corrente. Neste momento, a corrente est distribuda
do seguinte modo:

Elos

Para pagar o 16

Gerente

Viajante

1, 2, 4, 8

16

dia, entregaremos ao gerente o pedao com os 16

elos restantes, recebendo 15 elos divididos em pedaos de 1, 2, 4 e 8


elos. Se repetirmos o processo, pagaremos o hotel at o 31

dia sem

precisar de novos cortes. Assim, o mnimo nmero de cortes 4.

Soluo do Problema 1.11.

Para resolver este problema vamos usar

a estratgia de mudar a representao . O que signica isso? Vamos


reescrever o problema com outros ingredientes, porm sem alterar em
nada sua essncia. Primeiramente, enumere as casas do tabuleiro com
os nmeros

1, 2, . . . , 9,

como na Figura 1.4.

Vamos agora associar ao tabuleiro, um conjunto de nove pontos


tambm enumerados com os nmeros 1, 2, . . . , 9. Se for possvel sair
de uma casa

e chegar casa

com apenas uma jogada do cavalo,

colocaremos um segmento ligando os pontos

j.

Por exemplo,

1.5

Solues dos Problemas da Seo 1.4

25

Figura 1.4: Tabuleiro de 9 casas

possvel, saindo da casa 1 chegar casa 6 e a casa 8.

Desse modo,

o ponto com nmero 1 est ligado com o ponto com nmero 8.

Se

analisarmos todas as possveis ligaes entre os pontos obteremos um


esquema com o mostrado na Figura 1.5

2
7

Figura 1.5: Conexes das casas

Figura 1.6: Tabuleiro (a)

Assim, se colocarmos os cavalos como no tabuleiro (a), teremos


a situao descrita na Figura 1.6. Deste modo, ca evidente que no
podemos trocar a posio dos cavalos branco e preto sem que em algum
momento eles ocupem a mesma casa.

26

1.6

Primeiros Passos

Exerccios

1. Uma sacola contm meias cujas cores so branca, preta, amarela


e azul. Sem olhar para a sacola, qual a quantidade mnima de
meias que precisamos retirar da mesma para garantir pelo menos
um par de meias da mesma cor?
2. O pai do padre lho nico de meu pai. O que eu sou do padre?
3. Numa mesa h 5 cartas:

Cada carta tem de um lado um nmero natural e do outro lado


uma letra. Joo arma: Qualquer carta que tenha uma vogal
tem um nmero par do outro lado.

Pedro provou que Joo

mente virando somente uma das cartas. Qual das 5 cartas foi a
que Pedro virou?
4. A polcia prende 4 homens, um dos quais culpado de um furto.
Eles fazem as seguintes declaraes:

Arnaldo: Bernaldo o culpvel.

Bernaldo: Cernaldo o culpvel.

Dernaldo: eu no sou culpvel.

Cernaldo: Bernaldo mente ao dizer que eu sou culpvel.

Se se sabe que s uma destas declaraes a verdadeira, quem


culpvel pelo furto?

1.6

Exerccios

27

5. Numa cidade existe uma pessoa

que sempre mente teras,

quintas e sbados e completamente sincera o resto dos dias


da semana.

Felipe chega um certo dia na cidade e mantm o

seguinte dilogo com a pessoa

X:

 Felipe: Que dia hoje?




X: Sbado.

 Felipe: Que dia ser amanh?




X: Quarta-feira.

Em qual dia da semana foi mantido este dilogo?

6. Divida o relgio de parede abaixo em 6 partes iguais de forma tal


que a soma das horas que cam em cada parte seja a mesma.n

11
10

9
8

12

2
3

7. Joo adora Gabriela, que uma aluna excelente em Matemtica.


Joo armou um plano para dar um beijo nela, e descobriu que
poder fazer isso apenas dizendo uma frase. Que frase essa?

8. No plano se colocam 187 rodas dentadas do mesmo dimetro,


enumeradas de 1 at 187. A roda 1 acoplada com a roda 2, a 2
com a 3,

...,

a 186 com a 187 e esta ltima com a roda 1. Pode

tal sistema girar?

28

Primeiros Passos

9. Um canal, em forma quadrada, de 4 metros de largura rodeia um


castelo. A ponte do castelo est fechada e um intruso quer entrar
no castelo usando duas pranchas de 3,5 metros de comprimento.
Ser que o intruso consegue?

10. Os nmeros

1, 2, 3, . . . , 99 so escritos no quadro-negro e permi-

tido realizar a seguinte operao: apagar dois deles e substitulos pela diferena do maior com o menor. Fazemos esta operao
sucessivamente at restar apenas um ltimo nmero no quadro.
Pode o ltimo nmero que restou ser o zero?

11. Algum elege dois nmeros, no necessariamente distintos, no


conjunto de nmeros naturais

2, . . . , 20.

O valor da soma destes

A) e o valor do produto dos


nmeros dado unicamente a Karla ( K).
nmeros dado somente a Adriano (

 Pelo telefone

A diz a K: No possvel que descubras minha

soma.
 Uma hora mais tarde,

K lhe diz a A: Ah!

sabendo disso, j

sei quanto vale tua soma!


 Mais tarde

chama outra vez a

e lhe informa: Poxa,

agora eu tambm conheo teu produto!


Quais nmeros foram eleitos?

12. possvel cobrir um tabuleiro de xadrez com 31 domins onde


removemos as casas dos vrtices superior esquerdo e inferior direito?

13. Num saco encontram-se 64 moedas leves e 64 moedas pesadas.

1.6

Exerccios

29

possvel separar duas moedas de pesos diferentes com 7 pesagens?


14. Quantas vezes precisamos dobrar um papel de 1mm de espessura
para que a altura da pilha chegue da Terra Lua?
15. Descubra o erro da prova da armao abaixo:

Armao: Trs igual a dois.


Seja

um nmero diferente de zero. Temos que:

3x 3x = 2x 2x.
Colocando

xx

em evidncia, temos que:

3(x x) = 2(x x).


Cancelando

xx

em ambos os lados, obtemos que

3 = 2.

30

Primeiros Passos

2
Equaes e Inequaes
lgebra generosa; ela geralmente nos d mais do que lhe pedimos.
D'Alembert

Na antiguidade, todo conhecimento matemtico era passado de


gerao para gerao atravs de receitas. A falta de smbolos e notao
adequada complicava substancialmente a vida de quem precisava usar
a Matemtica e de quem apreciava sua beleza. Por exemplo, o uso de
letras (x,

y, z

etc.) para representar nmeros desconhecidos no tinha

sido inventado ainda.

Isso s veio ocorrer por volta dos meados do

sculo XVI, ou seja, a menos de 500 anos atrs.


Apesar disso, o conhecimento matemtico das antigas civilizaes
era surpreendente. Os egpcios, babilnios, gregos e vrios outros povos tinham o domnio de mtodos e tcnicas que so empregados hoje,
como solues de equaes do primeiro e segundo graus, inteiros que
so soma de quadrados e vrios outros conhecimentos. Especialmente
os gregos, cuja cultura matemtica resistiu aos tempos com a preservao de Os Elementos de Euclides , tinham desenvolvido e catalisado

31

32

Equaes e Inequaes

muitos dos avanos da poca.


Entretanto, todos os resultados tinham uma linguagem atravs dos
elementos de geometria, mesmo aqueles que s envolviam propriedades sobre os nmeros. Essa diculdade deve-se em parte aos sistemas
de numerao que eram utilizados pelos gregos e, posteriormente, pelos romanos, que eram muito pouco prticos para realizar operaes
matemticas.
Por volta de 1.100, viveu na ndia Bhaskara, um dos mais importantes matemticos de sua poca. Apesar de suas contribuies terem
sido muito profundas na Matemtica, incluindo-se a resultados sobre
equaes diofantinas, tudo indica que Bhaskara no foi o primeiro a
descobrir a frmula, que no Brasil chamamos de frmula de Bhaskara,
assim como Pitgoras no deve ter sido o primeiro a descobrir o teorema que leva o seu nome, j que 3.000 a.c.

os babilnios tinham

conhecimento de ternas pitagricas de nmeros inteiros bem grandes.


Apesar de ter conhecimento de como solucionar uma equao do
segundo grau, a frmula que Bhaskara usava no era exatamente igual
a que usamos hoje em dia, sendo mais uma receita de como encontrar
as razes de uma equao.

Para encontrar essas razes, os indianos

usavam a seguinte regra:

Multiplique ambos os membros da equao pelo nmero que vale


quatro vezes o coeciente do quadrado e some a eles um nmero igual
ao quadrado do coeciente original da incgnita. A soluo desejada
a raiz quadrada disso.
O uso de letras para representar as quantidades desconhecidas s
veio a se tornar mais popular com os rabes, que tambm desenvolveram um outro sistema de numerao, conhecido como indo-arbico .
Destaca-se tambm a participao do matemtico francs Franois

2.1

Equaes do Primeiro Grau

33

Viti, que aprimorou esse uso dos smbolos algbricos em sua obra In

artem analyticam isagoge e desenvolveu um outro mtodo para resolver a equao do segundo grau.
Na seo seguinte estudaremos com detalhe a equao do primeiro
grau, e como podemos utiliz-la para resolver alguns problemas em
Matemtica.

2.1

Equaes do Primeiro Grau

Iniciamos nossa discusso resolvendo o seguinte problema:

Exemplo 2.1.

Qual o nmero cujo dobro somado com sua quinta

parte igual a 121?

x, para
2x e sua

Soluo: Vamos utilizar uma letra qualquer, digamos a letra


designar esse nmero desconhecido. Assim, o dobro de
quinta parte

x/5.

Logo, usando as informaes do enunciado, obte-

mos que:

2x +

x
= 121,
5

ou ainda,

10x + x = 605,
onde

11x = 605.

Resolvendo, temos que

x = 605/11 = 55.

Se voc j teve contato com o procedimento de resoluo do exemplo acima, notou que o principal ingrediente a equao do primeiro

grau em uma varivel.

Denio 2.2.

Uma equao do primeiro grau na varivel

expresso da forma

ax + b = 0,

uma

34

onde

a 6= 0, b R

Equaes e Inequaes

um nmero real a ser encontrado.

Por exemplo, as seguintes equaes so do primeiro grau:

(a)

2x 3 = 0.

(b)

4x + 1 = 0.

(b)

3
x = 0.
2

Para trabalhar com equaes e resolv-las, vamos pensar no modelo da balana de dois pratos. Quando colocamos dois objetos com
o mesmo peso em cada prato da balana, os pratos se equilibram.
Quando os pratos esto equilibrados, podemos adicionar ou retirar a
mesma quantidade de ambos os pratos, que ainda assim eles permanecero equilibrados. Essa uma das principais propriedades quando
estamos trabalhando com uma equao. Em geral, para resolver uma
equao, utilizamos as seguintes propriedades da igualdade entre dois
nmeros:

Propriedade 1.

Se dois nmeros so iguais, ao adicionarmos a

mesma quantidade a cada um destes nmeros, eles ainda permanecem iguais. Em outras palavras, escrevendo em termos de letras, se
e

so dois nmeros iguais, ento

a=b

Note que podemos tomar

a+c

igual a

b + c,

ou seja,

a + c = b + c.

um nmero negativo, o que signica

que estamos subtraindo a mesma quantidade dos dois nmeros. Por


exemplo, se

um nmero qualquer que satisfaz

5x 3 = 6,

2.1

Equaes do Primeiro Grau

somando-se

35

3 a ambos os lados da equao acima, obtemos que x deve

satisfazer:

(5x 3) + 3 = 6 + 3,

Propriedade 2.

ou seja,

5x = 9.

Se dois nmeros so iguais, ao multiplicarmos a

mesma quantidade por cada um destes nmeros, eles ainda permanecem iguais. Em outras palavras, escrevendo em termos de letras, se
e

so dois nmeros iguais, ento

a=b
Por exemplo, se
igualdade por

1/5

5x = 9

ac

igual a

b c,

ou seja,

ac = bc.

podemos multiplicar ambos os lados da

para obter

x=

5x
9
= ,
5
5

encontrando o nmero que satisfaz a equao

5x 3 = 6.

Para nos familiarizarmos um pouco mais com a linguagem das


equaes, vamos pensar no seguinte problema:

Exemplo 2.3.

Para impressionar Pedro, Lucas props a seguinte

brincadeira:
- Escolha um nmero qualquer.
- J escolhi, disse Pedro.
- Multiplique este nmero por 6. A seguir, some 12. Divida o que
voc obteve por 3. Subtraia o dobro do nmero que voc escolheu. O
que sobrou igual a 4!
Pedro realmente cou impressionado com a habilidade de Lucas.
Mas no h nada de mgico nisso. Voc consegue explicar o que Lucas
fez?

36

Equaes e Inequaes

Soluo: Na verdade, Lucas tinha conhecimento de como operar com


equaes. Vamos ver o que Lucas fez de perto, passo a passo, utilizando a linguagem das equaes. Para isso, vamos chamar a quantidade que Pedro escolheu de

x:

Escolha um nmero:

Multiplique este nmero por 6:

A seguir, some 12:

Divida o que voc obteve por 3:

O que sobrou igual a 4!

x.
6x.

6x + 12.

6x + 12
= 2x + 4.
3
Subtraia o dobro do nmero que voc escolheu: 2x + 4 2x = 4.

Observao 2.4.

Devemos ter cuidado na hora de efetuar divises em

ambos os lados de uma equao, para no cometer o erro de dividir


os lados de uma igualdade por zero. Por exemplo, podemos dar uma
prova (obviamente) falsa de que

1 = 2,

utilizando o seguinte tipo de

argumento: sempre verdade que

x + 2x = 2x + x.
Logo,

x x = 2x 2x
Colocando

(x x)

em evidncia:

1(x x) = 2(x x)
Dividindo por

1 = 2.

(x x)

Qual o erro?

os dois lados da igualdade acima, temos que

2.1

Equaes do Primeiro Grau

37

Para encontrar a soluo da equao

ax + b = 0,

procedemos do

seguinte modo:

Somamos

a ambos os lados da equao, obtendo

ax + b + (b) = 0 + (b) ax = b.
Note que como somamos a mesma quantidade aos dois lados da
equao, ela no se alterou.

Dividimos os dois lados da equao por


altera a igualdade e nos d que o valor

a 6= 0. Isso tambm no
de x :

b
b
ax
=
x = .
a
a
a
Assim, a soluo da equao

ax + b = 0

b
x= .
a

2.1.1 Problemas Resolvidos


Vamos ver agora alguns problemas que podem ser solucionados utilizando as equaes do primeiro grau.

Problema 2.5.

Se

representa um dgito na base 10 e a soma

x11 + 11x + 1x1 = 777,


quem

x?

38

Equaes e Inequaes

Soluo: Para resolver este problema, precisamos nos recordar que se

abc

a escrita de um nmero qualquer na base

igual a

102 a + 10b + c.

10,

ento esse nmero

Assim, temos que

x11 = 100x + 11
11x = 110 + x
1x1 = 101 + 10x
Logo, temos a seguinte equao do primeiro grau:

100x + 11 + 110 + x + 101 + 10x = 777


Logo,

x=

Problema 2.6.

ou

111x + 222 = 777

777 222
= 5.
111

Determine se possvel completar o preenchimento

do tabuleiro abaixo com os nmeros naturais de

9,

sem repetio,

de modo que a soma de qualquer linha seja igual a de qualquer coluna


ou diagonal.

6
9

Soluo: Primeiro, observe que a soma de todos os nmeros naturais


de

45. Assim, se denotamos por

o valor comum da soma dos

elementos de uma linha, somando as trs linhas do tabuleiro, temos


que:

45 = 1 + 2 + + 9 = 3s,
Onde

deve ser igual a

15.

Assim, chamando de

primeira linha que falta ser preenchido,

o elemento da

2.1

Equaes do Primeiro Grau

39

1 + x + 6 = 15. Logo, x = 8. Assim, observando a coluna


contm 8 e 9, temos que sua soma maior que 15. Logo, no

temos que
que

possvel preencher o tabuleiro de modo que todas as linhas e colunas


tenham a mesma soma.
Os quadrados de nmeros com essas propriedades se chamam qua-

drados mgicos . Tente fazer um quadrado mgico. Voc j deve ter


percebido que no centro do quadrado no podemos colocar o nmero

9.

De fato, vamos descobrir no exemplo abaixo qual o nmero que

deve ser colocado no centro de um quadrado mgico.

Problema 2.7.

Descubra os valores de

de modo que seja possvel

completar o preenchimento do quadrado mgico abaixo:

x
Soluo: Para descobrir

x,

vamos utilizar o fato de que a soma de

qualquer linha, coluna ou diagonal igual a

15,

j obtido no exemplo

anterior. Se somarmos todas as linhas, colunas e diagonais que contm

x,

teremos que a soma ser

4 15 = 60,

pois existem exatamente uma

linha, uma coluna e duas diagonais que contm

x.

Note tambm que

cada elemento do quadrado mgico ser somado exatamente uma vez,


exceto

que ser somado quatro vezes. Assim:

1 + 2 + 3 + 4 + + 9 + 3x = 60,
onde temos que

45 + 3x = 60

e consequentemente

x = 5.

40

Equaes e Inequaes

O problema a seguir um fato curioso que desperta nossa ateno


para como a nossa intuio s vezes falha.

Problema 2.8.

Imagine que voc possui um o de cobre extrema-

mente longo, mas to longo que voc consegue dar a volta na Terra
com ele. Para simplicar a nossa vida e nossas contas, vamos supor
que a Terra uma bola redonda (o que no exatamente verdade)
sem nenhuma montanha ou depresso e que seu raio de exatamente

6.378.000

metros.

O o com seus milhes de metros est ajustado Terra, cando


bem colado ao cho ao longo do equador.

Digamos agora que voc

acrescente 1 metro ao o e o molde de modo que ele forme um crculo


enorme, cujo raio um pouco maior que o raio da Terra e tenha o
mesmo centro. Voc acha que essa folga ser de que tamanho?
Nossa intuio nos leva a acreditar que como aumentamos to
pouco o o, a folga que ele vai ter ser tambm muito pequena, digamos alguns poucos milmetros. Mas veremos que isso est completamente errado!

Soluo. Utilizaremos para isso a frmula que diz que o comprimento

de um crculo de raio

C = 2r,
(l-se pi ) um nmero irracional que vale aproximadamente
3, 1416 (veja a observao a seguir).
De fato, o comprimento da Terra CT calculado com essa frmula

onde

aproximadamente:

CT = 2rT
= 2 3, 1415 6.378.000 = 40.072.974

metros,

2.1

Equaes do Primeiro Grau

onde

rT

41

o raio da Terra.

x o tamanho da folga obtida em metros e rf o raio


do o, temos que a folga ser igual a x = rf rT . Logo, basta calcular
rf . Por um lado, o comprimento do o igual a CT + 1 = 40.072.975.
Se chamamos de

Logo,

40.072.975 = 2rf

onde

rf =

Fazendo o clculo acima, temos que

6.378.000, 16
rf rT = 0, 16
a

metros.

Assim,

rf

40.072.975
.
2
aproximadamente igual

aproximadamente igual a

x =

metros, ou seja, 16 centmetros!

Observao 2.9.

Vale observar que a folga obtida aumentando o o

no depende do raio em considerao. Por exemplo, se repetssemos


esse processo envolvendo a Lua em vez da Terra, obteramos que ao
aumentar o o em um metro, a folga obtida seria dos mesmos 16
centmetros. Verique isso!

Observao 2.10.

De fato, podemos denir (e calcular!) o nmero

de vrias maneiras prticas. Vamos considerar dois experimentos

(que se voc no conhece

Experimento 1:

deve fazer):

Pegar um cinto e fazer um crculo com ele. Calcule

o comprimento do cinturo e divida pelo dimetro do crculo obtido.

Experimento 2:

Pegar uma tampa de uma lata e medir o compri-

mento do crculo da tampa e dividir pelo dimetro da tampa.


Se voc efetuou os clculos acima com capricho, deve ter notado
que o nmero obtido aproximadamente o mesmo. Se nossos crculos
fossem perfeitos (eles nunca so: sempre tm algumas imperfeies)
obteramos o nmero

Uma aproximao para

= 3, 1415926535897932384626433832795.

42

2.2

Equaes e Inequaes

Sistemas de Equaes do Primeiro Grau

Nesta seo iremos discutir situaes onde queremos descobrir mais de


uma quantidade, que se relacionam de modo linear, ou seja, atravs
de equaes do primeiro grau. Por exemplo, considere o seguinte problema:

Exemplo 2.11.

Joo possui 14 reais e deseja gastar esse dinheiro em

chocolates e sanduches para distribuir com seus 6 amigos, de modo


que cada um que exatamente com um chocolate ou um sanduche.
Sabendo que cada chocolate custa 2 reais e cada sanduche custa 3
reais, quantos chocolates e sanduches Joo deve comprar?
Para resolver esse problema, vamos chamar de
chocolates que Joo deve comprar e

a quantidade de

o nmero de sanduches. Assim,

como Joo deseja gastar 14 reais, temos que

2x + 3y = 14.

(2.1)

Como Joo comprar exatamente 6 guloseimas, uma para cada amigo,


temos que

x + y = 6.

(2.2)

Note que no encontramos uma equao do primeiro grau em uma


varivel e sim duas equaes do primeiro grau em duas variveis. Esse
um caso particular de um sistema de equaes do primeiro grau em
vrias variveis.

Denio 2.12.
. . . , xn

Uma equao do primeiro grau nas variveis

uma expresso da forma

a1 x1 + a2 x2 + + an xn + b = 0,

x1 , x2 ,

2.2

Sistemas de Equaes do Primeiro Grau

onde os nmeros

a1 , a2 , . . . , an

43

so diferentes de zero e

um nmero

real.
Por exemplo,

2x 3y = 0
uma equao do primeiro grau nas variveis

2a b +

y.

Assim como,

c
=5
3

uma equao do primeiro grau nas variveis

a, b

c.

(r1 , r2 , . . . , rn ) formam uma soluo da


equao, se substituindo x1 por r1 , x2 por r2 , . . . , xn por rn , temos
que a equao acima satisfeita, isto , a1 r1 +a2 r2 + +an rn +b = 0.
Por exemplo, (3, 2) uma soluo da equao 2x 3y = 0 acima,
Dizemos que os nmeros

pois

2 3 3 2 = 0.
(2, 3)
no soluo da equao 2x 3y = 0, j que 2 2 3 3 = 5 6= 0.
c
Do mesmo modo, (2, 0, 3) soluo da equao 2a b +
= 5, pois
3
3
2 2 0 + = 5.
3
Denio 2.13. Um sistema de equaes do primeiro grau em n
variveis x1 , x2 , . . ., xn um conjunto de k equaes do primeiro
grau em algumas das variveis x1 , x2 , . . . , xn , isto , tem-se o seguinte
Note que a ordem que apresentamos os nmeros importa, pois

conjunto de equaes

a11 x1 + a12 x2 + + a1n xn + b1 = 0,

a x + a x + + a x + b = 0,
21 1
22 2
2n n
2

ak1 x1 + ak2 x2 + + akn xn + bk = 0,

(2.3)

44

onde alguns dos elementos

Equaes e Inequaes

aij (1 i k, 1 j n)

podem ser zero.

Porm, em cada uma das equaes do sistema algum coeciente


diferente de zero e, alm disso, cada varivel

xj

aij

aparece em alguma

equao com coeciente distinto de zero.

(r1 , r2 , . . . , rn ) formam uma soluo do


se (r1 , r2 , . . . , rn ) soluo para todas as

Dizemos que os nmeros


sistema de equaes (2.3)
equaes simultaneamente.

Quando resolvemos um sistema de equaes do primeiro grau, podem acontecer trs situaes:

(a) o sistema tem uma nica soluo;

(b) o sistema tem uma innidade de solues;

(c) o sistema no possui soluo.

A seguir ilustramos com exemplos cada uma das situaes acima.

Situao (a):

Retomamos o sistema proposto no Exemplo 2.11, o

qual se encaixa neste caso.

2x + 3y = 14,
x + y = 6.
Isolamos o valor de uma das variveis numa das equaes. Por convenincia nos clculos isolamos o valor de

x na segunda equao, obtendo:

x = 6 y.
A seguir, substitumos esse valor na outra equao, obtendo uma equa-

2.2

Sistemas de Equaes do Primeiro Grau

45

o do primeiro grau. Resolvendo temos:

2(6 y) + 3y = 14,
12 2y + 3y = 14,

y = 2.

Assim,

y = 2.

Imediatamente, encontramos o valor de

Vamos agora resolver alguns problemas semelhantes.

Situao (b):
x, y

x = 6 2 = 4.

Consideremos os sistema de primeiro grau nas variveis

dado por

x + y z 1 = 0,

(2.4)

x y 1 = 0.
Da segunda equao segue-se que

x = y + 1.

(2.5)

Substituindo esta expresso na primeira equao obtemos

(y + 1) + y z 1 = 0,

2y z = 0,

z = 2y.

Notemos que as variveis

xez

so resolvidas em funo da varivel

a qual no possui nenhuma restrio, de modo que se


valor real
valor

t.

ento

(2.6)

y,

assumir um

cam automaticamente determinadas por este

Isto , para todo

real, de (2.5) e (2.6) tem-se que

x = t + 1,

y = t,

z = 2t

soluo do sistema (2.4) e, portanto, temos innitas solues para


este.

46

Situao (c):
variveis

x, y

Equaes e Inequaes

Consideremos agora o sistema de primeiro grau nas

dado por

x + y + 2z 1 = 0,
x + z 2 = 0,

y + z 3 = 0.

(2.7)

Neste caso, da segunda e da terceira equao segue-se que

x=2z

y = 3 z.

Substituindo estas expresses na primeira equao obtm-se

(2 z) + (3 z) + 2z 1 = 0 4 = 0,
o que uma incompatibilidade. Logo, este sistema no tem soluo.

Observao 2.14. Os sistemas de equaes de primeiro grau so tambm conhecidos como sistemas de equaes lineares. Quando um sistema de equaes lineares envolve muitas variveis no to fcil
resolv-lo se no se organiza com cuidado seu processo de resoluo.
Existe uma teoria bem conhecida e amplamente divulgada sobre mtodos de resoluo para esse tipo de sistemas. Um dos mtodos mais
usado e eciente para resolver sistemas lineares o mtodo de eliminao gaussiana. O leitor interessado pode consultar [7].

2.2.1 Problemas Resolvidos


O problema a seguir foi proposto na primeira fase da Olimpada Brasileira de Matemtica.

2.2

Sistemas de Equaes do Primeiro Grau

Problema 2.15.

47

Passarinhos brincam em volta de uma velha rvore.

Se dois passarinhos pousam em cada galho, um passarinho ca voando.


Se todos os passarinhos pousam, com trs em cada galho, um galho ca
vazio. Quantos so os passarinhos?
Soluo: Vamos chamar de

o nmero de passarinhos e

o nmero

de galhos da rvore. Temos que se dois passarinhos pousam em cada


galho, um passarinho ca voando, ou seja,

2g = p 1.
Alm disso, se todos os passarinhos pousam, com trs em um mesmo
galho, um galho ca vazio:

3(g 1) = p.
Substituindo na equao anterior, temos que
segue-se que

g=4

Problema 2.16.

p = 9.

Quanto medem as reas

A1

2g = 3g 3 1,
e

A2

onde

na gura abaixo,

sabendo que o quadrado tem lado 1 e as curvas so arcos de crculos


com centros nos vrtices

V1

V2

do quadrado, respectivamente.

V2
A2
A1

V
Soluo: Aplicando relaes de reas na gura temos que

A1 + A2 = 4 ,
A1 + 2A2 = 1,

48

Equaes e Inequaes

ou seja, chegamos a um sistema de equaes do primeiro grau com


duas incgnitas

A1

A2 .

Da primeira equao temos que

A1 =

A2 ;
4

substituindo esta na segunda equao obtemos

A2 + 2A2 = 1,
4
de onde

Logo,

A2 = 1

Problema 2.17.

+ A2 = 1.
4

A1 = 4 1 4 =

1.

Carlos e Cludio so dois irmos temperamentais

que trabalham carregando e descarregando caminhes de cimento. Para


Carlos e Cludio tanto faz carregar ou descarregar o caminho, o trabalho realizado por eles o mesmo. Quando esto de bem, trabalham
juntos e conseguem carregar um caminho em 15 minutos. Cludio
mais forte e trabalha mais rpido conseguindo carregar sozinho um
caminho em 20 minutos.
(a) Um dia, Cludio adoeceu e Carlos teve que carregar os caminhes
sozinho. Quanto tempo ele leva para carregar cada um?
(b) Quando os dois brigam, Carlos costuma se vingar descarregando
o caminho, enquanto Cludio o carrega com sacos de cimento.
Quanto tempo Cludio levaria para carregar o caminho com
Carlos descarregando?
Soluo: Vamos chamar de
rega por minuto e

a quantidade de sacos que Cludio car-

a quantidade de sacos que Carlos carrega por

2.3

Equao do Segundo Grau

49

minuto. Como Cludio carrega mais que Carlos, sabemos que

y < x.

Do enunciado, sabemos que os dois juntos carregam um caminho em


15 minutos. Se um caminho tem capacidade para

c sacos, temos que:

15x + 15y = c.
Alm disso, sabemos que Cludio sozinho carrega o mesmo caminho
em

20

minutos. Logo,

20x = c.
Assim, igualando as duas equaes, temos que

15x + 15y = 20x,

onde

Logo, dividindo ambos os lados por

15y = 20x 15x = 5x.


5, temos que 3y = x.

Assim, Clu-

dio carrega trs vezes mais sacos que Carlos e a resposta do primeiro
item

20 3

minutos, j que

60y = 20 3y = 20x = c.

Para descobrir quanto tempo os dois levam para carregar o caminho quando esto brigados, observamos que a cada minuto eles carregam

xy

minutos, j que

2.3

3y y = 2y
30 2y = 60y = c.

sacos, ou seja,

sacos. Logo, precisam de

30

Equao do Segundo Grau

Como j mencionamos em nossa introduo, o conhecimento de mtodos para solucionar as equaes do segundo grau remonta s civilizaes da antiguidade, como os babilnios e egpcios. Apesar disso,
a frmula que conhecemos por frmula de Bhaskara , em homenagem
ao matemtico indiano de mesmo nome e que determina as solues
de uma equao do segundo grau, s veio a aparecer do modo que
usamos muito mais tarde, com o francs Viti.

Nesta seo iremos

deduzir esta frmula e aplic-la a alguns problemas interessantes.

50

Equaes e Inequaes

2.3.1 Completando Quadrados


Um modo de resolver uma equao do segundo grau o mtodo de

completar quadrados. Ele consiste em escrever a equao numa forma


equivalente que nos permita concluir quais so as solues diretamente.
Vamos ilustrar isso com um exemplo, resolvendo a equao

x2 6x 8 = 0.
Podemos escrever essa equao como:

x2 6x = 8.
Somando

(x 3)

Logo,

ao lado esquerdo, obtemos

x2 6x + 9

que o mesmo que

. Assim, somando 9 a ambos os lados da equao, obtemos:

x3=

(x 3)2 = 9 + 8 = 17.

x 3 = 17. Logo, as solues

x1 = 3 + 17 e x2 = 3 17.

17

Denio 2.18.

ou

so:

A equao do segundo grau com coecientes

a, b

uma expresso da forma:

ax2 + bx + c = 0,
onde

a 6= 0, b, c R

(2.8)

uma varivel real a ser determinada.

Para encontrar as solues desta equao, vamos proceder do seguinte modo: isolando o termo que no contm a varivel
direito da igualdade na equao (2.8)

ax2 + bx = c

do lado

2.3

Equao do Segundo Grau

e dividindo os dois lados por

51

a,

obtemos:

b
c
x2 + x =
.
a
a
Agora vamos acrescentar um nmero em ambos os lados da equao acima, de modo que o lado esquerdo da igualdade seja um quadrado perfeito. Para isso, observe que necessrio adicionar

b2
aos
4a2

dois lados da igualdade. Assim, temos que:

 2

2
b
c
b2 4ac
b
b
b2
2
.
x+
=x +2 x+
= 2 =
2a
2a
2a
4a
a
4a2
b2 4ac de discriminante da equao
maiscula (l-se delta ) do alfabeto

Em geral, chamamos a expresso


(2.8) e denotamos pela letra

grego. Assim, podemos escrever a igualdade anterior como:

b
x+
2a

2
=

b2 4ac
= 2.
2
4a
4a

(2.9)

Por isso, para que exista algum nmero real satisfazendo a igualdade acima, devemos ter que

0,

j que o termo da esquerda na

igualdade maior ou igual a zero. Extraindo a raiz quadrada quando

0,

temos as solues:

b
x+
=
2a

b2 4ac
2a

b
b2 4ac
x+
=
.
2a
2a

Assim, obtemos as seguintes solues:

b
x1 = +
2a
e

b2 4ac
b +
=
2a
2a

52

b
x2 =
2a

Equaes e Inequaes

b2 4ac
b
=
.
4a2
2a

Em resumo,

Se

>0

existem duas solues reais.

Se

=0

s existe uma soluo real ( x1

Se

<0

no existe soluo real.

= x2 = b/2a).

A seguir apresentamos alguns exemplos.

Exemplo 2.19.

Encontre as solues da equao

Soluo: Observe que

a = 2, b = 4

c = 2.

2x2 4x + 2 = 0.

Logo,

= b2 4ac = (4)2 4 2 2 = 0.
Assim, a nica soluo

Exemplo 2.20.

x=

b
4
= = 1.
2a
4

Encontre as razes da seguinte equao do segundo

grau:

x2 x 1 = 0.
Soluo: Basta aplicarmos diretamente a frmula que acabamos de
deduzir. Como

a = 1, b = 1

c = 1,

calculando

temos:

= b2 4ac = (1)2 4 1 (1) = 5.


Logo, as solues so

b +
1+ 5
=
x1 =
2a
2

b
1 5
x2 =
=
.
2a
2

2.3

Equao do Segundo Grau

Exemplo 2.21.

Sabendo que

53

um nmero real que satisfaz

x=1+

1+
determine os valores possveis de

1
x

x.

Soluo: A soluo desse problema consiste numa simples manipulao algbrica, que feita com cuidado nos levar a uma equao do
segundo grau. Com efeito,

1+
Logo,

1+

1
x+1
=
.
x
x

=1+

1
x
1 + 2x
,
x=
1+x
1+

Ento devemos ter

x
1 + 2x
=
.
1+x
1+x

de onde segue-se que

x2 + x = 1 + 2x x2 x 1 = 0.
Resolvendo a equao tem-se

1+ 5
x1 =
2

Observao 2.22.

O nmero

(1 +

1 5
x2 =
.
2

5)/2 chamado de razo urea.

Este nmero recebe essa denominao pois, frequentemente, as propores mais belas e que a natureza nos proporciona esto prximas
da razo urea. Por exemplo, no arranjo das ptalas de uma rosa, nas
espirais que aparecem no abacaxi, na arquitetura do Parthenon, nos
quadros de da Vinci e nos ancestrais de um zango podemos encontrar a razo urea.

54

Equaes e Inequaes

O problema a seguir est relacionado com a seqncia de Fibonacci


e com a razo urea.

Dizemos que uma seqncia de nmeros

satisfaz a relao de Fibonacci se, para todo

n 0,

temos que

an+2 = an+1 + an .

Exemplo 2.23.
para algum

x 6= 0

forma

(2.10)

Encontre todas as sequncias

an

da forma

an = x n

que satisfazem a relao de Fibonacci.

Soluo: Sabendo que


n

an

an

satisfaz a relao de Fibonacci e que

, podemos concluir que para todo

n0

an

da

tem-se

xn+2 xn+1 xn = 0.
Colocando

xn

em evidncia na equao acima, temos que

xn (x2 x 1) = 0
xn = 0 ou x2 x 1 = 0. Como
2
portanto, x x 1 = 0. Observando a

Logo, temos duas possibilidades:

x 6= 0,

temos que

xn 6= 0

e,

soluo do Exemplo 2.20 temos que as nicas sequncias so

an =

!n
1+ 5
2

ou

an =

!n
1 5
.
2

Observao 2.24. Se an e bn satisfazem a relao de Fibonacci (2.10),


ento dados nmeros reais

bn
com

qualquer sequncia da forma

an +

satisfaz a relao. Pode-se provar que as sequncias dessa forma,

an = xn1

bn = xn2

calculados anteriormente, so as nicas

sequncias que satisfazem a relao. Veja, por exemplo, [4].

2.3

Equao do Segundo Grau

55

2.3.2 Relao entre Coecientes e Razes


ax2 + bx + c = 0, com a 6= 0, j calculamos explicitamente as suas razes x1 e x2 . Vamos estabelecer agora as relaes entre
a, b e c e as razes x1 e x2 . Vamos supor 0. Como j sabemos,
Dada a equao

temos que

Assim,

b +
b
x1 =
e
x2 =
.
2a
2a
somando x1 com x2 tem-se

2b
b
b + b
+
=
= .
x1 + x2 =
2a
2a
2a
a

Por outro lado, fazendo o produto

x1 x2

obtemos

!
!
b +
b
x1 x2 =

2a
2a


b+ b
b2
=
=
4a2
4a2
4ac
c
= 2 = .
4a
a
Em particular, quando

Teorema 2.25.

a = 1,

Os nmeros

(2.11)

(2.12)

temos o seguinte resultado.

so as razes da equao

x2 sx + p = 0

(2.13)

se, e somente se,

+ =s

= p.

(2.14)

56

Demonstrao. Com efeito, se

Equaes e Inequaes

so as razes de (2.13) ento os

clculos feitos em (2.11) e (2.12) nos do (2.14). Reciprocamente, se


vale (2.14) ento da igualdade

(x )(x ) = x2 sx + p
segue-se que

so as razes de (2.13).

Observao 2.26.

ax2 + bx + c = 0, com
a 6= 0, podemos escrev-la como a(x2 sx + p) = 0, com s = b/a e
p = c/a. Supondo que a equao x2 sx + p = 0 tem razes e , a
Em geral, dada a equao

igualdade

ax2 + bx + c = a(x2 sx + p) = a(x )(x )


nos permite concluir que
grau

(2.15)

so as razes da equao de segundo

ax + bx + c = 0.

A equao (2.15) mostra que se

raiz de um polinmio do se-

gundo grau, ento a diviso desse polinmio pelo polinmio

(x )

uma diviso exata. Voltaremos a tratar desse assunto no Teorema 8.5.

Exemplo 2.27.

Paulo cercou uma regio retangular de rea 28

m2

com 24 metros de corda. Encontre as dimenses dessa regio.


Soluo: Se chamamos de

os lados do retngulo construdo por

Paulo, as condies sobre o permetro e a rea desse retngulo nos


levam s seguintes equaes:

Como j observamos,

a + b = 12,
ab = 28.
x2 12x + 28 = 0.
= 122 428 = 32. Utilizando

so razes da equao

Calculando o discriminante, obtemos

2.3

Equao do Segundo Grau

57

a frmula, temos que as solues so

12 + 32
a=
=6+2 2
2
e

12 32
b=
= 6 2 2.
2

Exemplo 2.28.

Mostre que a equao

raiz inteira positiva, se

x2 + bx + 17 = 0

no possui

um inteiro no negativo.

n pon + m = b,

Soluo: Suponhamos que a equao possui alguma raiz inteira

m a outra raiz (podendo ser m = n). Ento,


onde m = n b dever ser necessariamente um nmero inteiro. Por
outro lado, m e n so nmeros inteiros tais que m n = 17, o que s
possvel se m = 1 ou n = 1, o que nos daria em qualquer um dos
casos que 1 + b + 17 = 0 (b = 18), sendo isto uma contradio com
o fato de b ser inteiro no negativo.
sitiva e seja

Exemplo 2.29.

Numa reunio havia pelo menos 12 pessoas e todos

os presentes apertaram as mos entre si. Descubra quantas pessoas


estavam presentes na festa, sabendo que houve menos que 75 apertos
de mo.

aperto de mo associaremos

a o nmero de apertos de mo e enumerar


do conjunto P = {1, 2, . . . , n}. A cada
um par (i, j), signicando que a pessoa i

j.

Assim, os apertos de mo envolvendo a

Soluo: Vamos denotar por


as pessoas com os nmeros

apertou a mo da pessoa
pessoa

foram



A1 = (1, 2), (1, 3), . . . , (1, n) .

58

Equaes e Inequaes

Do mesmo modo, denimos os apertos de mo envolvendo a pessoa


que no envolvem a pessoa

1,

como



A2 = (2, 3), (2, 4), . . . , (2, n) .
Note que o aperto

(2, 1)

aperta a mo de 2, ento

o mesmo que o aperto

Ai Aj =

para

i 6= j .

contm todos os apertos de mo.

para

1 i n 1.

Observe tambm que todos os

apertos aparecem em um dos conjuntos

elementos do conjunto

j que se

aperta a mo de 1. Analogamente,



Ai = (i, i + 1), (i, i + 2), . . . , (i, n) ,
Note que

(1, 2),

Ai .

Logo, se

Assim,

|X|

A1 An1

denota o nmero de

o nmero de apertos de mo, temos

|(A1 A2 An1 )| = |A1 | + |A2 | + + |An1 |

= (n 1) + (n 2) + + 2 + 1
=

Portanto,

(n 1)n
= a.
2

n2 n 2a = 0 deve admitir admite uma raiz inteira, maior

ou igual a 12. Deste modo, basta descobrirmos para que valores de

a < 75 a equao acima admite alguma raiz inteira n 12. Denotemos


as razes da equao por n1 e n2 e suponhamos que n1 12. Das
relaes

n1 n2 = 2a,
n1 + n2 = 1,

n2 = n1 1 11.
n2 n1 11 12 = 132, pois n2 11
conclumos que

Assim, podemos deduzir que


e

n1 12.

n1 13
a 78, sendo

Observe que o mesmo raciocnio nos leva a concluir que se

ento

n2 n1 = 2a 12 13 = 156,

o que nos daria

2.3

Equao do Segundo Grau

isto impossvel pois

a < 75.

59

Assim, a raiz positiva para tal equao

n1 = 12 como
considerarmos a = 66.

no pode ser maior ou igual que 13, restando somente


soluo.

De fato, essa soluo possvel, se

Logo, haviam 12 pessoas na festa.

2.3.3 Equaes Biquadradas


A deduo da soluo da equao do segundo grau nos permite resolver
equaes de grau mais alto, desde que elas se apresentem numa forma
peculiar, que nos permita reduzi-las a uma equao do segundo grau.
Por exemplo,

Exemplo 2.30.

Resolva a equao

x4 2x2 + 1 = 0.

(2.16)

Apesar da equao acima ser de grau quatro, podemos solucion-la


utilizando o que aprendemos at agora. O truque ser denotar por
o valor

Soluo: Denote por

y = x2 .

(y 1)2 . Logo, y = 1.
ou x = 1.

Assim,

0 = y 2 2y + 1 =
segue-se que x = 1

Neste caso, temos que

x2 = y = 1,

de onde

De modo geral consideremos a equao

ax2k + bxk + c = 0,
e faamos a mudana

y = xk .

k N,

(2.17)

Ento, a equao se transforma na

seguinte;

ay 2 + by + c = 0,

(2.18)

60

Equaes e Inequaes

a qual j sabemos resolver. Logo, se (2.18) no possui soluo ento

y =

(2.17) tambm no ter soluo e no caso em que

seja uma

raiz de (2.18) ento as solues para (2.17), correspondentes raiz

podem ser encontradas resolvendo a equao simples

xk = ,
a qual tem as seguintes possibilidades:

uma nica soluo:

nenhuma soluo: se

duas solues:

x=

<0

x=k

se

se

mpar;

par;

>0

par.

2.3.4 O Mtodo de Viti


A maneira que Franois Viti (1540-1603) descobriu para resolver a
equao do segundo grau baseia-se em relacionar a equao

ax2 + bx + c = 0

(2.19)

como uma equao do tipo

Ay 2 + B = 0,
onde

(2.20)

A e B so nmeros que dependem de a, b, c, de modo que qualquer

soluo da equao (2.20) determinar uma soluo da equao (2.19).


Note que a ltima equao possui solues

r
y1 =

r
e

y2 =

B
,
A

se

B
0.
A

2.3

Equao do Segundo Grau

61

Para fazer isso, usamos o seguinte truque: escrevendo

x = u+v

como

u e v , a equao (2.19) se escreve como

a soma de duas novas variveis

a(u + v)2 + b(u + v) + c = 0,


a qual, desenvolvendo o quadrado, equivale a

au2 + 2auv + av 2 + bu + bv + c = 0.
Agrupando convenientemente, podemos escrever a expresso acima
como uma equao na varivel

v,

isto ,

av 2 + (2au + b)v + au2 + bu + c = 0.


Assim, podemos obter uma equao do tipo da equao (2.20), escolhendo o valor de

de modo que o termo

(2au + b)v

se anule. Esco-

u = b/2a temos que


 2
b
b
b2
b2
2
av + a
b + c = 0 av 2 +

+ c = 0,
2a
2a
4a 2a

lhendo

o que equivalente a

av 2 +

b2 + 4ac
= 0.
4a

Observando que a equao assumiu a forma da equao (2.20), temos


que suas solues so

r
v1 =

b2 4ac
4a2

Lembrando que

r
e

v2 =

u = b/2a

e que

b2 4ac
,
4a2

x=u+v

se

= b2 4ac 0.

temos que as solues da

equao (2.19) so

x1 =

b
+ v1
2a

como j obtivemos anteriormente.

x2 =

b
+ v2 ,
2a

62

2.4

Equaes e Inequaes

Inequaes

Inequaes aparecem de maneira natural em vrias situaes dentro


do contexto matemtico, assim como no prprio dia a dia.

Exemplo 2.31.

Numa loja de esportes as bolas de tnis Welson en-

traram em promoo, passando a custar cada uma trs reais. Pedro


que um assduo jogador de tnis quer aproveitar ao mximo a oferta
da loja, mas ele s dispe de cem reais. Qual a maior quantidade
possvel de bolas que Pedro pode comprar?

Soluo. Se denotamos por

o nmero de bolas que Pedro compra,

ento devemos achar o maior valor possvel de

tal que

3x 100.

(2.21)

Notemos que o problema se reduz a encontrar o maior mltiplo

99 = 3 33
3 34 = 102 > 100

positivo de 3 que seja menor ou igual a 100. Observe que


o maior mltiplo de 3 menor ou igual a 100, pois

e Pedro no teria oramento para efetuar a compra. Logo, a soluo

x = 33,

ou seja, Pedro poder comprar 33 bolas.

Observemos que no exemplo anterior o que zemos foi achar o


maior valor inteiro de
nmero

real menor

x tal que 3x100 < 0; porm note que qualquer


que 100/3 satisfaz que 3x 100 < 0. Isto um

caso particular de resoluo de uma inequao, chamada inequao do


primeiro grau.

2.5

Inequao do Primeiro Grau

2.5

63

Inequao do Primeiro Grau

Uma inequao do primeiro grau uma relao de uma das formas


abaixo

ax + b < 0, ax + b > 0,
ax + b 0, ax + b 0,
onde

a, b R

(2.22)

a 6= 0.

O conjunto soluo de uma inequao do primeiro grau o conjunto

de nmeros reais que satisfazem a inequao, isto , o conjunto

de nmeros que quando substitudos na inequao tornam a desigualdade verdadeira.

Para achar tal conjunto ser de vital importncia

tomar em conta as seguintes propriedades das desigualdades entre dois


nmeros

Invarincia do sinal por adio de nmeros reais:


a

nmeros reais tais que

qualquer nmero real


tipo:

<,

ou

c.

a b,

a+c b+c

ento

para

O mesmo vale com as desigualdades do

>.

Invarincia do sinal por multiplicao de nmeros reais


positivos: sejam a e b nmeros reais tais que a b, ento
ac bc

c. Resultados
<, ou >.

para qualquer nmero real positivo

logos valem para as desigualdades do tipo:

sejam

an-

Mudana do sinal por multiplicao de nmeros reais


negativos: sejam a e b nmeros reais tais que a b, ento ac
bc

c. Resultados
<, ou >.

para qualquer nmero real negativo

valem para as desigualdades do tipo:

anlogos

64

Equaes e Inequaes

Vejamos como solucionar as inequaes estritas

ax + b < 0

ax + b > 0.

Para isto, dividimos a anlise em dois casos.

Caso 1:

a>0

Inequao ax + b < 0:
que

x + b/a < 0

neste caso, dividindo por

obtemos

b/a, em ambos os membros


que x < b/a. Portanto,

e somando

ltima inequao, temos

desta

= {x R; x < b/a},

o qual representamos no seguinte desenho:

b/a

Inequao ax + b > 0:

procedendo do mesmo modo que o

caso anterior, obtemos que o conjunto soluo vem dado por


S

= {x R; x > b/a},

representado no desenho abaixo:

b/a

Caso 2:

a<0

Inequao ax + b < 0:

sinal da inequao se inverte, obtendo assim que


logo temos que

x > b/a
S

ao
x + b/a > 0,

neste caso, quando dividimos por

e, consequentemente,

= {x R; x > b/a},

cuja representao na reta a seguinte:

2.5

Inequao do Primeiro Grau

65

b/a

Inequao ax + b > 0:

similarmente, o conjunto soluo vem

dado por
S

= {x R; x < b/a},

cuja representao a seguinte:

b/a

Observao 2.32.
ax + b 0

Notemos que se queremos resolver as inequaes

ax + b 0,

ento o conjunto soluo S em cada um

dos casos acima continua o mesmo acrescentado apenas do ponto

x=

b/a.
Vejamos agora um exemplo simples.

Exemplo 2.33.

Para resolver a inequao

8x 4 0,

primeiramente

dividimos por 8 a inequao (prevalecendo o sinal da desigualdade)

1/2 em ambos
x 4/8 + 1/2 1/2, ou seja,

e imediatamente adicionamos
para obter

os membros da mesma,

= {x R; x 1/2}.

A seguir damos alguns exemplos que podem ser resolvidos usando


inequaes lineares.

Exemplo 2.34.

Sem fazer os clculos, diga qual dos nmeros

a = 3456784 3456786 + 3456785 e b = 34567852 3456788


maior?

66

Equaes e Inequaes

x ao nmero 3456784 ento das denies


2
de a e b temos que a = x (x + 2) + (x + 1) e b = (x + 1) (x + 4).
2
2
Logo, a = x + 3x + 1 e b = x + x 3. Se supomos que a b, ento
Soluo. Se chamamos de

x2 + 3x + 1 x2 + x 3,
e somando

x2 x+3 a ambos os membros desta desigualdade obtemos


2x + 4 0.

A soluo desta inequao do primeiro grau o conjunto dos


tais que

x 2,

x = 3456784. Logo,
a ser menor ou igual a b falsa, sendo ento

mas isto falso, desde que

nossa suposio inicial de

xR

a > b.
O prximo exemplo j foi tratado antes (ver Problema 2.7), porm
apresentamos a seguir uma soluo diferente usando inequaes do
primeiro grau.

Exemplo 2.35.

Um quadrado mgico

33

um quadrado de lado 3

dividido em 9 quadradinhos de lado 1 de forma tal que os nmeros de 1


at 9 so colocados um a um em cada quadradinho com a propriedade
de que a soma dos elementos de qualquer linha, coluna ou diagonal
sempre a mesma. Provar que no quadradinho do centro de tal quadrado
mgico dever aparecer, obrigatoriamente, o nmero 5.
Soluo. Primeiramente observamos que a soma

45,

1+2+3++9 =

logo como h trs linhas e em cada uma destas guram nmeros

diferentes temos que a soma dos elementos de cada linha 15. Logo,
a soma dos elementos de cada coluna ou diagonal tambm 15.
Chamemos de

o nmero que aparece no centro do quadrado

mgico, como mostra o desenho a seguir.

2.5

Inequao do Primeiro Grau

67

Agora fazemos as seguintes observaes:

O nmero

no pode ser 9, pois nesse caso em alguma linha,

coluna ou diagonal que contm o quadrado central aparecer


o nmero 8, que somado com 9 d

17 > 15

e isto no pode

acontecer.

O nmero

x no pode ser 1,

pois nesse caso formaria uma linha,

coluna ou diagonal com o nmero 2 e um outro nmero que


chamamos de

y,

ento

impossvel.

1 + 2 + y = 15 y = 12,

o qual

Feitas as observaes anteriores, temos ento que o nmero

x forma

uma linha, coluna ou diagonal com o nmero 9 e algum outro nmero


que chamamos de

z,

logo

z = 15 (x + 9) 1 6 x 1,
de onde segue que

x 5.

x aparece numa linha, coluna ou diagonal


com o nmero 1 e algum outro nmero que chamamos de s, logo
s = 15 (x + 1) = 14 x 9, de onde temos que x 5. Finalmente,
como 5 x 5 segue-se que x = 5.
Por outro lado, o nmero

Exemplo 2.36.

Num tringulo com lados de comprimento

traamos perpendiculares desde um ponto arbitrrio


de comprimento

c,

P,

a, b

sobre o lado

at cada um dos lados restantes (ver a Figura 2.1).

Se estas perpendiculares medem

a > b,

ento

68

(a) Qual a posio onde deve ser colocado

Equaes e Inequaes

de maneira que

` = x+y

de maneira que

` = x+y

seja mnimo?
(b) Qual a posio onde deve ser colocado
seja mximo?

C
a
x P
c

b
A

Figura 2.1: No desenho, os segmentos que partem do ponto

diculares aos lados

AC

a rea do tringulo e notemos que divi-

dindo este em dois tringulos menores: um com base

so perpen-

BC

Soluo. Denotemos por

outro com base

e altura

e altura

y , temos que
ax by
+
= S,
2
2

de onde se segue que

ax = 2S by
2S by
x=
.
a
Somando

em ambos os lados da ltima igualdade, obtemos

2S by
+y
a
2S by + ay
=
a
2S a b
=
+
y,
a
a

x+y =

2.6

Inequao do Segundo Grau

69

logo

` = + y,
onde

ab
2S
e =
.
a
a
Agora notemos que 0 y hb , onde hb denota a altura relativa ao
lado de comprimento b no tringulo dado. Como positivo, por ser
a > b, temos ento que 0 y hb e, portanto, + y
+ hb , de onde
=

0 ` + hb .
` atingido quando y = 0, portanto
P deve ser colocado no vrtice A, e o valor mximo obtido quando
y = hb , portanto P deve ser colocado no vrtice B .

Resumindo, o valor mnimo de

2.6

Inequao do Segundo Grau

Agora passamos a discutir a soluo das inequaes do segundo grau,


que possuem um maior grau de diculdade quando comparadas com
as inequaes do primeiro grau. Ser de vital importncia o uso das
propriedades da funo quadrtica

ax2 + bx + c, estudadas no captulo

anterior.
Uma inequao do segundo grau uma relao de uma das formas
abaixo

ax2 + bx + c < 0, ax2 + bx + c > 0,


ax2 + bx + c 0, ax2 + bx + c 0,

(2.23)

a, b, c R e a 6= 0. Por simplicidade, chamaremos o nmero a de


2
coeciente lder da funo quadrtica ax + bx + c.
onde

70

Equaes e Inequaes

x2 3x + 2 > 0 fatoramos
2
equao x 3x + 2 = 0 so 1 e 2,

Por exemplo, para resolver a inequao


o trinmio usando que as razes da
isto ,

x2 3x + 2 = (x 1)(x 2).
O trinmio toma valores positivos quando o produto
positivo, ou seja, quando os fatores
sinal:

(x 1)

(x 2)

(x 1)(x 2) for

tenham o mesmo

Ambos positivos:

x1>0x>1
e

x 2 > 0 x > 2,
logo

x > 2.

Ambos negativos:

x1<0x<1
e

x 2 < 0 x < 2,
logo

Portanto,

x < 1.
x2 3x + 2 > 0

se, e somente se,

x<1

ou

x > 2.

A seguir explicamos como podemos resolver a inequao do segundo grau de forma geral.
Suponhamos primeiramente que queremos resolver a inequao

ax2 + bx + c > 0.

(2.24)

2.6

Inequao do Segundo Grau

71

Notemos que valem as seguintes igualdades:


b
c
ax + bx + c = a x + x +
a
a


b
b2
b2
c
2
=a x + x+ 2 2 +
a
4a
4a
a


 2

2
b
c
b
b
2

=a x + x+ 2 a
a
4a
4a2 a

2
b

=a x+
,

2a
4a
2

onde

= b2 4ac.

(2.25)

Considerando esta igualdade, dividimos em vrios

casos:

Caso 1: = b2 4ac > 0.

em conta o sinal de

(a > 0).

Nesta situao procedemos tomando

a.

Usando (2.25) notamos que basta resolver a inequao


2
b

a x+

> 0.
2a
4a
Como

a > 0,

multiplicando por

1/a

em ambos os membros da

desigualdade anterior o sinal desta no muda, obtendo-se ento


2
b

x+
2 > 0.
2a
4a
Agora usamos que

>0

para obtermos que

72

onde

b
x+
2a

2

!2
2


x+
2a
2a
!
!

b
b+
x+
= x+
2a
2a
!
!
b
b +
= x
x
2a
2a

2 =
4a

b
e
2a

= (x )(x ) > 0,
=

Agora notamos que

(x )

Equaes e Inequaes

b+
so as razes de
2a

(x )(x ) > 0

ax2 + bx + c = 0.

se os fatores

(x )

so ambos positivos ou ambos negativos. No primeiro

x >

caso (ambos positivos) temos que

x > ,

mas como

< , ento x > . No segundo caso (ambos negativos), temos


que x < e x < , logo x < , novamente por ser < .
Resumindo, a soluo da inequao vem dada pelo conjunto
S

= {x R; x <

ou

x > },

com a seguinte representao na reta:

(a < 0).

Esta situao bem similar anterior, a nica dife-

rena que ao multiplicar por

1/a o sinal se inverte tendo ento

que resolver a inequao

b
x+
2a

2

< 0,
4a2

2.6

Inequao do Segundo Grau

73

a qual equivalente a provar (seguindo os mesmos passos do


caso anterior) que

(x )(x ) < 0,

com

b
e
2a

b+
razes de
2a

ax2 + bx + c = 0.

Notemos que a desigualdade acima vlida sempre que os sinais

(x ) e (x ) forem diferentes. Por exemplo,


se x > 0 e x < 0 temos ento que x deve satisfazer a
desigualdade < x < , mas isso impossvel considerando que
neste caso > , por ser a < 0. No caso restante, se x < 0
e x > 0 temos ento que < x < , o que possvel.
dos fatores

Portanto, o conjunto soluo, neste caso, dado por


S

= {x R; < x < },

cuja representao na reta :

Caso 2: = b2 4ac = 0.

Usando novamente (2.25), devemos

resolver a inequao

b
a x+
2a
a qual vlida para qualquer

a < 0.

Caso 3: = b2 4ac < 0.

os valores de

2
> 0,

b
x 6= 2a
,

se

a > 0

Neste caso, quando

e sempre falsa, se

positivo todos

reais so soluo para (2.24), pois a desigualdade

b
ax + bx + c = a x +
2a
2

2

> 0,
4a

74

sempre satisfeita, dado que

4a
> 0.

Equaes e Inequaes

Por outro lado, se

a negativo

no temos nenhuma soluo possvel para a inequao (2.24) j que


2
b

ax + bx + c = a x +

2a
4a
2

sempre negativo, dado que

Observao 2.37.

4a
< 0.

Para a desigualdade do tipo

ax2 + bx + c < 0
so obtidos resultados similares, seguindo o mesmo processo descrito
anteriormente. Alm disso, para as inequaes

ax2 + bx + c 0

ax2 + bx + c 0

os resultados so os mesmos, acrescentados apenas dos pontos


ou

b/2a,

dependendo do caso.

Exemplo 2.38.

Provar que a soma de um nmero positivo com seu

inverso sempre maior ou igual que 2.


Soluo. Seja

x > 0,

ento devemos provar que

x+

1
2.
x

Partimos da seguinte desigualdade, que sabemos vale para qualquer

x R:

(x 1)2 0

logo

x2 2x + 1 0 x2 + 1 2x.

2.6

Se

Inequao do Segundo Grau

75

positivo, podemos dividir ambos os membros da ltima desi-

gualdade sem alterar o sinal da mesma, ou seja,

x+

1
2,
x

conforme queramos provar.

2.6.1 Mximos e Mnimos das Funes Quadrticas


f (x) = ax2 + bx + c,

A funo quadrtica

como j foi observado ante-

riormente, satisfaz a identidade


2
b

ax + bx + c = a x +
,
2a
4a
2

onde

f (x)

= b2 4ac.

O valor mnimo (mximo ) da funo quadrtica

o menor (maior) valor possvel que pode assumir

fazemos

do trinmio obtido quando


Similarmente, quando
quando

f (x)

quando

percorrer o conjunto dos reais.

Da igualdade (2.26) segue-se que, quando

x=

a < 0

x =

Sejam

a, b

a > 0

b
2a
e este vale

o valor mnimo

f ( 2a
) = 4a
.

o valor mximo do trinmio obtido

b
, valendo tambm
2a

Exemplo 2.39.
que

(2.26)

f ( 2a
) = 4a

reais positivos tais que

a + b = 1.

Provar

ab 1/4.

ab = a(1 a) = a2 + a. Denindo f (a) =


a2 + a, basta provar que f (a) 1/4 para qualquer 0 < a < 1.
Completando o quadrado a funo f (a), obtemos
Soluo. Notemos que

f (a) = (a2 a) = (a2 a + 1/4 1/4) = (a 1/2)2 + 1/4,


logo este assume seu valor mximo igual a

1/4,

quando

a = 1/2.

76

Equaes e Inequaes

Alguns problemas de mximos ou mnimos no parecem que possam ser resolvidos achando o mximo ou mnimo de funes quadrticas. Porm, estes problemas podem ser reformulados de forma tal
que isto seja possvel. Vejamos um exemplo.

Exemplo 2.40.

Na gura abaixo

tro do crculo de raio

r.

ABCD

um retngulo inscrito den-

Encontre as dimenses que nos do a maior

rea possvel do retngulo

ABCD.
D

C
r y
x

Soluo. A rea do retngulo vem dada pela frmula

A = 2x 2y = 4xy.
Usando o teorema de Pitgoras, temos que

y=

r 2 x2 ,

(2.27)

logo, substituindo esta ltima igualdade na frmula de rea anterior,


obtemos

A = 4x r2 x2 .

No muito difcil nos convencermos de que as dimenses, que nos


do a maior rea possvel para o retngulo

ABCD,

so as mesmas

que nos do o mximo para o quadrado desta rea, ou seja, basta


encontrar as dimenses que maximizam

A2 .

A vantagem que tem esta

2.7

Miscelnea

77

reformulao do problema que

A2

tem uma expresso mais simples,

dada por

A2 = 16x2 (r2 x2 ) = 16r2 x2 16x4 .


Agora fazemos a mudana

z = x2 ,

para obter


A2 = 16z 2 + 16r2 z = 16 z
de onde segue que o menor valor de
portanto quando

x=

A2

r2
2

2

+ 4r4 ,

obtido quando

z =

r2
e
2

r . Usando agora a igualdade (2.27) temos que


2

r
y=

r2

r2
r
= .
2
2

Ento, o retngulo de maior rea possvel o quadrado de lado

2r

= r 2.

2.7

Miscelnea

Nesta seo combinamos a teoria desenvolvida nos tpicos anteriores


para resolver outros tipos de equaes com um nvel de complexidade
maior.

2.7.1 Equaes Modulares


Uma equao modular aquela na qual a varivel incgnita aparece
sob o sinal de mdulo. Por exemplo, so equaes modulares
(a)

|2x 5| = 3;

(b)

|2x 3| = 1 3x;

78

(c)

Equaes e Inequaes

|3 x| |x + 1| = 4.

Para resolver equaes modulares se usam basicamente trs mtodos:


(1) eliminao do mdulo pela denio;
(2) elevao ao quadrado de ambos os membros da equao;
(3) partio em intervalos.
Ilustramos a seguir estes mtodos com os exemplos dados em (a),
(b) e (c).

Exemplo 2.41.

|2x 5| = 3.

Resolver a equao

Soluo: O mtodo (1) pode ser utilizado para resolver esta equao.
Para isto, usamos a denio de mdulo:

a
|a| =
a
de onde segue-se a propriedade: seja

se

a 0,

se

a < 0.

b um nmero no negativo, ento

|a| = b a = b
Logo,

ou

a = b.

soluo da equao se, e somente se,

satisfaz uma das

equaes de primeiro grau a seguir:

2x 5 = 3

ou

2x 5 = 3.

Da primeira equao obtemos a soluo


a soluo

x2 = 1.

x1 = 4

e da segunda obtemos

2.7

Miscelnea

79

Exemplo 2.42.

Resolver a equao

|2x 3| = 1 3x

Soluo: Resolveremos esta equao pelos mtodos (1) e (2).

Mtodo (1): Aplicando a denio de mdulo temos que resolver a


equao equivalente a resolver os sistemas mistos

(a)

2x 3 0,
2x 3 = 1 3x,

ou

(b)

2x 3 < 0,
(2x 3) = 1 3x.

O sistema (a) no tem soluo visto que a soluo da equao do

2x 3 = 1 3x 5x = 4 x = 4/5 a qual no satisfaz


a desigualdade 2x 3 0. Por outro lado, no sistema (b) a soluo
da equao (2x 3) = 1 3x tem por soluo x = 2 a qual satisfaz
a inequao 2x 3 < 0. Logo, a nica soluo da equao x = 2.

primeiro grau

Mtodo (2): Observemos que a equao equivalente ao sistema


misto

1 3x 0,

(2x 3)2 = (1 3x)2 .


Resolvendo agora a equao de segundo grau

(2x 3)2 = (1 3x)2

5x + 6x 8 = 0, temos que as possveis solues


x2 = 2, mas x1 descartada pois no satisfaz que

qual equivalente a

x1 = 4/5 e
13x1 0. Assim, a soluo do sistema misto e, portanto, da equao
modular apenas x2 = 2.
so

Exemplo 2.43. |3 x| |x + 1| = 4.
Soluo. Neste caso usaremos o mtodo de partio em intervalos que
consiste no seguinte: marcamos na reta real os valores onde
e

|x + 1|

se anulam, neste caso,

x1 = 3

x2 = 1.

|3 x|

Com isto a reta

80

numrica dividida em 3 intervalos

Equaes e Inequaes

x < 1, 1 x 3

x > 3.

Agora analisamos a equao em cada intervalo:

Intervalo x < 1: Neste caso a equao modular toma a forma

3 x (x 1) = 4 4 = 4,
Portanto, todo o intervalo

x < 1

soluo.

Intervalo 1 x 3: Neste caso a equao modular toma a forma

3 x (x + 1) = 4 2 2x = 4,
de onde segue-se que

x = 1.

x = 1.

Portanto, neste intervalo a soluo

Intervalo x > 3: Neste caso a equao modular toma a forma

3 + x (x + 1) = 4 4 = 4,
o que uma contradio. Portanto, neste intervalo no temos soluo.
Em resumo, a soluo da equao modular o intervalo

x 1.

2.7.2 Um Sistema de Equaes No lineares


O seguinte exemplo nos mostra como podemos combinar a tcnica de
resoluo de sistemas lineares e de equaes de segundo grau para
resolver sistemas mais complicados.

Exemplo 2.44.

Resolva o sistema de equaes:

x2 + 3x (x2 2)3 = 3,

x2 + 3x + (x2 2)3 = 1.

2.8

Exerccios

81

Soluo. Propomos a seguinte mudana de variveis:

u=

x2 + 3x

v = (x2 2)3 .

Assim, o sistema se converte no sistema de equaes do primeiro grau

u v = 3,
u + v = 1,
o qual tem como soluo

u=2

v = 1.

Verique! Assim,

x2 + 3x = 2 x2 + 3x = 4,
sendo

x=1

x = 4

as solues desta equao do segundo grau.

Por outro lado

(x2 2)3 = 1,
x2 2 = 1, sendo x = 1 e x = 1 as solues desta equao.
Logo, a soluo do sistema x = 1, que a nica que satisfaz u = 2
e v = 1 simultaneamente.
de onde

2.8

Exerccios

1. Observe as multiplicaes a seguir:


(a)

12.345.679 18 = 222.222.222

(b)

12.345.679 27 = 333.333.333

(c)

12.345.679 54 = 666.666.666

Para obter
to?

999.999.999 devemos multiplicar 12.345.679 por quan-

82

2. Outro

dia

ganhei

250

Equaes e Inequaes

reais, incluindo o pagamento de horas

extras. O salrio (sem horas extras) excede em

200

reais o que

recebi pelas horas extras. Qual o meu salrio sem horas extras?
3. Uma torneira

A enche sozinha um tanque em 10 h, uma torneira

enche o mesmo tanque sozinha em 15 h. Em quantas horas

as duas torneiras juntas enchero o tanque?


4. O dobro de um nmero, mais a sua tera parte, mais a sua quarta
parte somam 31. Determine o nmero.
5. Uma certa importncia deve ser dividida entre 10 pessoas em
partes iguais.

Se a partilha fosse feita somente entre 8 dessas

pessoas, cada uma destas receberia R$5.000,00 a mais. Calcule


a importncia.
6. Roberto disse a Valria: Pense um nmero, dobre esse nmero,
some 12 ao resultado, divida o novo resultado por 2.

Quanto

deu?  Valria disse 15 ao Roberto, que imediatamente revelou o nmero original que Valria havia pensado. Calcule esse
nmero.
7. Por

2/3

de um lote de peas iguais, um comerciante pagou

R$8.000,00 a mais do que pagaria pelos

2/5 do mesmo lote.

Qual

o preo do lote todo?


8. Determine um nmero real

a para que as expresses

3a+6
2a+10
e
8
6

sejam iguais.
9. Se voc multiplicar um nmero real

por ele mesmo e do resul-

tado subtrair 14, voc vai obter o quntuplo do nmero


esse nmero?

x.

Qual

2.8

Exerccios

83

10. Eu tenho o dobro da idade que tu tinhas quando eu tinha a


tua idade. Quando tu tiveres a minha idade, a soma das nossas
idades ser de 45 anos. Quais so as nossas idades?
11. Um homem gastou tudo o que tinha no bolso em trs lojas. Em
cada uma gastou 1 real a mais do que a metade do que tinha ao
entrar. Quanto o homem tinha ao entrar na primeira loja?
12. Com os algarismos
rismos

xy

Quanto

x, y

yx, cuja soma


valem x, y e z ?
e

formam-se os nmeros de dois alga o nmero de trs algarismos

zxz .

13. Quantos so os nmeros inteiros de 2 algarismos que so iguais


ao dobro do produto de seus algarismos?
14. Obter dois nmeros consecutivos inteiros cuja soma seja igual a
57.
15. Qual o nmero que, adicionado ao triplo do seu quadrado, vale
14?
16. O produto de um nmero positivo pela sua tera parte igual a
12. Qual esse nmero?
17. Determine dois nmeros consecutivos mpares cujo produto seja
195.
18. A diferena entre as idades de dois irmos 3 anos e o produto
de suas idades 270. Qual a idade de cada um?
19. Calcule as dimenses de um retngulo de 16 cm de permetro e

15 cm2

de rea.

84

Equaes e Inequaes

20. A diferena de um nmero e o seu inverso

8
.
3

Qual esse

nmero?

21. A soma de dois nmeros 12 e a soma de seus quadrados 74.


Determine os dois nmeros.

22. Um pai tinha 30 anos quando seu lho nasceu. Se multiplicarmos


as idades que possuem hoje, obtm-se um produto que igual
a trs vezes o quadrado da idade do lho.

Quais so as suas

idades?

23. Os elefantes de um zoolgico esto de dieta juntos. Num perodo


de 10 dias devem comer uma quantidade de cenouras igual ao
quadrado da quantidade que um coelho come em 30 dias. Em
um dia os elefantes e o coelho comem juntos 1.444 kg de cenoura.
Quantos kilos de cenoura os elefantes comem em 1 dia?

24. Sejam

Calcule as seguintes expresses, em

(a)
(b)
(c)

ax2 + bx + c, com a 6= 0.
funo de a, b e c:

as razes do polinmio

1 + 2
;
2

1 + 2 ;

4 + 4 .
1
2

25. O nmero

condies, determine o valor do

Nessas

p(x) = 2x4 +bx3 +cx2 +dx+e que satisfaz


p(x) = p(1 x).

26. Encontre o polinmio


a equao

x2 7x 2c = 0.
coeciente c.

a raiz da equao

2.8

Exerccios

85

27. (OBM) Dois meninos jogam o seguinte jogo. O primeiro escolhe dois nmeros inteiros diferentes de zero e o segundo monta
uma equao do segundo grau usando como coecientes os dois
nmeros escolhidos pelo primeiro jogador e 1.998, na ordem

a e b o se1.998x2 + ax + b = 0 ou

que quiser (ou seja, se o primeiro jogador escolhe


gundo jogador pode montar a equao

ax2 + 1.998x + b = 0 etc.)

O primeiro jogador considerado ven-

cedor se a equao tiver duas razes racionais diferentes. Mostre


que o primeiro jogador pode ganhar sempre.
28. (OBM) Mostre que a equao
solues onde

x, y, z

x2 + y 2 + z 2 = 3xyz

tem innitas

so nmeros inteiros.

29. (Gazeta Matemtica, Romnia) Considere a equao

a2 x2 (b2 2ac)x + c2 = 0,
a, b e c so nmeros inteiros positivos. Se n N
p(n) = 0, mostre que n um quadrado perfeito.
onde

30. (Gazeta Matemtica, Romnia) Sejam


equao

a, b Z.

tal que

Sabendo que a

(ax b)2 + (bx a)2 = x,


tem uma raiz inteira, encontre os valores de suas razes.
31. (Gazeta Matemtica, Romnia) Resolva a equao:

Obs.:

[x]


2x2
= x.
x2 + 1

o menor inteiro maior ou igual a

x.

86

Equaes e Inequaes

32. Demonstrar que:


(a)

n4 + 4

no primo se

n > 1;

(b) generalize, mostrando que

n4 + 4n

no primo, para todo

n > 1.
33. Para fazer 12 bolinhos, preciso exatamente de 100 g de acar,
50 g de manteiga, meio litro de leite e 400 g de farinha. Qual a
maior quantidade desses bolinhos que serei capaz de fazer com
500 g de acar, 300 g de manteiga, 4 litros de leite e 5 kg de
farinha ?
34. Dadas as fraes

966666555557
966666555558

966666555558
,
966666555559

qual maior?
35. Achar o maior valor inteiro positivo de

tal que

n200 < 5300 .


36. Achar o menor valor inteiro positivo de
1

tal que

10 11 10 11 10 11 10 11 > 100000.
37. Nove cpias de certas notas custam menos de R$ 10,00 e dez
cpias das mesmas notas (com o mesmo preo) custam mais de
R$ 11,00. Quanto custa uma cpia das notas?
38. Se enumeram de 1 at

n as pginas de um livro.

Ao somar estes

nmeros, por engano um deles somado duas vezes, obtendo-se


o resultado incorreto: 1.986. Qual o nmero da pgina que foi
somado duas vezes?

2.8

Exerccios

87

39. Determine os valores de

ax ax + 12

para os quais a funo quadrtica

sempre positiva.

40. Ache os valores de

para os quais cada uma das seguintes ex-

presses positiva:

(a)

x
2
x +9

(b)

x3
x+1

(c)

x2 1
x2 3x

41. Resolver a equao:

[x]{x} + x = 2{x} + 10,


[x] denota a parte inteira de x. Por exemplo, [2, 46] = 2
[5, 83] = 5. O nmero {x} chamado parte fracionria de x e
denido por {x} = x [x].

onde

42. Mostre que entre os retngulos com um mesmo permetro, o de


maior rea um quadrado.
43. Entre todos os tringulos issceles com permetro

xado, ache

as dimenses dos lados daquele que possui a maior rea.


44. (OBM Jnior 1993)
dada uma equao do segundo grau

x2 + ax + b = 0,

com

a1 e b1 . Consideramos a equao do segundo grau


x + a1 x + b1 = 0. Se a equao x2 + a1 x + b1 = 0 tem razes
2
inteiras a2 e b2 , consideramos a equao x + a2 x + b2 = 0. Se a
2
equao x +a2 x+b2 = 0 tem razes inteiras a3 e b3 , consideramos
2
a equao x +a3 x+b3 = 0. E assim por diante. Se encontramos
uma equao com < 0 ou com razes que no sejam inteiros,
razes inteiras

encerramos o processo.

88

Equaes e Inequaes

Por exemplo, se comeamos com a equao

x2 = 0

podemos

continuar o processo indenidamente. Pede-se:

(a) Determine uma outra equao que, como

x2 = 0,

nos per-

mita continuar o processo indenidamente;


(b) Determine todas as equaes do segundo grau completas a
partir das quais possamos continuar o processo indenidamente.

Referncias Bibliogrcas
[1] AIGNER, M. e ZIEGLER, G. (2002). As Provas esto

no Livro. Edgard Blcher.


[2] GARCIA, A. e LEQUAIN, I. (2003). Elementos de l-

gebra. Projeto Euclides, IMPA.


[3] LIMA, E. L.; CARVALHO, P. C. P.; WAGNER, E. e
MORGADO, A.C. (2004). A Matemtica do Ensino M-

dio. Volume 1. Sociedade Brasileira de Matemtica.


[4] LIMA, E.L.; CARVALHO, P. C. P.; WAGNER, E. e
MORGADO, A.C. (2004). A Matemtica do Ensino M-

dio. Volume 2. Sociedade Brasileira de Matemtica.


[5] LIMA,E.L.;

CARVALHO,P.

C.

P.;

WAGNER,E.

MORGADO,A.C. (2004). A Matemtica do Ensino M-

dio. Volume 3. Sociedade Brasileira de Matemtica.


[6] LIMA, E.L.; CARVALHO, P. C. P.; WAGNER,E. e
MORGADO, A.C. (2001). Temas e Problemas. Sociedade Brasileira de Matemtica.
[7] LIMA, E.L. (2001). lgebra Linear. Sociedade Brasileira
de Matemtica.

285

286

REFERNCIAS BIBLIOGRFICAS

[8] MORAIS FILHO, D. C. (2007). Um Convite Matem-

tica. EDUFCG.
[9] MORGADO, A.;

CARVALHO, J.;

CARVALHO, P.;

FERNANDEZ, P. (1991). Anlise Combinatria e Pro-

babilidade . Sociedade Brasileira de Matemtica.


[10] RIBENBOIM, P. (2001). Nmeros Primos: Mistrios e

Recordes. Sociedade Brasileira de Matemtica.


[11] SANTOS, J. P. O. (1993) Introduo Teoria dos N-

meros. IMPA.
[12] SANTOS, J. P. O.; MELLO, M. P. e MURARI, I. T.
C. (2006). Introduo Anlise Combinatria. Editora
Unicamp.
[13] SOARES, M. G. (2005). Clculo em uma Varivel Com-

plexa. Sociedade Brasileira de Matemtica.

Mestrado Profissional

em Matemtica em Rede Nacional

Iniciao Matemtica

Autores:
Krerley Oliveira

Adn J. Corcho

Unidade II:
Captulos III e IV

3
Divisibilidade
Os nmeros governam o mundo.
Plato

A teoria dos nmeros o ramo da Matemtica que estuda os mistrios dos nmeros e teve sua origem na antiga Grcia. Os belssimos
problemas ligados a esta rea constituem, at hoje, uma das principais fontes inspiradoras dos amantes da Matemtica. Alm disso, essa
rea possui vrias aplicaes teis a humanidade, como por exemplo,
o processo de criptograa usado em transaes pela Internet.
Alguns problemas em teoria dos nmeros demoram sculos para

ltimo teorema de Fermat , que


arma que no existe nenhum conjunto de inteiros positivos x, y, z e n
com n maior que 2 que satisfaa xn + y n = z n . Esse problema foi obserem resolvidos, como por exemplo o

jeto de fervorosas pesquisas durante mais de 300 anos e foi nalmente


demonstrado em 1995 pelo matemtico Andrew Wiles.
Ainda hoje persistem muitas questes naturais e simples sem resposta.

Por exemplo, ningum sabe mostrar (apesar de todo mundo

89

90

Divisibilidade

todo natural par soma de dois primos. Essa a famosa conjectura de Goldbach . Essa simplicidade de se
acreditar que verdade!)

que

anunciar problemas e a extrema diculdade em resolv-los faz desta


rea um grande atrativo para os matemticos do mundo todo.
Este captulo ser dedicado ao estudo de algumas propriedades
bsicas relativas aos nmeros inteiros.

3.1

Conceitos Fundamentais e Diviso Euclidiana

Denotamos por

o conjunto dos nmeros inteiros formado pelo con-

junto dos nmeros naturais


nmeros negativos. Ou seja,

N = {1, 2, 3, . . .} munido do zero e


Z = {. . . , 3, 2, 1, 0, 1, 2, 3, . . .}.

dos

Comeamos observando que a soma, diferena e produto de nmeros inteiros tambm sero nmeros inteiros. Entretanto, o quociente
de dois inteiros pode ser um inteiro ou no.
Uma das propriedades fundamentais dos nmeros naturais que utilizaremos ao longo do texto o conhecido

princpio da boa ordenao ,

que arma o seguinte:

Princpio da Boa Ordenao: todo subconjunto no vazio A N


possui um elemento menor que todos os outros elementos deste, ou
seja, existe a A tal que a n para todo n A.
Por exemplo, se

mento de

o conjunto dos nmeros pares, o menor ele-

A o nmero 2.

Por outro lado, observamos que o conjunto

dos nmeros inteiros no goza da boa ordenao.


Apesar do princpio da boa ordenao parecer inocente e natural,
muitos resultados importantes a respeito dos nmeros naturais decor-

3.1

Conceitos Fundamentais e Diviso Euclidiana

91

rem do mesmo, como veremos ao longo de todo este captulo.

Denio 3.1. Sejam a e b inteiros. Dizemos que a divide b se existe


um inteiro q tal que b = aq . Tambm usaremos as frases a
de b ou b mltiplo de a para signicar esta situao.
a | b

Usaremos a notao

para representar todas as frases equi-

valentes ditas anteriormente. Se


veremos

divisor

no for divisor de

b,

ento escre-

a - b.

Exemplo 3.2. 7 | 21 pois 21 = 7 3. Por outro lado 3 - 8 pois

considerando o conjunto M = {3m, m N} = {3, 6, 9, 12, . . .} dos


mltiplos positivos de 3 vemos que 8 no pertence ao mesmo.
A seguinte proposio um bom exerccio para entender os conceitos enunciados acima.

Proposio 3.3. Sejam a, b e c nmeros inteiros. Ento,


(a) se a | b e b | c ento a | c;
(b) se a | b e a | c ento a | (b + c) e a | (b c);
(c) se a e b so positivos e a | b ento 0 < a b;
(d) se a | b e b | a ento a = b ou a = b.
Demonstrao.

Se

a|b

b|c

ento existem inteiros

q1

q2

tais que

b = aq1

(3.1)

c = bq2 .

(3.2)

92

Divisibilidade

Substituindo (3.1) em (3.2) temos que

c = aq1 q2 = aq,

onde

q = q1 q2 Z,

(3.3)

provando isto a armao feita em (a).


Agora provaremos (b).

Com efeito, se

igualdades

a | b

a | c

valem as

b = aq1 ,

q1 Z

(3.4)

c = aq2 ,

q2 Z.

(3.5)

Operando com os ambos lados das igualdades (3.4) e (3.5) temos que

b + c = a(q1 + q2 )
| {z }

rZ

b c = a(q1 q2 ),
| {z }
sZ

obtendo assim o resultado desejado.


Continuamos agora com a prova de (c). De fato, se
ambos positivos, ento

b = aq

com

q 1.
Logo, multiplicando por

a | b,

sendo

(3.6)

a ambos lados de (3.6) temos (como a posi-

tivo) que

b = aq a > 0,
como espervamos.
Finalmente, provaremos (d). Com este propsito observamos que
se

a|b

b|a

(c) temos

|a| = |b|

|a| divide |b| e |b| divide |a|. Portanto, pelo item


que |a| |b| e |b| |a|, ou seja, |a| |b| |a|. Logo,
consequentemente a = b ou a = b.
ento

Conceitos Fundamentais e Diviso Euclidiana

3.1

93

Exemplo 3.4. Prove que o nmero N = 545362 7 no divisvel por

5.

Soluo.

Vamos mostrar isso utilizando o mtodo do absurdo.

este nmero fosse divisvel por 5, ento

45362

5q ,

ou seja,

seria divisvel por

45362

5
5, o

7 = 5q .

Logo,

Se

7 =

que um absurdo.

O prximo passo de nossa discusso ver o que acontece quando


um nmero no divisvel por outro. Por exemplo, analisemos se 31
divisvel por 7 e para isto listaremos a diferena entre 31 e os mltiplos
positivos de 7, isto :

r1 = 31 7 1 = 24,
r2 = 31 7 2 = 17,
r3 = 31 7 3 = 10,
r4 = 31 7 4 = 3,

r5 = 31 7 5 = 4,

r6 = 31 7 6 = 11,
.
.
.

Claramente 31 no divisvel por 7, pois caso contrrio teramos


que alguma das diferenas acima seria igual a zero, o que impossvel
pois as diferenas
e com

q 5

rq = 31 7q

com

1 q 4

so todas positivas

so todas negativas. Entretanto, notamos que entre as

diferenas positivas a nica que menor que 7 corresponde ao caso

q = 4.

O resultado seguinte nos diz o que acontece no caso geral da

diviso de um inteiro

por um inteiro positivo

a.

94

Teorema 3.5

(Diviso Euclidiana)

Divisibilidade

. Dados dois inteiros a e b, sendo

a positivo, existem nicos inteiros q e r tais que


0 r < a.

b = aq + r,

Se a - b, ento r satisfaz a desigualdade estrita 0 < r < a.


Demonstrao.
a,

Por simplicidade, suporemos que


e

r = b.

b<
r = 0.

positivo. Se

b = a, ento tomamos q = 1 e
Assim, assumiremos tambm que b > a > 0. Consideremos o conjunto
basta tomar

q=0

Se

R = {b aq Z; b aq 0} N {0}
Notemos que o conjunto

no vazio, pois

(3.7)

b a R,

j que

b a > 0. Deste modo, pelo princpio da boa ordenao temos que


R admite um menor elemento, que denotaremos por r. Claramente
r = b aq 0, para algum q 0. Alm disso, r < a pois caso
contrrio

r = b aq a b a(q + 1) 0.

(3.8)

a > 0 b a(q + 1) < b aq.

(3.9)

Por outro lado,

Das desigualdades (3.8) e (3.9) segue que

0 b a(q + 1) < b aq,


contradizendo o fato de que
de

r = baq o menor elemento no negativo

R.
Agora provaremos que de fato

q,

escolhidos desta forma, so

nicos. Com efeito, suponhamos que existem outros inteiros


tais que

b = aq1 + r1 ,

0 r1 < a.

r1

q1

3.1

Conceitos Fundamentais e Diviso Euclidiana

aq + r = aq1 + r1 .

Ento resulta que

95

Logo,

(r r1 ) = (q1 q)a;

(3.10)

rr1 mltiplo de a. Mas, em virtude de a < rr1 < a,


o nico valor que r r1 pode tomar, sendo este mltiplo de a,
r r1 = 0. Portanto, r = r1 , de onde se deduz diretamente de (3.10)
que q = q1 .
sendo assim,

Os nmeros

respectivamente, de

no enunciado do teorema acima so chamados,

quociente

resto

da diviso de

por

a.

Um resultado imediato da diviso euclidiana o seguinte.

Corolrio 3.6. Dados dois nmeros naturais a e b com 1 < a b,

existe um nmero natural n tal que

na b < (n + 1)a.

Demonstrao.
0r<a

Pela diviso euclidiana, existem nicos

tais que

b = aq + r.

Assim

q, r N

com

aq b = aq + r < aq + a = a(q + 1).


Basta agora tomar

q=n

para obtermos o resultado.

Os exemplos a seguir apresentam a utilidade do Teorema 3.5.

Exemplo 3.7. Se a um natural com a 3, ento a2 deixa resto 1


na diviso por a 1. Consequentemente, a 1 divide a2 1.

Soluo.

a2 1 = (a 1)(a + 1) temos que


1 < a 1, de onde segue o resultado.

Usando a identidade

a2 = (a 1)(a + 1) + 1

com

96

Divisibilidade

O prximo exemplo, como veremos, motiva a procura de caminhos ecientes para encontrar o resto que deixa um nmero quando
dividido por outro.

Exemplo 3.8. Um turista brasileiro chega a Cuba e troca parte de


seu dinheiro na casa de cmbio, recebendo 175 notas de 50 pesos e
213 notas de 20 pesos. Ele decide trocar este dinheiro pela maior
quantidade possvel das famosas moedas de 3 pesos cubanos, porque
elas tm gravada a imagem do guerrilheiro Che Guevara. Quanto
sobrou do dinheiro depois de fazer a troca pelas moedas?
Soluo.
nmero

Para resolver este problema basta achar o resto que deixa o

n = 175 50 + 213 20

quando dividido por 3. Entretanto,

queremos destacar que no preciso fazer os produtos e a soma envolvidos no nmero


que aparece em
assim um novo

n.

Em lugar de fazer isto substitumos cada nmero

n pelo resto que este deixa na diviso por 3, formando


nmero n1 , ou seja,
n1 = 1 2 + 0 2 = 2.

Agora procuramos o resto que

n1

deixa na diviso por 3, que obvi-

amente 2. A surpresa que este resto o mesmo que deixa

na

diviso por 3. Logo, sobraram 2 pesos depois de fazer a troca.

A soluo do exemplo anterior uma aplicao particular do seguinte lema que de muita utilidade na resoluo de problemas.

Lema 3.9

(Lema dos Restos)

. A soma e o produto de quaisquer dois

nmeros naturais deixa o mesmo resto que a soma e o produto dos


seus restos, na diviso por um inteiro positivo a.

Conceitos Fundamentais e Diviso Euclidiana

3.1

Demonstrao.

Sejam

ambos os nmeros por

n1 , n2 Z.
a temos que

n1 = aq1 + r1
com

0 r1 , r2 < a.

97

Fazendo a diviso com resto de

n2 = aq2 + r2 ,

Ento,

n1 n2 = (aq1 + r1 )(aq2 + r2 )
= a2 q1 q2 + aq1 r2 + aq2 r1 + r1 r2
= a(aq1 q2 + q1 r2 + q2 r1 ) + r1 r2

(3.11)

= aq + r1 r2 ,
onde

q = aq1 q2 +q1 r2 +q2 r1 .

Agora dividimos

r1 r2 = ap + r,

p Z,

r1 r2 por a para obtermos

0 r < a.

(3.12)

Das igualdades (3.11) e (3.12) segue que

n1 n2 = aq + ap + r = a(p + q) + r,

0 r < a.

(3.13)

Portanto, de (3.12) e (3.13) conclumos que os restos que deixam


e

r1 r2

na diviso por

n1 n2

so iguais, cando provado o resultado para o

produto. A prova para a soma anloga.

Observao 3.10. A vantagem do lema que em certos problemas


que envolvem nmeros muito grandes podemos substituir estes por nmeros muito menores e mais confortveis para trabalhar.

Vejamos como aplicar o lema dos restos nos seguintes exemplos a


seguir.

Exemplo 3.11. Prove que o produto de dois nmeros naturais consecutivos sempre divisvel por 2.

98

Soluo.
2.

Divisibilidade

Se

n N temos

que provar que

Quando fazemos a diviso de

r=0

para o resto:

[r = 0]

ou

r = 1.

an = n(n + 1) divisvel

por

por 2 temos duas possibilidades

Analisemos os dois casos por separado.

Neste caso o resto que deixa

an

na diviso por 2 o

mesmo que o resto que deixa 0(0+1)=0, logo

an

divisvel por

2.

[r = 1]

Neste caso podemos substituir

an

por 1(1+1)=2 e o

resto que este ltimo deixa quando dividido por 2 0, logo

an

tambm divisvel por 2.

Mostraremos agora como utilizar o exemplo anterior pra resolver


um dos problemas da

1a

Olimpada Brasileira de Matemtica.

Exemplo 3.12. Prove que se n mpar, ento n2 1 mltiplo de

8.

Soluo.
k Z.

Como

mpar, podemos escrever

n = 2m + 1,

para algum

Logo

n2 1 = (2m + 1)2 1 = 4m2 + 4m + 1 1 = 4m2 + 4m.


Assim,

n2 1 = 4m(m + 1).
m(m + 1)
q Z, de aonde

Observe que de acordo com o exemplo 3.11,


2. Portanto,

m(m + 1) = 2q

para algum

n2 1 = 4m(m + 1) = 4 2q = 8q,
como queramos demonstrar.

mltiplo de

Bases Numricas

3.2

99

Exemplo 3.13. Prove que em qualquer tringulo retngulo com lados

inteiros, pelo menos um deles mltiplo de 3.


Soluo.

Comecemos analisando quais so os restos possveis para a

diviso por 3 de um nmero que quadrado. De acordo com o lema


dos restos temos a seguinte tabela para os restos de

n e n2 ,

na diviso

por 3:

n n2
0

Resumindo, se um nmero no mltiplo de 3 ento o resto da diviso


de seu quadrado por 3 deve ser igual a 1.
Agora denotemos por

os catetos e por

a hipotenusa. Supo-

a2 e b2 deixam resto
12 + 12 = 2 na diviso

nhamos que nenhum deles divisvel por 3. Ento


1 na diviso por 3. Logo,

a2 + b 2

deixa resto

por 3; mas isto uma contradio pois, pelo Teorema de Pitgoras,

a2 + b 2 = c 2

3.2

c2

deixa resto 1 quando dividido por 3.

Bases Numricas

Comeamos esta seo com uma brincadeira interessante.


Joo, ao sair da aula de matemtica do professor Peitgoras, encontrou Pedro e lhe props a seguinte brincadeira:
 Pense numa pea de domin, Pedro. Vou adivinhar que pea
essa usando uma frmula mgica.
 Ok, Joo. Pode comear, j pensei.

100

Divisibilidade

Figura 3.1: Pea de Domin

- Escolha um dos nmeros na pea e multiplique por 5.

Depois

disso some trs a esse resultado. Multiplique agora o nmero que voc
obteve por dois. Some isto com o outro nmero da pea. Qual foi o
resultado?
 Foi 40.
 Ento a pea que voc escolheu foi a 3 com 4!
 Como voc acertou? Me ensina!
Claro que de mgico Joo no tinha nada e decidiu contar seu
segredo a Pedro.
O jogo funciona assim: cada parte da pea de domin pode ser
considerada como um dos dgitos de um nmero de 2 algarismos, o qual
denotamos por

n = xy = 10x + y

(veja a Figura 3.1). Acompanhando

os passos de Joo, temos que:

(5x + 3)2 + y = 40 10x + y = 34,


que claramente, tem por solues:

x=3

y = 4,

(3.14)

usando a represen-

tao de 34 na base decimal.


No sistema de numerao decimal, tambm conhecido como sistema numrico na base

10,

todo nmero pode ser representado como

uma sequncia de 10 smbolos, constitudos pelo


rismos

1, 2, 3, . . . , 9.

Por exemplo,

345

(zero) e os alga-

escreve-se na base decimal da

Bases Numricas

3.2

101

seguinte forma

345 = 300 + 40 + 5 = 3 102 + 4 10 + 5,


assim como

2768

se escreve da forma

2768 = 2000 + 700 + 60 + 8 = 2 103 + 7 102 + 6 10 + 8.


a = an an1 . . . a1 a0 o nmero inteiro
algarismos an , an1 , . . . , a1 e a0 , nessa ordem,

De modo geral, se denotamos por


positivo formado pelos
ento

se escreve na base decimal da forma

a = an 10n + an1 10n1 + . . . + a1 10 + a0

(3.15)

Antes de provar alguns dos critrios de divisibilidade mais populares do sistema de numerao decimal, provamos uma identidade
muito til.

Lema 3.14. Sejam a, b, n N. Temos que


an bn = (a b)(an1 + an2 b + + abn2 + bn1 ).

Consequentemente, se 0 < b < a, ento a b divide an bn .


Demonstrao.
1.

Primeiro provaremos que a propriedade vale para

Com efeito, considerando a soma geomtrica

s = 1 + a + a2 + + an1
e multiplicando

por

temos que

as = (a + a2 + + an1 ) + an = s 1 + an .

b=

102

Assim,

(a 1)s = as s = an 1,

Divisibilidade

de onde se segue que

an 1 = (a 1)(an1 + an2 + + a + 1).


Da temos a validade para
Para

bN

(3.16)

b = 1.

qualquer, observe que

esta expresso e (3.16) tem-se



an bn = bn ( ab )n 1 .

Usando



an bn = bn ( ab 1) ( ab )n1 + ( ab )n2 + + ( ab ) + 1


= (a b)bn1 ( ab )n1 + ( ab )n2 + + ( ab ) + 1

(3.17)

= (a b)(an1 + an2 b + + abn2 + bn1 ),

obtendo-se a igualdade clamada.

Proposio 3.15

(Critrios de Divisibilidade)

um inteiro positivo, ento

. Seja a = an . . . a1 a0

(a) a divisvel por 10 se, e somente se, a0 for igual a 0;


(b) a divisvel por 3 ou por 9 se, e somente se, a soma dos seus
dgitos divisvel por 3 ou por 9, respectivamente;
(c) a divisvel por 5 se, e somente se, a0 for igual a 0 ou 5.
Demonstrao.

Utilizando a representao decimal (3.15) temos que

a = 10(an 10n1 + an1 10n2 + + a1 ) + a0 .


Ento, pela Proposio 3.3-(b) tem-se que

10 | a0 ,

prondose-se assim o critrio (a).

10 | a

se, e somente se,

Para provar (b) observemos que

a = an 10n + an1 10n1 + + 10a1 + a0

= an (10n 1) + an1 (10n1 1) + + (10 1)a1

+ an + an1 + + a1 + a0 .

(3.18)

3.2

Bases Numricas

103

Pelo Lema 3.14 temos que


segue-se

10j 1 = 9qj

para todo

1 j n,

da

a = 9(an qn + an1 qn1 + + a1 ) + an + an1 + + a1 + a0 .


Ento, aplicando novamente o item (b) da Proposio 3.3 temos que

9|a

se, e somente se,

9 | (an + an1 + + a1 + a0 ).

A prova para o caso da divisibilidade por 3 segue de maneira idntica,


logo ca provado o item (b).
A prova do item (c) segue de maneira muito semelhante e deixamos
a mesma a cargo do leitor.

Exemplo 3.16. Prove sem fazer muitas contas que o nmero


N = 13424136 + 1234567890

divisvel por 3.
Soluo.

Note que no precisamos fazer a soma dos nmeros ante-

riores. Para mostrar isso, basta aplicar o item (b) da Proposio 3.3 e
o item (b) da Proposio 3.15, observando que cada um dos nmeros
acima divisvel por

3,

pois a soma de seus dgitos um mltiplo de

3.
Finalizamos esta seo com uma aplicao da diviso euclidiana
que nos mostra que, analogamente representao decimal, qualquer
nmero admite uma representao nica em qualquer outra base numrica.

104

Teorema 3.17

(Bases Numricas)

Divisibilidade

. Dados a, b N, com b > 1, exis-

tem nicos nmeros naturais r0 , r1 , . . . , rn tais que 0 ri b 1,


0 i n, e satisfazendo
a = rn bn + rn1 bn1 + + r1 b + r0 .

A representao acima dita representao de a na base b e usaremos


a notao
a = (rn cn1 . . . r1 r0 )b ,

para fazer referncia a esta.


Demonstrao.

Apliquemos sucessivamente a diviso euclidiana como

segue:

a = bq0 + r0 ,

r0 < b,

q0 = bq1 + r1 ,

r1 < b,

q1 = bq2 + r2 ,

r2 < b,

.
.
.

.
.
.

.
.
.

qj1 = bqj + rj ,

.
.
.

rj < b,

a > q0 > q1 > q2 > > qj1 , para algum


j = n deveremos ter que qn1 < b. Logo, qj = 0 para todo j n,
assim como rj = 0 para todo j n + 1. Das igualdades acima, para
e assim por diante. Como

3.2

Bases Numricas

1 j n,

105

tem-se

a = bq0 + r0 ,
bq0 = b2 q1 + br1 ,
b2 q1 = b3 q2 + b2 r2 ,
.
.
.

.
.
.

(3.19)

.
.
.

bn1 qn2 = bn qn + bn1 rn1


bn qn1 = bn+1 0 + bn rn .
Efetuando a soma de todas as igualdades em (3.19) obtemos

a = rn bn + rn1 bn1 + + r1 b + r0 .
A unicidade dos nmeros

ri

vem da unicidade dos restos na diviso

euclidiana.

Observao 3.18. O sistema de numerao na base 2 tambm conhecido como sistema binrio e o sistema habitualmente utilizado no
funcionamento dos computadores.

Exemplo 3.19. Se deseja pesar qualquer nmero inteiro de gramas de

ouro, entre 1g e 100g , numa balana de dois pratos, onde os pesos s


podem ser usados no prato esquerdo da balana. Mostre que a escolha
adequada de 7 pesos diferentes suciente para realizar esta tarefa.
Demonstrao.

Usando o sistema de numerao em base

qualquer nmero
nica como

tal que

1 a 100

temos que

pode ser expressado de forma

a = r6 26 + r5 25 + r4 24 + r3 23 + r2 22 + r1 2 + r0 1,

106

com

ri {0, 1}, 0 i 1.

Observe que

2n 128,

Divisibilidade

com

n 7,

logo

estas potncias no so consideradas. notemos tambm que o fato de


cada

ri

ser

ou

nos diz que no precisamos repetir nenhum dos

pesos na realizao de qualquer pesada. Logo, os pesos

1, 22 , 23 , 24 , 25 , 26
so sucientes para realizar as pesadas de gramas de ouro entre

1g

100g .

3.3

Mximo Divisor Comum e Mnimo Mltiplo Comum

Nesta seo estudaremos dois conceitos fundamentais, que aparecem


naturalmente em vrios problemas de divisibilidade, assim como a
relao existente entre eles.

3.3.1

Mximo Divisor Comum

O primeiro destes conceitos est relacionado com os inteiros positivos


que dividem simultaneamente a dois inteiros prexados e denominado

mximo divisor comum .

Daqui por diante s consideraremos os divisores positivos dos nmeros.

Denio 3.20 (Mximo Divisor Comum) . Sejam a e b inteiros diferentes de zero. O mximo divisor comum, resumidamente mdc, entre
a e b o nmero d que satisfaz as seguintes condies:
(a) d um divisor comum de a e b, isto , d | a e d | b;

3.3

Mximo Divisor Comum e Mnimo Mltiplo Comum

107

(b) d o maior inteiro positivo com a propriedade (a).


Neste caso, denotamos o mdc entre a e b por d = mdc(a, b) ou por
d = (a, b). Se (a, b) = 1, ento dizemos que a e b so primos entre si.

Exemplo 3.21. Observando que 12 = 6 2, 18 = 6 3 temos que

mdc.(12, 18) = 6. Por outro lado, mdc. (4, 15) = 1, logo os nmeros 4
e 15 so primos entre si.
Vejamos agora algumas das propriedades mais importantes dos
divisores comuns de dois inteiros.

Proposio 3.22. Sejam a e b dois inteiros. Ento valem as seguintes


armaes.

(a) Se a mltiplo de b, ento (a, b) = b.


(b) Se a = bq + c, c 6= 0, ento o conjunto dos divisores comuns dos
nmeros a e b coincide com o conjunto dos divisores comuns dos
nmeros b e c. Particularmente, (a, b) = (b, c).
Demonstrao.

Comeamos com a prova de (a).

Com efeito, todo

a e b um divisor de b. Reciprocamente,
usando que a mltiplo de b, todo divisor de b tambm um divisor
de a, ou seja, um divisor comum dos nmeros a e b. Portanto, o
conjunto dos divisores comuns dos nmeros a e b igual ao conjunto
dos divisores de b. Como o maior divisor de b ele mesmo, resulta que
(a, b) = b.
divisor comum dos nmeros

Vejamos (b).

Usando o item (b) da Proposio 3.3 temos que

todo divisor comum de

tambm divide

e, consequentemente,

b e c. Pela mesma razo todo divisor comum de b e c


tambm divide a e, consequentemente, um divisor de a e b. Portanto

um divisor de

108
os divisores comuns de
de

Divisibilidade

c.

so os mesmos que os divisores comuns

Particularmente, tambm coincidem os maiores divisores

comuns, ou seja,

(a, b) = (b, c).

O teorema a seguir uma das ferramentas bsicas na resoluo


de problemas que envolvem o mdc entre dois nmeros. O resultado
foi provado pela primeira vez por Claude-Gaspard Bachet de Mziriac
(1581-1638) e mais tarde generalizado para polinmios por tienne
Bzout (1730-1783).

Frequentemente, na literatura se enuncia este

resultado como teorema (ou identidade) de Bzout, esquecendo-se o


nome de Bachet.

Teorema 3.23

(Teorema de Bachet-Bzout)

. Se d o mdc de a e b,

Demonstrao.

Considere a combinao linear

ento existem nmeros inteiros x0 e y0 tais que d = (a, b) = ax0 + by0 .


ax + by ,

onde

percorrem todos os inteiros. Este conjunto de inteiros, denotado por

Ca,b = {ax + by; x, y Z},


inclui valores positivos e negativos. Alm disso, escolhendo
vemos que

Ca,b

x = y = 0,

tambm contm o zero.

Pelo princpio da boa ordenao, podemos escolher

x0

y0

tais que

= ax0 +by0 seja o menor nmero inteiro positivo contido no conjunto


Ca,b .
Agora mostraremos que

|a

| b.

Provaremos que

|a

e o

outro segue analogamente. Usaremos para este propsito o mtodo de


reduo ao absurdo, ou seja, vamos supor que
contradio.

-a

e obteremos uma

Mximo Divisor Comum e Mnimo Mltiplo Comum

3.3

- a segue que
0 < r < . Portanto,

Usando a diviso euclidiana, de


e

tais que

a = q + r

com

109
existem inteiros

r = a q = a q(ax0 + by0 ) = a(1 qx0 ) + b(qy0 )


e assim

est no conjunto

Ca,b ,

o que contradiz a hiptese de

menor elemento positivo contido em

ser o

Ca,b .

divide a e b s resta provar que = d. Com


d = (a, b), podemos escrever a = da1 , b = db1 e

Uma vez que


desde que

efeito,

= ax0 + by0 = d(a1 x0 + b1 y0 ).


Assim

d | .

Logo pela parte (c) da Proposio 3.3, conclumos que

d . Agora d <
d = = ax0 + by0 .

impossvel pois

d =

mdc(a, b), e portanto

A seguinte proposio resume algumas consequncias importantes


da demonstrao dada ao teorema de Bzout.

Proposio 3.24. Sejam d, N e a, b, c Z. Ento valem as


seguintes armaes:

(a) Se d | a e d | b, ento d | (a, b).


(b) O mdc.(a, b) o menor valor positivo de ax + by , onde x e y
percorrem todos os nmeros inteiros.
(c) (a, b) = (a, b).
(d) Se d | a e d | b, ento ( ad , db ) = d1 (a, b). Consequentemente,


a
b
,
(a, b) (a, b)


= 1.

110

Divisibilidade

(e) Se (a, c) = (b, c) = 1, ento (ab, c) = 1.


(f) Se c | ab e (b, c) = 1, ento c | a.
Demonstrao.

A prova de (a) consequncia imediata da igualdade

(a, b) = ax0 + by0

anunciada no teorema de Bzout; assim como (b)

segue diretamente da demonstrao dada a este teorema.


Para provar (c), primeiro observamos que

(a)x + (b)y = (ax + by)


Usando o item (a) e o fato de

onde

x, y Z.

ser positivo, da igualdade acima segue

que



(a, b) = min (a)x + (b)y > 0; x, y Z


= min ax + by ; x, y Z
= (a, b).

A armao feita em (d) segue diretamente de (c), observando que





a b
a b
(a, b) = d , d
=d
,
.
d d
d d
Continuamos com a prova de (e). De
existem inteiros

xj

yj , j = 1, 2,

(a, c) = (b, c) = 1,

tais que

ax1 + cy1 = 1,
bx2 + cy2 = 1.
Multiplicando lado a lado as igualdades obtemos

(x1 x2 )ab + (ax1 y2 + y1 bx2 + cy1 y2 )c = 1.


|{z}
|
{z
}
x

temos que

Mximo Divisor Comum e Mnimo Mltiplo Comum

3.3

Ento, usando o item (b) e a igualdade acima resulta que

111

(ab, c) = 1.

Finalmente, provaremos (f ). Das hipteses temos que existem inteiros

x0

y0

tais que

bx0 + cy0 = 1.
Multiplicamos a igualdade acima por

a em ambos lados para obtermos

abx0 + acy0 = a.
Por outro lado,

ab = cq

para algum inteiro

q.

Usando esta condio

na ltima igualdade temos que

cqx0 + acy0 = c(qx0 + ay0 ) = a,


logo

c | a.

3.3.2

Algoritmo de Euclides

Apesar de conhecermos propriedades tericas do mdc entre dois inteiros, encontr-lo de fato pode ser uma tarefa complicada, sem auxlio
das ferramentas corretas. Lembrando o seu signicado, o leitor talvez
pudesse pensar que devemos calcular todos os divisores de
os divisores de

a,

todos

e descobrir qual o maior elemento comum aos dois

conjuntos.
Para achar o mdc se faz uso de um importante mtodo denominado

algoritmo de Euclides .

Teorema 3.25 (Algoritmo de Euclides) . Dados dois inteiros positivos,


a e b, aplicamos sucessivamente a diviso euclidiana para obter a se-

112

Divisibilidade

guinte sequncia de igualdades

b = aq1 + r1 ,

a = r1 q2 + r2 ,

r = r q + r ,
1
2 3
3

rn2 = rn1 qn + rn ,

r
n1 = rn qn+1 ,

0 r1 < a,
0 r2 < r1 ,
0 r3 < r2 ,

(3.20)

0 rn < rn1 ,

at algum rn dividir rn1 . Assim, o mdc.(a, b) = rn , ou seja, o


ltimo resto no-nulo no processo de diviso anterior.

Observao 3.26. Quando lidamos com nmeros pequenos achar o


mdc uma tarefa fcil pois podemos calcular o mdc valendo-nos das
fatoraes dos nmeros envolvidos. No entanto, quando estamos trabalhando com nmeros grandes o algoritmo de Euclides, em geral,
mais fcil que a fatorao, podendo ser esta ltima bem difcil.
Demonstrao do algoritmo de Euclides.

Comeamos observando que

o processo de diviso (3.20) nito. Com efeito, a sequncia de nme-

rk estritamente decrescente e est contida no conjunto


{r Z, 0 r < a}, portanto no pode conter mais do que a intei-

ros inteiros

ros positivos. Examinando as igualdades (3.20) de cima para baixo e


usando a Proposio 3.22 temos que

(a, b) = (a, r1 ) = (r1 , r2 ) = = (rn1 , rn ) = rn .

Mximo Divisor Comum e Mnimo Mltiplo Comum

3.3

113

Observao 3.27. Notemos que o teorema de Bzout tambm pode

ser obtido como consequncia do processo de diviso (3.20). Com


efeito, podemos escrever
rn
= rn2 rn1 qn o
rn = rn2 (rn3 rn2 qn1 )qn .
rn1 = rn3 rn2 qn1

Logo, conseguimos escrever rn em termos de rn2 e rn3 . Utilizando a


expresso rn2 = rn4 rn3 qn2 podemos escrever rn como combinao de rn3 e rn4 . Repetindo este processo vrias vezes, conclumos
que existem x, y Z tais que
d = rn = xr1 + yr2 .

Ora, como r1 = b aq1 e r2 = a r1 q2 = a(1 + q1 q2 ) bq2 , ento,


substituindo estes valores na ltima igualdade obtemos o Teorema de
Bzout.

Exemplo 3.28. Achar o mximo divisor comum dos nmeros 471 e


1.176.

Soluo.

Aplicando o algoritmo de Euclides obtemos a seguinte sequn-

cia de divises com resto:

1176 = 471 2 + 234,


471 = 234 2 + 3,
234 = 78 3,

ento o mdc(471, 1176)

= 3.

Exemplo 3.29. Provar que a frao


nmero natural n.

2n + 8
irredutvel para todo
4n + 15

114

Soluo.

Divisibilidade

Usando o algoritmo de Euclides temos que

4n + 15 = (2n + 8) 1 + 2n + 7,
2n + 8 = (2n + 7) 1 + 1,
2n + 7 = (2n + 7) 1.

Ento o mdc. (4n

+ 15, 2n + 8) = 1

e portanto

primos entre si para qualquer valor de

4n + 15

2n + 8

so

n.

Exemplo 3.30. Achar o mdc.(111


. . 111}, 11
. . 11})
| .{z
| .{z
100 vezes

Soluo.

60 vezes

Primeiro escrevemos os nmeros na base decimal, isto ,

111
. . 111} = 1099 + 1098 + + 1,
| .{z
100 vezes

11
. . 11} = 1059 + 1058 + + 1.
| .{z
60 vezes

Aplicamos agora o algoritmo de Euclides para obter as seguintes igualdades

111
. . 111} = (1059 + 1058 + + 1)1040 + 1039 + 1038 + + 1,
| .{z
100 vezes

1059 + 1058 + + 1 = (1039 + 1038 + + 1)1020 +


+ 1019 + 1018 + + 1,

1039 + 1038 + + 1 = (1019 + 1018 + + 1)1020 +


+ 1019 + 1018 + + 1.

Disso resulta que

19
18
. . 11} .
} | {z } = 10 + 10 + + 1 = |11 .{z

mdc.(111 . . . 111, 11 . . . 11)

{z

100 vezes

60 vezes

20 vezes

Mximo Divisor Comum e Mnimo Mltiplo Comum

3.3

3.3.3

115

Mnimo Mltiplo Comum

Agora passamos ao segundo conceito importante desta seo. O mesmo


est relacionado com os inteiros positivos que so simultaneamente
mltiplos de dois inteiros prexados e denominado

mnimo mltiplo

comum.

Denio 3.31

(Mnimo Mltiplo Comum)

. Sejam a e b inteiros

diferentes de zero. O mnimo mltiplo comum, resumidamente mmc,


entre a e b o inteiro positivo m que satisfaz as seguintes condies:
(a) m um mltiplo comum de a e b, isto , a | m e b | m;
(b) m o menor inteiro positivo com a propriedade (a).

Neste caso, denotamos o mmc entre a e b por m = mmc(a, b) ou por


m = [a, b].
Resumimos a seguir algumas das propriedades fundamentais do
mmc de dois inteiros.

Proposio 3.32. Sejam a, b, c Z e Z. Ento valem as se-

guintes armaes:

(a) se c mltiplo comum de a e b, ento [a, b] | c;


(b) [a, b] = [a, b];
(c) |ab| = [a, b] (a, b).
Demonstrao.
de

por

[a, b]

Comeamos com a prova de (a). A diviso com resto

nos d

c = [a, b]q + r,

0 r < [a, b].

(3.21)

116

Da igualdade anterior, basta provar que


desejado. Suponhamos, pelo contrrio,
que tanto

como

dividem

r = 0 para obter o
que 0 < r < [a, b].

[a, b].

mltiplo comum de

resultado
Notemos

Logo, pelo item (b) da Pro-

posio 3.3 e a igualdade (3.21), temos que


ou seja,

Divisibilidade

r,
[a, b],

tambm dividem

e no pode ser menor que

contradizendo nossa suposio.


Prosseguimos com a prova de (b). Observemos que
plo comum de

b,

[a, b]

mlti-

logo pelo item (i) vale que

[a, b] [a, b].


Por outro lado, [a, b] = q1 a = q2 b, para alguns inteiros
[a,b]
logo,
um mltiplo comum de a e b. Portanto,

[a, b]

[a, b]
[a, b] [a, b].

(3.22)

q1

q2 ;

(3.23)

Das igualdades (3.22) e (3.23) segue que

[a, b] [a, b] [a, b],


de onde vem diretamente o resultado.
Para provar (c) podemos supor sem perda de generalidade que

so positivos devido s igualdades

[a, b] = [a, b] = [a, b] = [a, b].


Dividiremos a prova em dois casos:

Caso 1: (a, b) = 1.
b | [a, b] e [a, b] = qa, para algum q N. Ento b | qa
e alm disso (a, b) = 1. Logo, pelo item (v) da Proposio 3.24 temos
que b | q . Portanto, b q e consequentemente
Sabemos que

ab aq = [a, b].

(3.24)

3.3

Mximo Divisor Comum e Mnimo Mltiplo Comum

Entretanto, da denio de

[a, b]

117

vale que

[a, b] ab.

(3.25)

Das desigualdades (3.24) e (3.25) segue que

ab = [a, b] = [a, b] 1 = [a, b] (a, b).

ab [a, b] ab.

Caso 2: (a, b) > 1.


Da parte (c) da Proposio 3.24 sabemos que

a
, b
(a,b) (a,b)

Assim,

= 1.

Aplicando o caso anterior vale que


 

a
a
b
b
a
b

=
,
,

.
(a, b) (a, b)
(a, b) (a, b)
(a, b) (a, b)
Multiplicamos esta ltima igualdade por

(a, b)2

e usamos o item (b)

provado anteriormente, assim como a parte (d) da Proposio 3.24


para obter




a
b
a
b
ab = (a, b)
,
(a, b)
,
= [a, b] (a, b).
(a, b) (a, b)
(a, b) (a, b)

Exemplo 3.33. Dois amigos passeiam de bicicleta, na mesma direo, em torno a uma pista circular. Para dar uma volta completa um
deles demora 15 minutos e o outro demora 18 minutos. Eles partem
juntos e combinam interromper o passeio quando os dois se encontrarem pela primeira vez no ponto de partida. Quantas voltas deu cada
um?
Soluo.

Denotemos por

n1

n2 , respectivamente, o nmero de voltas

que d cada um dos amigos. Notemos que o tempo total da corrida


o menor valor positivo de

que satisfaz as igualdades

T = 15n1 = 18n2 ,

118

Divisibilidade

ou seja

T = [15, 18] =
Portanto,

n1 = 6

15 18
= 90.
3

n2 = 5.

Finalizamos esta seo com um exemplo que nos fornece uma bela
interpretao geomtrica do mnimo mltiplo comum.

O mesmo foi

proposto na Olimpada Brasileira de Matemtica.

Exemplo 3.34. Um retngulo de lados inteiros AB = m e CD = n,


dividido em quadrados de lado 1. Em cada um dos vrtices ele possui
um pequeno orifcio. Um raio de luz entra no retngulo por um dos
vrtices, na direo da bissetriz do ngulo reto, e reetido sucessivamente nos lados do retngulo. Quantos quadrados so atravessados
pelo raio de luz?
D

Figura 3.2: Interpretao geomtrica do mmc

Soluo.
n,

me
mmc( m,n).

Se zermos alguns testes preliminares dando valores a

veremos que em cada caso a resposta coincidir com o

Provemos que isto de fato vale para

prova nos auxiliaremos da Figura 3.2.

quaisquer. Para realizar a

Mximo Divisor Comum e Mnimo Mltiplo Comum

3.3

119

Primeiramente, notemos que cada vez que o raio de luz atravessa


um quadrado ele avana uma unidade tanto na direo horizontal como
na direo vertical. Usando este fato fazemos as observaes a seguir.

Se o raio entra pelo vrtice

A,

ter que atravessar

quadrados

BC , imediatamente mais m para chegar ao


lado AD , depois mais m para chegar novamente ao lado BC , e
assim sucessivamente. Alm disso, depois do raio percorrer pm
quadrados, com p N, estar batendo no lado BC ou no lado
AD.
at chegar ao lado

Analogamente o raio bater no lado

Somente nos vrtices

somente se, atravessar

qn

AB

quadrados, com

B, C

ou no lado

DC

se, e

q N.

do retngulo pode acontecer que

o raio incidente saia do retngulo, terminando assim o processo


de reexo.

Usando as observaes acima fcil ver que o raio chegar a um


vrtice quando chegar simultaneamente a dois lados perpendiculares
do retngulo. Portanto, deve ter atravessado um nmero
dos tal que

ou seja,

dever ser um mltiplo comum

n. claro que a primeira vez que o raio chega a um vrtice


nmero x o menor mltiplo comum de m e n, isto , x = [m, n].
de

x = pm = qn,

x de quadra-

Finalmente, observamos que nenhum dos quadrados atravessado


duas vezes no percurso do raio de

A at bater no primeiro vrtice, pois

como vemos na gura numa das direes os quadrados atravessados


sero todos cinzas e na outra direo, sero todos brancos.

120

3.3.4

Divisibilidade

Equaes Diofantinas Lineares

Consideremos a equao

ax + by = c,
onde

a, b, c Z,

com

a 6= 0

b 6= 0.

(3.26)

equao diofantina linear

A equao (3.26) chamada de

soluo desta qualquer par de inteiros

(x, y)

e uma

que satisfaam (3.26).

(x, y),

conhecido que todos os pontos do plano, com coordenadas

que satisfazem a igualdade (3.26) representam, geometricamente, uma


reta. Logo, as solues de uma equao diofantina linear so os pontos
de coordenadas inteiras do plano cartesiano, que esto dispostos sobre
a reta que esta representa. Por exemplo, os pontos
so solues da equao diofantina
3

3x 2y = 1,

(1, 2)

(1, 1)

veja a Figura 3.3.

1
0

-1

-2
-3
-3

-2

-1

Figura 3.3: A equao da reta

3x 2y = 1.

Naturalmente nos perguntamos: sempre possvel achar solues


para uma equao diofantina linear?

A resposta no; o prximo

resultado nos diz quando isto possvel. Alm disso, se uma equao
diofantina linear tem uma soluo na verdade ela tem uma innidade
de solues.

Mximo Divisor Comum e Mnimo Mltiplo Comum

3.3

121

Proposio 3.35. A equao diofantina linear


ax + by = c,

a, b, c Z, com a 6= 0 e b 6= 0,

(3.27)

tem soluo se, e somente se, d | c, onde d = (a, b). Alm disso,
se (x0 , y0 ) uma soluo, ento o conjunto de solues de (3.27)
constitudo por todos os pares de inteiros (x, y) da forma:
x = x0 + t db

Demonstrao.

e y = y0 t ad ,

t Z.

(3.28)

(x0 , y0 ) uma soluo


ax0 + by0 = c. Usando que d = (a, b) sabemos que
q1 e q2 , tais que dq1 = a e dq2 = b. Portanto, se

Primeiramente suponhamos que

de (3.27), logo
existem inteiros

verica a igualdade

dq1 x0 + dq2 y0 = d(q1 x0 + q2 y0 ) = c,


de onde segue obviamente que

d | c.

Reciprocamente, suponhamos que

d|c

e portanto

c = qd

com

inteiro. O teorema de Bzout nos garante a existncia de dois inteiros,

x0

y0 ,

ltima

ax0 + by0 = d. Multiplicando


igualdade por q temos que
tais que

ambos os lados desta

ax0 q + by0 q = c,
logo o par

(x1 , y1 ),

com

x1 = x 0 q

y1 = y0 q ,

soluo da equao

diofantina.
Resta provar agora que temos innitas solues da forma (3.28).

(x, y) uma outra soluo qualquer alm de (x0 , y0 ),


vale que ax0 + by0 = c = ax + by , de onde ax0 + by0 = ax + by . Desta
igualdade obtemos a(x x0 ) = b(y0 y) e dividimos esta ltima por
d para obtermos
a
b
(x x0 ) = (y0 y).
d
d

Com efeito, sendo

122

Divisibilidade

Como
existe

( ad , db ) = 1, ento
inteiro t tal que

temos que

x = x0 + t db

a
d

| (y0 y)

b
d

| (x x0 ).

y = y0 t ad .

Por outro lado, fcil vericar que para qualquer inteiro


achadas acima para

Logo,

t as expresses

resolvem a equao diofantina.

A seguir damos um exemplo de como proceder para resolver equaes diofantinas.

Exemplo 3.36. Achar todas as solues inteiras da equao


12x + 33y = 27.

Soluo.

Observemos que

(12, 33) = 3

o tem innitas solues.


e teremos as restantes.

e que

3 | 27,

logo a equa-

Como sabemos, basta achar uma delas

Para achar esta soluo particular podemos

trabalhar de duas maneiras, que descrevemos a seguir:

Alternativa 1:

reduzimos a equao forma equivalente

4x + 11y = 9,
e por tentativa e erro vemos que

x0 = 5

y0 = 1

solucionam a

mesma. Ento pela Proposio 3.35 temos que

x = 5 + 11t

y = 4t 1,

t Z,

esgotam todas as solues que procuramos.

Alternativa 2:
mdc

(12, 33),

aplicamos o algoritmo de Euclides para achar o

obtendo os seguintes resultados:

33 = 12 2 + 9,
12 = 9 1 + 3,
9 = 3 3 + 0.

3.4

Nmeros Primos e Compostos

123

Da segunda e primeira igualdades temos, respectivamente, que

3 = 12 9 1

9 = 33 12 2.

Usando estas duas obtemos

3 = 12 (33 12 2) 1
= 12 33 + 12 2
= 3 12 1 33,

x0 = 3 e y0 = 1, garantidos pelo teorema de


Bzout, que validam 3 = 12x0 +33y0 . Multiplicamos por 9 esta ltima
ou seja, achamos

igualdade para obter

27 = 12(9x0 ) + 33(9y0 ).
Portanto,

x
e0 = 9x0 = 27

a equao diofantina.

ye0 = 9y0 = 9 resolvem, particularmente,

Analogamente, como na alternativa anterior,

podemos escrever a soluo geral da forma:

x = 27 + 11s

3.4

y = 4s 9,

s Z.

Nmeros Primos e Compostos

Ao longo da histria da Matemtica, os nmeros primos foram protagonistas de clebres problemas que motivaram o desenvolvimento
de teorias e tcnicas pelas mentes mais frteis, como Fermat, Euler e
Gauss.

At hoje muitos desses problemas, simples de enunciar, que

envolvem nmeros primos so desaos intelectuais para toda a humanidade.

124

Divisibilidade

Esta seo ser dedicada ao estudo de propriedades bsicas dos


nmeros primos.

Todo nmero natural

menos 2 divisores, claramente 1 e

n.

maior do que 1 tem pelo

Isto motiva a seguinte denio.

Denio 3.37 (Nmeros Primos e Compostos) . Um inteiro positivo


n 2 dito primo se os nicos divisores que ele tem so 1 e ele

prprio; caso contrrio, dito composto.

Observao 3.38. De modo geral o nmero 1 no considerado nem


primo nem composto.

Exemplo 3.39. Os nmeros 2, 3, 5, 7, e 11 so primos e os nmeros


10, 15, 35 e 348 so compostos.

Exemplo 3.40. O nmero n = 220 254 composto.


Soluo.

Escrevemos

de outra forma, com o objetivo de facilitar

nosso trabalho. Com efeito, observemos que

n = (210 )2 (252 )2 = 10242 6252 ,


logo composto por ser diferena de quadrados. Alm disso,

n = 10242 6252 ,

= (1024 625)(1024 + 625),

= 399 1649,

= 3 133 1649.
Portanto, podemos concluir que

3 | n.

Proposio 3.41. Seja n > 1 um nmero inteiro. Ento


(a) o menor divisor de n diferente de 1 um nmero primo;

(3.29)

Nmeros Primos e Compostos

3.4

125

(b) se n composto, o seu menor divisor diferente de 1 no maior

que n. Em outras palavras, se n no possui divisores diferentes

de 1, menores ou igual que n, ento n primo.


Demonstrao.
n, diferente de
1 < q < p; mas

Comeamos provando (a). Seja


1. Se

q | n,

n = pq com
por p obtemos

Portanto,

desigualdade

tal que

p | n,

e isto contradiz a hiptese levantada sobre

p
q p.

Para provar (b) denotamos por


de 1.

o menor divisor de

fosse composto teria algum divisor

q|p
o que nos diz que

o menor divisor de

n,

p.

diferente

Multiplicando ambos lados da

n = pq p2 ,

e consequentemente vale

n p.

Agora vamos enunciar um dos resultados mais clssicos da Matemtica, que garante a existncia de innitos nmeros primos.

At

onde se conhece, a demonstrao a seguir foi a primeira demonstrao


escrita utilizando o mtodo de reduo ao absurdo e devida a Euclides cerca de 300 a.C. Para outras seis provas, incluindo a moderna
prova de Fustenberg, recomendamos os livros [1] e [10].

Teorema 3.42 (Teorema de Euclides). A quantidade de nmeros primos innita.

Demonstrao.

Faremos a prova por reduo ao absurdo.

Suponha

que existe uma quantidade nita de nmeros primos e denotemos estes


por

p1 , p2 , p3 , . . . , pk .

126

Divisibilidade

Consideremos o nmero

n = p1 p2 p 3 pk + 1
e chamemos de

o seu menor divisor primo. Obviamente

cide com nenhum dos nmeros


como ele divide

n,

pi , 1 i k ,

no coin-

pois caso contrrio,

teria que dividir 1, o que impossvel. Logo, te-

mos uma contradio hiptese de termos uma quantidade nita de


primos.
Os nmeros primos tambm podem ser caracterizados da seguinte
maneira:

Proposio 3.43. Um inteiro positivo p primo se, e somente se,


satisfaz a seguinte propriedade:

p | ab = p | a ou

p|b

(3.30)

Primeiramente, suponhamos que

p primo e que p - b,

onde a, b Z.
Demonstrao.
logo

(p, b) = 1.

Ento, pelo item (f ) da Proposio 3.24 temos que

p | a.

Reciprocamente, suponhamos que, a propriedade 3.30 vlida e

alm disso vamos supor, pelo absurdo, que

p = d1 d2 ,
De (3.30) segue que

com

p | d1

ou

no primo. Ento,

1 < d1 < p, 1 < d2 < p.

p | d2 ;

p d1 ,

ou

(3.31)

consequentemente

p d2 ,

contradizendo isto o armado em (3.31).

(3.32)

Procurando Primos

3.5

3.5

127

Procurando Primos

Os nmeros primos alm de belos e desaadores do ponto de vista


matemtico, so extremamente importantes para as atividades usuais
de nosso dia a dia. Por exemplo, nenhuma transao bancria ou pela
internet estaria segura sem o uso de nmeros primos muito grandes.
Assim, surge naturalmente a pergunta de como podemos produzi-los
em grandes quantidades. Essa pergunta sempre intrigou os matemticos e continua sem soluo at os dias atuais.

Apesar deles serem

abundantes, em quantidade innita de acordo com o Teorema 3.42,


no existe nenhum mtodo razovel de produo de nmeros primos,
mesmo tendo em mos a alta tecnologia de hoje em dia. Porm, ao
longo do tempo algumas frmulas e algoritmos se mostraram teis
para a descoberta de nmeros primos.

3.5.1

O Crivo de Eratstenes

O crivo de Eratstenes

um algoritmo que nos permite achar todos

os nmeros primos que so menores ou iguais que um natural

dado.

Segundo a tradio, este mtodo foi criado pelo matemtico grego


Eratstenes (285-194 a.C.).
O mtodo consiste nos seguintes passos: escrevemos os nmeros de
forma ordenada a partir de 2, isto ,

2, 3, 4, 5, 6, 7, 8, 9, 10, 11, 12, 13, 14, 15, 16, 17, . . . , n

(3.33)

Observamos que o primeiro primo que aparece em (3.33) 2 e


imediatamente apagamos da lista (3.33) todos os mltiplos de
2 maiores que ele, por serem compostos; resta assim a seguinte

128

Divisibilidade

lista

2, 3, 5, 7, 9, 11, 13, 15, 17 . . .

O primeiro nmero no apagado que aparece na lista restante


3, que tambm primo. Imediatamente apagamos da lista todos
os mltiplos de 3 maiores que ele, por serem compostos; resta
agora a lista

2, 3, 5, 7, 11, 13, 17, . . .

O primeiro nmero no apagado que aparece na lista que restou


do passo anterior 5, que tambm primo.

Imediatamente

apagamos da lista todos os mltiplos de 5 maiores que ele, por


serem compostos.

Repetimos este processo at que o primeiro nmero no apagado


da lista em questo seja maior que

n, pois graas Proposio

3.41-(b) a partir desse momento todos os nmeros restantes so


os primos menores ou iguais que

n = 40,

Por exemplo, se

n..

temos que

40 = 6, 324555.

aplicando o mtodo:
2

10

11

12

13

14

15

16

17

18

19

20

21

22

23

24

25

26

27

28

29

30

31

32

33

34

35

36

37

38

39

40

Passo 1: ordenamos os nmeros

11

13

15

17

19

21

23

25

27

29

31

33

35

37

39

Ento,

Procurando Primos

3.5

129

Passo 2: tiramos os mltiplos de 2

2
11

13
23

31

7
17

25
35

19
29

37

Passo 3: tiramos os mltiplos de 3

2
11

13

7
17

23

19
29

31

37
Passo 4: tiramos os mltiplos de 5

Como

72 = 49 > 40,

paramos agora.

Observao 3.44. Note que ao comear a apagar os mltiplos de um

nmero primo p podemos comear a apagar a partir de p2 , pois se


supomos que existe um nmero composto m no apagado menor que
p2 , temos que m = p1 q1 , sendo p1 seu menor divisor primo. Ento,

pelo item (b) da Proposio 3.41, p1 < m < p, logo m deveria ter
sido apagado pois mltiplo de um primo menor que p.

3.5.2

Primos de Mersenne

Marin Mersenne (1588-1648) foi um monge francs que nasceu na cidade de Maine e foi um dos grandes inuenciadores da Matemtica

130

Divisibilidade

11

13

17

19

23

29

31

37

41

43

47

53

59

61

67

71

73

79

83

89

97

101

103

107

109

113

127

131

137

139

149

151

157

163

167

173

179

181

191

193

197

199

211

223

227

229

233

239

241

251

257

263

269

271

277

281

283

293

307

311

313

317

331

337

347

349

353

359

367

373

379

383

389

397

401

409

419

421

431

433

439

443

449

457

461

463

467

479

487

491

499

503

509

521

523

541

Tabela 3.1: Os primeiros 100 nmeros primos

francesa nos sculos XVI e XVII. Apaixonado pelos nmeros, teve entre seus correspondentes Descartes, Fermat, Pascal e Galileu. Entre
suas vrias descobertas, ele estudou os nmeros da forma:

Mn = 2n 1.
Observe que vale o seguinte fato a respeito desses nmeros:

Proposio 3.45. Se Mn primo, ento n primo.


Demonstrao.

Provar essa proposio equivale a mostrar que a sua

forma contrarrecproca vale. Ou seja, que se

n = a.b,

com

a b > 1,

ento

Mn

composto, digamos

tambm composto.

De fato,

usando o Lema 3.14, podemos decomp-lo do seguinte modo:



Ma.b = 2ab 1 = 2a(b1) 2a(b2) + + 2a + 1 2b 1 .

3.5

Procurando Primos

131

Porm, no verdade a recproca da armao acima. Por exemplo, Hudalricus Regius mostrou em 1536 que
no primo, j que

2.047 = 23 89.

M11 = 211 1 = 2.047

Em 1643, Mersenne armou que para

n = 2, 3, 5, 7, 13, 17, 19, 31, 67, 127 e 257,


os valores de
menores que

Mn so todos primos
257, Mn composto.

e para todos os outros valores de

Hoje sabemos que Mersenne errou na sua armao, esquecendo


trs valores de

n onde Mn primo:

61, 89 e 107 e incluindo

M67 e M257

como nmeros primos. Para mais informaes, sugerimos a pgina

web

http://primes.utm.edu/mersenne/index.html.
Finalizamos esta seo, com um critrio interessante, devido matemtica francesa Sophie Germain (1776-1831), que nos permite saber
quando um nmero no primo.

Proposio 3.46

(Identidade de Sophie Germain)

vale a igualdade

. Dados a, b R,

a4 + 4b4 = (a2 + 2b2 + 2ab)(a2 + 2b2 2ab).

Demonstrao.

A prova segue das seguintes igualdades:

a4 + 4b4 = a4 + 4a2 b2 + 4b4 4a2 b2


= (a2 + 2b2 )2 4a2 b2

= (a2 + 2b2 + 2ab)(a2 + 2b2 2ab).

Como aplicao desta identidade vejamos os seguintes exemplos.

132

Divisibilidade

Exemplo 3.47. qn = n4 + 4n composto, para todo n N.


Soluo.

O conjunto dos nmeros naturais particionado em duas

classes disjuntas:o conjunto dos nmeros pares e o conjunto dos nmeros mpares. Estudaremos cada classe por separado. Assim,

n um nmero par, ento n = 2m para algum inteiro positivo


m 1. Deste modo,
se

n4 + 4n = (2m)4 + 42m = 16m4 + 24m ,



= 2 8m4 + 24m1 .
Portanto, neste caso,

n4 + 4n 2.

Logo, se

nmero inteiro positivo par temos que

n +4

n>1

qualquer

no um nmero

primo;

se

um nmero mpar, ento

positivo

m 1.

n = 2m + 1

para algum inteiro

Assim,

n4 + 4n = (2m + 1)4 + 42m+1 = (2m + 1)4 + 4 42m

= (2m + 1)4 + 4 24m = (2m + 1)4 + 4 (2m )4 .

Logo, tomando

a = 2m + 1

b = 2m ,

o resultado uma con-

sequncia direta da identidade de Sophie Germain.

Exemplo 3.48. 520 + 230 um nmero composto.


Soluo.

Escrevemos

520 + 230 = 554 + 22 228 = 55

4

+ 4 27

4

de onde podemos usar a Identidade de Sophie Germain com

b=2

para comprovar que o nmero

20

5 +2

30

composto.

a = 55

Procurando Primos

3.5

3.5.3

133

O Teorema Fundamental da Aritmtica

Os nmeros primos so as

clulas

dos nmeros naturais, no sentido

de que qualquer nmero natural produto de nmeros primos. Por


exemplo,

560 = 56 10 = 7 8 5 2 = 7 2 2 2 5 2,
onde cada um dos fatores que aparecem no produto so nmeros primos. Perguntamo-nos, o que acontece se comeamos com uma outra
fatorao inicial de 560, por exemplo,

560 = 28 20.

Vejamos:

560 = 28 20 = 14 2 10 2 = 7 2 2 5 2 2.
Surpreendentemente chegamos mesma representao anterior, salvo
a ordem dos fatores.

2
2
2
2

Figura 3.4: O nmero 560 composto de 4 clulas do tipo 2, uma clula

do tipo 7 e uma clula do tipo 5.

O fato observado acima vale para qualquer nmero natural maior


que 1.

Especicamente, temos o seguinte resultado conhecido como

teorema fundamental da aritmtica .

134

Divisibilidade

Teorema 3.49 (Teorema Fundamental da Aritmtica) . Todo nmero


natural n maior que 1 pode ser escrito como um produto
n = p1 1 p2 2 p3 3 pmm ,

(3.34)

onde m 1 um nmero natural, i N e pi primo para todo


1 i m . Alm disso, a fatorao em (3.34) nica se exigirmos
que p1 < p2 < < pm .
Demonstrao.

Seja

um inteiro maior que 1. Denotando por

p1

seu

menor divisor primo tem-se que

n = p1 1 ,
Se

1 = 1,

contrrio,

1 1 < n.

N1 = p1 e a fatorao desejada obtida. Caso


denotando por p2 o menor divisor primo de 1 tem-se que
ento

n = p1 p2 2 ,

1 2 < 1 .

2 = 1, ento n = p1 p2 e novamente chegamos fatorao desejada.


Caso contrrio, denotando por p3 o menor divisor primo de 2 tem-se

Se

que

n = p1 p2 p3 3 ,

1 3 < 2 .

Continuando este processo sucessivamente obtemos ento uma sequncia estritamente decrescente de nmeros naturais

n ,

ou seja,

n > 1 > 2 > 2 > > n > n+1 > 1,


Ento, pelo princpio da boa ordem, s pode existir uma quantidade
nita de ndices

tais que

n > 1

e consequentemente

onde segue que

n = p1 p2 pn .

n+1 = 1,

de

3.5

Procurando Primos

135

Notemos que na representao acima os

pi

podem-se repetir, resul-

tando nalmente a representao desejada em (3.34).


Provaremos agora a unicidade de tal fatorao. Com efeito, suponha que existem duas fatoraes:

p1 1 p2 2 p3 3 pmm = n = q11 q22 q33 qss


Pela Proposio 3.43 temos que cada

qj ,

por serem primos.

Portanto, cada

pi

pi

divide algum

qj ,

logo

pi =

aparece no lado direito da

qj tambm
aparece no lado esquerdo da igualdade. Ento, como os pi s e os qj s
so diferentes dois a dois e organizados crescentemente, temos m = s
igualdade acima, e, um argumento anlogo nos d que cada

e a igualdade se reduz a

p1 1 p2 2 p3 3 pmm = p1 1 p2 2 p3 3 pmm .
1 seja diferente de 1 ;
que 1 < 1 . Portanto,

Suponhamos agora que


neralidade vamos supor

sem perda de ge-

p2 2 p3 3 pmm = p1 1 1 p2 2 p3 3 pmm ,
1 1 > 0 ento, pela Proposio 3.43 temos que p1 divide algum pj , com j > 1, o que impossvel. Portanto, 1 = 1 .
Similarmente provamos que i = i , com i = 1, . . . , n.
e como

Observao 3.50. O teorema fundamental da aritmtica foi enun-

ciado precisamente por Gauss (1777-1855). Seus antecessores, Fermat, Euler, Lagrange e Legendre, utilizavam este teorema sem a preocupao de t-lo enunciado ou demonstrado com preciso. Uma prova
alternativa deste teorema ser apresentada no Captulo 6, usando o
mtodo de induo.

136

Divisibilidade

Exemplo 3.51. Prove que um nmero n par se, e somente se, o

nmero 2 aparece na fatorao de n em fatores primos.


Soluo.
ento

Obviamente, se 2 aparece na fatorao em primos de


par. Ora, se

par temos que

n = 2q .

Por outro lado

N,
qe

se fatoram, respectivamente, como

m
q = q11 q22 qm

n = p1 1 p2 2 ps s .

Logo,

m
2 q11 q22 qm
= p1 1 p2 2 ps s .
Pela unicidade da fatorao, para algum
respondente

pi

i,

com

1 i s,

o cor-

deve ser igual a 2. Portanto, 2 aparece na fatorao de

n.

Exemplo 3.52. Seja A = {1, 2, 3, 4, 5, 6, 7}. possvel decompor


o conjunto A em dois subconjuntos disjuntos tais que o produto dos
elementos de um seja igual ao produto dos elementos do outro?
Soluo.

Mostraremos que impossvel fazer esta decomposio. Com

efeito, suponha que existem tais conjuntos,

A2 = {q1 , q2 , . . . , qs }.

Ento

A1 = {p1 , p2 , . . . , pr }

p1 p2 pr = q1 q 2 qs
| {z } | {z }

e alm disso, como os conjuntos


nmero 5 aparece no produto

A1

A2

so disjuntos, temos que o

ou no produto

mas no em ambos

simultaneamente. Por outro lado, o Teorema 3.49 nos diz que a fatora-

igual fatorao em primos de , logo o nmero


aparecer tanto no produto como no produto , contra-

o em primos de
5 deveria

dizendo isto o fato anterior. Portanto no existe uma decomposio


com as condies exigidas.

Procurando Primos

3.5

137

Exemplo 3.53. Encontre todos os nmeros inteiros e positivos n com


a propriedade de que o conjunto

A = {n, n + 1, n + 2, n + 3, n + 4, n + 5}

pode ser particionado em dois subconjuntos tais que o produto dos


elementos de um dos subconjuntos seja igual ao produto dos elementos
do outro.
Demonstrao.

Digamos que seja possvel essa decomposio para al-

n e vamos denotar os conjuntos que obtemos com a decomposio


por A1 e A2 . Observando a decomposio dos elementos dos subconjuntos em fatores primos, temos que todo fator primo de A1 tambm
dever pertencer a A2 . No conjunto dos seis nmeros s podemos ter
um mltiplo de 7, por isso no podemos tomar n como mltiplo deste
primo. Analogamente para primos maiores que 7. Analisando o primo
5, conclumos que n e n + 5 so mltiplos de 5, pois se no, cairamos
na anlise anterior. Assim, os nmeros n + 1, n + 2, n + 3 e n + 4 so

da forma 2 3 . Como entre eles existem dois mpares, logo teremos
duas potncias de 3 cuja diferena 2, um absurdo. Assim, no existe
n que satisfaz as condies do enunciado.
gum

Finalizamos esta seo com um exemplo que mostra como podemos


combinar os fatos estudados para resolver problemas mais difceis

Exemplo 3.54. Encontre todos os nmeros que so formados por 4

algarismos da forma aabb e que sejam quadrados perfeitos.

138

Divisibilidade

Soluo.

Como o nmero

aabb

um quadrado perfeito, signica que:

n2 =aabb

n2 =103 a + 102 a + 10b + b = 103 + 102 a + (10 + 1) b
n2 =1100 a + 11 b


n2 =11 100a + b = 11 99a + a + b .

112 | N 2 .
Segue-se ento que 11 | (99a+a+b). Portanto, 11 | (a+b). Como aabb
tem 4 algarismos, segue-se que a 6= 0; portanto a {1, 2, 3, . . . , 9} e
b {0, 1, 2, . . . , 9}. De onde a + b 18. Logo, necessariamente
devemos ter a + b = 11. Podemos observar que a 6= 1, pois se a = 1
ento b = 10. Analogamente, b 6= 0, 1. Portanto,
Como 11 primo fcil ver, usando a Proposio 3.43, que

a {2, 3, 4, . . . , 9}

b {2, 3, 4, . . . , 9}.

Como em todo nmero quadrado perfeito o algarismo das unidades


somente pode acabar em

0, 1, 4, 5, 6

9.

Segue-se que

b {4, 5, 6, 9}.
b 6= 5, pois todo nmero que acaba em 5 quando elevado
quadrado sempre acaba em 25. Assim,

Certamente
ao

b {4, 6, 9}.

Se

b = 4,

ento

a = 7.

Neste caso o nmero seria

7.744

que

um quadrado perfeito;
Se

b = 6,

ento

a = 5.

Neste caso o nmero seria

um quadrado perfeito;

5.566

que no

Exerccios

3.6

Se

b = 9,

139
ento

a = 2.

Neste caso o nmero seria

que no

um quadrado perfeito.

aabb = 7.744 = 882 .

Finalmente, a nica soluo possvel

3.6

2.299

Exerccios

1. Encontre o resto que deixa


(a)

2001 2002 2003 2004 + 20052

(b)

2100

(c)

(1237156 + 34)

quando dividido por 7;

quando dividido por 3;

28

2. Provar que o nmero


nmero natural
3. Prove que se

quando dividido por 111.

n5 + 4n

divisvel por 5 para qualquer

n.
mpar

(a)

n3 n

divisvel por 24;

(b)

n2 1

divisvel por 8;

(c)

n2 + (n + 2)2 + (n + 4)2 + 1

4. O nmero

21093 2

5. Prove que

(999994)1234567890 1

6. O nmero

N = 42005 + 20054

divisvel por 12.

divisvel por

7. Demonstre que o nmero

10932

divisvel por

333331.

primo?

1 |000 {z
. . . 00} 1
2006 zeros

composto.

140

Divisibilidade

8. Utilizando o fato de que o resto de um quadrado quando dividido


por 4 s pode ser 0 ou 1, d uma outra soluo para o problema
do Exemplo 3.54.
9. Dados trs inteiros,

x, y, z ,

tais que

no so ambos mpares e que

xy

x2 + y 2 = z 2 ,

mostre que

mltiplo de 6.

10. Demonstre que o quadrado de um inteiro da forma


ou

8n ou 8n+1

8n + 4.

11. Trs nmeros primos

p, q

gresso aritmtica, ou seja,

r, maiores que 3, formam uma proq = p + d e r = p + 2d. Prove que d

divisvel por 6.
12. Demonstrar que existem innitos nmeros primos da forma

e da forma

6m + 5,

onde

4m+

m Z.

13. Encontrar o ltimo dgito dos nmeros


(a)

19892005 ;

(b)

777777 + 250 ;

(c)

1 + 22 + 32 + + 20052 .

14. Prove que a soma dos quadrados de cinco nmeros consecutivos


no um quadrado perfeito.
15. Prove que

1 |00 {z
00} 5 |00 {z
00} 1
100zeros

16. Seja

no um cubo perfeito.

100zeros

um inteiro positivo. Enuncie e prove o critrio de divisi-

bilidade por

no sistema de numerao de base

17. Prove que os nmeros

b.

3.6

Exerccios
(a)

(b)

141

1 1
1
+ + + , com n > 1,
2 3
n
1 1
1
n = + + +
, com n > 0,
3 5
2n + 1
n = 1 +

no so inteiros.

p(n) = am nm + am1 nm1 + + a0 de


grau m 1 com coecientes inteiros e n N. Prove que p(n)
um nmero composto para innitos valores de n.

18. Considere o polinmio

Sugesto: Use o fato de que existe a N tal que = |p(a)| > 1


e mostre que divide a p(k + a), para todo k Z.
19. Dizemos que um conjunto

An

formado por

inteiros positivos

escritos no sistema binrio (base 2) regular se, para qualquer

inteiro no negativo a quantidade de nmeros de

templam

An

que con-

na representao binria par. Dizemos que

irregular se, pelo menos para algum

An

s, este nmero mpar.

De-

monstre que um sistema irregular pode se converter em regular


excluindo-se apenas um nico elemento do mesmo, e, um sistema
regular pode se converter em irregular excluindo-se qualquer um
dos seus elementos.

20. Seja

n um inteiro positivo.

Demonstrar que todos os coecientes

do desenvolvimento do binmio de Newton


se, e somente se,

da forma

(a + b)n

so mpares

2 1.

(x0 , y0 ) uma soluo da equao diofantina linear


ax by = 1, ento a rea do tringulo cujos vrtices so (0, 0),
(b, a) e (x0 , y0 ) 1/2.

21. Prove que se

142

Divisibilidade

22. Qual a menor distncia possvel entre dois pontos

(x1 , y1 )

(x2 , y2 ), com coordenadas inteiras, situados sobre a reta denida


pela equao diofantina ax + by = c?

4
O Princpio da Casa dos
Pombos
Uma vez um matemti o me falou que o verdadeiro prazer no est
em a har a verdade, mas em pro urar por ela.
Leo Tolstoy

Um interessante instrumento elementar para tratar problemas matemticos relacionados existncia de elementos de conjuntos validando

princpio de Dirichlet , tambm conheprincpio da casa dos pombos (PCP) . Este princpio foi

certas exigncias o chamado


cido como

usado por Dirichlet (1805-1859) para resolver problemas na Teoria


dos Nmeros, entretanto ele possui um grande nmero de aplicaes
em diversos ramos da Matemtica como Combinatria e Geometria.
A seguir enunciamos a verso mais simples do PCP.

Proposio 4.1

(PCP  Verso Simples)

. Se distribumos N + 1

pombos em N casas, ento alguma das casas contm dois ou mais


pombos.
143

144

P1

P2

C1

C2

O Princpio da Casa dos Pombos

PN
CN

PN +1
Figura 4.1: Em cada casa

denotado por

Pj .

Cj , 1 j N ,

coloca-se um nico pombo,

O pombo restante, denotado por

PN +1 ,

deve ir para

alguma das casas, juntando-se ao que j se encontrava contido nela

Demonstrao.

A prova deste princpio muito fcil e decorre de fa-

zer uma simples contagem dos pombos contidos em todas as casas depois de distribudos. Com efeito, suponhamos pelo contrrio que em
cada casa no existe mais do que um pombo, ento contando todos
os pombos contidos nas

casas no teremos mais do que

contradizendo isto a hipteses de termos


nas

N +1

pombos,

pombos distribudos

casas (ver Figura 4.1).

No difcil detectar quando o princpio pode ser usado, mas a


principal diculdade para aplic-lo reside em identicar, em cada problema, quem faz papel de pombos e quem faz papel de casas.
Nas seguintes sees discutiremos vrios exemplos de diferentes
naturezas onde o

princpio da casa dos pombos

aplicado com sucesso.

4.1

Primeiros Exemplos

4.1

145

Primeiros Exemplos

Exemplo 4.2. Numa oresta crescem 1.000 jaqueiras. conhecido

que uma jaqueira no contm mais do que 600 frutos. Prove que
existem 2 jaqueiras na oresta que tm a mesma quantidade de frutos.
Soluo.

Temos 1.000 jaqueiras, representando os pombos, e 601 casas

identicadas pelos nmeros

0, 1, 2, 3, . . . , 600.

O nmero

associado

a cada casa signica que nela sero colocadas jaqueiras que tm exatamente

frutos. Como

1000 > 602 = 601 + 1,

PCP

nos garante

que existem duas jaqueiras com a mesma quantidade de frutos.

Exemplo 4.3. Em uma reunio h n pessoas. Mostre que existem

duas pessoas que conhecem exatamente o mesmo nmero de pessoas.


Soluo.

Os pombos neste caso so as

radas com os nmeros

n pessoas.

As casas so enume-

0, 1, 2, . . . , n 1, indicando estes que na mesma

sero colocadas pessoas que tm essa quantidade de conhecidos. No-

0 ou n 1 permanece
desocupada, pois a possibilidade de conhecer 0 e n 1 pessoas no
acontece simultaneamente. Logo, nas n 1 casas restantes haver

temos que uma das casas enumeradas com

uma ocupada por dois ou mais pombos, depois de serem distribudos.

Portanto, existem no mnimo duas pessoas com o mesmo nmero de


conhecidos.

Exemplo 4.4. Dados 8 nmeros inteiros mostre que existem dois

deles cuja diferena divisvel por 7.


Soluo.

Consideramos os 8 nmeros como sendo os pombos e as casas

como sendo os

7+1

nmeros o

possveis restos na diviso por 7. Como temos

PCP

8=

nos diz que existem dois nmeros dentro dos

146

O Princpio da Casa dos Pombos

8 dados que tm o mesmo resto quando divididos por 7. Finalmente,


observamos que se dois nmeros deixam o mesmo resto na diviso por
7 ento a diferena entre eles divisvel por 7.
Uma forma alternativa e muito til na qual pode-se apresentar o
princpio da casa dos pombos a seguinte:

Proposio 4.5

(PCP  Verso Alternativa)

. Se a soma de n n-

meros naturais igual S , ento existe pelo menos um deles que no


maior que S/n, assim como existe pelo menos um deles que no
menor que S/n.

Exemplo 4.6. Numa famlia formada por 5 pessoas a soma das idades
de 245 anos. Prove que podem ser selecionados 3 membros da famlia
cuja soma das idades no menor que 147.
Soluo.

Temos um total de

5
3

5!
3!2!

= 10

trios diferentes formados

por membros da famlia. Alm disso, cada pessoa aparece exatamente


em

4
2

4!
2!2!

=6

trios. Ento, denotando por

dos membros de cada trio

Tj , j = 1, 2 . . . 10,

Ej

a soma das idades

temos que

E1 + E2 + + E10 = 6 245 = 1470;


consequentemente existe algum trio

4.2

Tj

tal que

Ej

1470
10

= 147.

Uma Verso mais Geral

A seguinte verso mais geral do

PCP

bastante til na resoluo de

alguns problemas.

Proposio 4.7 (PCP  Verso Geral). Se distribumos N k + 1 pom-

bos em N casas, ento alguma das casas contm pelo menos k + 1


pombos.

Uma Verso mais Geral

4.2

147

A prova deste enunciado mais geral similar anterior. Com efeito,


suponhamos pelo contrrio que em cada casa no existe mais do que

pombos, ento contando todos os pombos contidos nas

teremos mais do que


termos

Nk + 1

Nk

casas no

pombos, contradizendo isto a hipteses de

pombos distribudos nas

Notemos que se

casas.

k = 1, esta verso mais geral coincide com a verso

mais simples.

Exemplo 4.8. Num colgio com 16 salas so distribudas canetas nas


cores preta, azul e vermelha para realizar uma prova de concurso. Se
cada sala recebe canetas da mesma cor ento prove que existem pelo
menos 6 salas que receberam canetas da mesma cor.
Soluo.

Fazendo a diviso com resto de 16 por 3 temos que

3 5 + 1.

Consideramos as 16 salas como sendo os pombos e as trs

cores, preto, azul e vermelho como sendo as casas.


colocar cada sala em uma das trs cores. Assim, o
e

16 =

Logo, podemos

PCP

com

N =3

k = 5 nos d que existe uma casa com pelo menos 6 pombos, ou seja,

existem no mnimo 6 salas que receberam canetas da mesma cor.

Exemplo 4.9. Uma equipe formada por seis alunos de Matemtica

selecionada para representar o Brasil numa olimpada internacional.


Mostre que necessariamente existem trs deles que se conhecem mutuamente, ou trs deles que no se conhecem mutuamente.
Soluo.
aluno

Resolveremos o problema com o auxlio da Figura 4.2. Cada

Aj ,

com

j = 1, 2, . . . , 6,

um hexgono regular.

representado por um dos vrtices de

Quando dois alunos se conhecem traamos o

segmento de reta que liga os vrtices correspondentes com uma linha


contnua; caso contrrio traamos este segmento com uma linha pontilhada.

Logo, usando este esquema, o problema equivale a provar

148

O Princpio da Casa dos Pombos

que sempre existe um tringulo de lados contnuos ou um tringulo de


lados pontilhados com vrtices no conjunto

A = {A1 , A2 , . . . , A6 }.

Temos 5 segmentos (pombos) incidindo no vrtice

A1 ,

cada um

deles contnuo ou pontilhado (estes dois tipos de linhas so consideradas como as casas).

Como

5 = 2 2 + 1,

pelo

PCP

temos que 3

dos 5 segmentos so contnuos ou pontilhados. Suponhamos que 3 so


contnuos (caso contrrio o argumento similar) e denotemos estes

A1 A3 , A1 A4 e A1 A6
A3 A4 , A3 A6 ou A4 A6 for
se ligam com A1 formam

por

(ver Figura 4.2).

Se algum dos segmentos

contnuo ento este segmento junto aos que


um tringulo de lados contnuos. Por outro

lado, se nenhum deles for contnuo, ento eles formam um tringulo


de lados pontilhados, completando isto a demonstrao.

A3

A2

A4

A1

A5
Figura 4.2: O tringulo

A1 A2 A5

A6
indica que os alunos

conhecem mutuamente e o tringulo


e

A6

se conhecem mutuamente

A1 A4 A6

A1 , A2

A5

indica que os alunos

no se

A1 , A4

Aplicaes na Teoria dos Nmeros

4.3

4.3

149

Aplicaes na Teoria dos Nmeros

Nesta seo apresentamos alguns exemplos de aplicaes do

PCP

na

Teoria dos Nmeros. A primeira delas :

Exemplo 4.10. Se n e m so nmeros naturais, ento o conjunto

A = {m + 1, m + 2, . . . , m + n} possui algum divisor de n.

Soluo.

Temos

n nmeros diferentes no conjunto acima.

Vamos utili-

zar o mtodo de reduo ao absurdo. Se no existisse nenhum mltiplo


de

n,

quando dividssemos os nmeros do conjunto

por

n,

os res-

B = {1, 2, . . . , n 1}, que possui n 1


elementos. Logo, devem existir dois nmeros m + i e m + j , com
1 i < j n tais que o resto da diviso de m + i por n o mesmo
que o resto da diviso de m + j por n. Logo, m + j (m + i) um
mltiplo de n, o que implica que n > j i 1 mltiplo de n menor
que n (absurdo!). Logo, deve existir algum mltiplo de n no conjunto
A.

tos pertenceriam ao conjunto

Como consequncia desse exemplo, podemos resolver o prximo


problema.

Exemplo 4.11. Demonstrar que todo inteiro tem um mltiplo cuja


representao decimal comea com o bloco de dgitos 1234567890.
Soluo.

m e n so inteiros positivos, pelo exemplo anterior um


dos nmero m + 1, m + 2, . . . , m + n mltiplo de n. Assim, dado n
n+1
um inteiro qualquer, escolhe-se m = 123456789010
. Deste modo,
todos os inteiros m + 1, m + 2, . . . , m + n comeam com 1234567890 e
algum deles mltiplo de n.
Se

150

O Princpio da Casa dos Pombos

Exemplo 4.12. Dado um nmero inteiro positivo n, mostre que existe


um mltiplo de n que se escreve com os algarismos 0 e 1 apenas. (Por
exemplo, se n = 3, temos 111 ou 1.101 etc.)
Soluo.

Consideramos os

n+1

nmeros

1, 11, 111, 1111, . . . , 111


1}
| {z

(4.1)

n+1vezes

como sendo os pombos e

casas enumeradas com os nmeros

0, 1, 2, 3, . . . , n 1,
ou seja, com os possveis restos na diviso por

n.

Similarmente ao

exemplo anterior existem dois nmeros na lista (4.1) que deixam o


mesmo resto na diviso por
o menor mltiplo de

n.

e, portanto, a diferena entre o maior e

Obviamente a diferena entre dois nmeros

quaisquer da lista (4.1) resulta em um nmero formado apenas pelos


algarismos 0 e 1.

Exemplo 4.13. Prove que entre n + 1 elementos escolhidos no conjunto {1,2,3, . . . , 2n} existem dois que so primos relativos.

Soluo.

A escolha das casas e dos pombos neste exemplo no to b-

n + 1 nmeros escolhidos do conjunto


escolhidas como sendo os n conjuntos:

via. Os pombos representam os

{1, 2, . . . , 2n}

e as casas so

Cj = {2j 1, 2j}, 1 j n.
Logo, pelo
tos

PCP, quando distribumos os n + 1 nmeros nos n conjun-

Cj , 1 j n, dois deles caro juntos em algum conjunto Cj ,

seja, estes nmeros sero consecutivos e portanto primos entre si.

ou

Aplicaes Geomtricas

4.4

151

Finalizaremos esta seo com uma outra prova do teorema de


Bachet-Bzout, (veja o Teorema 3.23).

Exemplo 4.14. Seja d = (a, b) o mdc entre os nmeros naturais a e


b. Ento, existem x e y nmeros inteiros tais que
ax + by = d.

Soluo.
b

Denotando por

m = a/d

n = b/d,

podemos supor que

so primos entre si. Realmente, se podemos escrever

mx + ny = 1
ento, substituindo os valores de

na equao acima, temos que

ax + by = d.
Se (a, b) = 1, considere a sequncia A = {a, 2a, . . . , ba}. Armamos
que existe algum nmero no conjunto A que deixa resto 1 quando
dividido por b. De fato, se isso no ocorresse, teramos b nmeros em
A deixando no mximo b 1 restos diferentes quando divididos por
b. Logo, pelo PCP, dois deles, digamos ia e ja com b > j > i 1,
devem deixar o mesmo resto quando divididos por b. assim, (j i)a
divisvel por b. Como estamos supondo que (a, b) = 1, temos que b
deve dividir j i > 0. Como b > j i, temos um absurdo.
Assim, algum dos nmeros em a deixa resto 1 quando divididos
por b. Digamos que esse nmero seja ax. Logo, ax 1 mltiplo de
b, onde ax 1 = by , o que encerra nossa prova.

4.4

Aplicaes Geomtricas

Na geometria tambm encontramos belas aplicaes do


os problemas a seguir para constatar isto.

PCP. Vejamos

152

O Princpio da Casa dos Pombos

Exemplo 4.15. Mostre que se tomamos cinco pontos quaisquer sobre


um quadrado de lado 1, ento pelo menos dois deles no distam mais

que 2/2.

Soluo.
1/2,

Vamos dividir o quadrado em quatro quadradinhos de lado

como mostra a gura. Logo, pelo

PCP

pelo menos dois deles de-

vem estar no mesmo quadradinho, uma vez que temos 4 quadradinhos


e 5 pontos. Logo, como a maior distncia num quadrado a diagonal,
o Teorema de Pitgoras nos garante que a distncia desses dois pontos
no mximo

2/2,

como queramos mostrar.

Exemplo 4.16. Na regio delimitada por um tringulo equiltero de

lado 4 so marcados 10 pontos no interior deste. Prove que existe ao


menos um par destes pontos cuja distncia entre eles no maior que

3.
Soluo.

Dividimos o tringulo equiltero de lado 4 em 16 tringulos

equilteros menores de lado 1, conforme a Figura 4.3.


Agora pintamos os tringulos nas cores branco e cinza de maneira
que dois tringulos vizinhos, isto , com um lado comum, so pintados
de cores diferentes. Se tivssemos dois pontos no mesmo tringulo a
distncia mxima possvel entre eles seria 1 e o problema estaria resolvido. Se tivssemos pontos em tringulos vizinhos, a maior distncia
possvel entre eles seria

e tambm isto resolveria o problema. Se

no tivssemos nenhum dos casos anteriores, no seria difcil ver que

4.5

Miscelnea

153

Figura 4.3: O tringulo

DBE

equiltero de lado

os 10 pontos deveriam estar situados sobre os 10 tringulos brancos,


contendo cada tringulo exatamente um ponto. Dividindo o tringulo

3/2 pelo PCP temos que


pelo menos dois dos 6 pontos contidos em DBE esto num destes 4

tringulos, logo a distncia entre eles no maior que 3/2 <


3. Com

DBE

em 4 tringulos congruentes de lado

isto terminamos nossa prova.

4.5

Miscelnea

Os problemas que apresentamos a seguir usam o PCP combinado com


outras idias que so muito empregadas nas suas solues.

Exemplo 4.17. Em cada quadradinho de um tabuleiro 3 3 colocado

um dos nmeros: -1, 0 ou 1. Prove que entre todas as somas das


linhas, colunas e diagonais do tabuleiro h duas que so iguais. Por
exemplo, no tabuleiro abaixo a soma da segunda linha 2, que coincide
com a soma da terceira coluna.

154

Soluo.

O Princpio da Casa dos Pombos

-1

-1

-1

S = a1 + a2 + a3 , onde cada a1 , a2 e a3 podem tomar


valores: 1, 0 e 1. Ento, temos 7 valores possveis para S (casas),
que so: 3, 2, 1, 0, 1, 2, 3.
O tabuleiro 33 tem 3 linhas, 3 colunas e 2 diagonais, portanto, ao
Seja

somarmos os elementos de cada uma das linhas, colunas e diagonais,


obteremos 8 nmeros (pombos).

Como existem somente

possveis para estes nmeros, pelo

PCP

valores

pelo menos dois deles devem

ser iguais.

Exemplo 4.18. Dado qualquer conjunto A formado por 10 nmeros

naturais escolhidos entre 1 e 99, inclusos, demonstre que existem dois


subconjuntos disjuntos e no vazios de A tal que a soma dos seus respectivos elementos igual.
Soluo:

conhecido que

vazios diferentes.

tem

210 1 = 1.023

subconjuntos no-

A soma dos elementos de cada um deles d uma

quantidade menor do que 1.000, pois o subconjunto com no mximo


10 elementos de maior soma possvel o formado por
e nesse caso

90 + 91 + + 99 = 945.

90, 91, . . . , 99,

Agora consideramos os pombos

como sendo os 1.023 subconjuntos distintos de

e as casas como

sendo as somas possveis dos elementos de cada um dos conjuntos.


Logo, como o nmero de conjuntos maior que o nmero de somas
possveis, devem existir dois conjuntos
a soma dos elementos de

de

A,

de tal modo que

igual soma dos elementos de

C.

Se

4.5

e
e

Miscelnea

155

C so disjuntos, acabou a prova. Se no, considere D = B B C


E = C B C . Logo, os conjuntos D e E so disjuntos e a soma

dos seus elementos a mesma, pois retiramos de ambos a mesma


quantidade.

Exemplo 4.19. Qual o maior nmero de quadradinhos de um ta-

buleiro de 8 8 que podem ser pintados de preto, de forma tal que


qualquer arranjo de trs quadradinhos, como mostra a Figura 4.4, tenha pelo menos um dos quadradinhos no pintado de preto?

Figura 4.4: Tridomins

Soluo.

Primeiramente, pintamos o tabuleiro de

88 como um tabu-

leiro de jogar xadrez, ou seja, 32 quadradinhos pintados de branco e


32 quadradinhos pintados de preto (ver Figura 4.5).

Figura 4.5: Tabuleiro de xadrez

156

O Princpio da Casa dos Pombos

Notemos que uma vez pintado o tabuleiro desta forma satisfeita


a exigncia do problema, pois nunca temos 2 quadradinhos vizinhos
(quadradinhos com um lado comum) pintados de preto.
Mostraremos agora que se pintamos 33 quadradinhos de preto ento a condio exigida no problema falha.
tabuleiro em 16 quadrados de

22

(casas) e pintamos 33 quadra-

dinhos de preto (pombos); ento, como


geral do

PCP

De fato, se dividimos o

um dos 16 quadrados de

33 = 16 2 + 1, pela verso
2 2 contm 3 quadradinhos

pintados de preto. Portanto, este ltimo contm um arranjo como na


Figura 4.4 completamente pintado de preto.

Resumindo, o nmero mximo de quadradinhos que podemos pintar de preto 32.

Exemplo 4.20. Dados sete nmeros reais arbitrrios, demonstre que


existem dois deles, digamos x e y , tais que
0
Soluo.

1
xy

1 + xy
3

Primeiramente observamos que a expresso

sar na frmula

tan( ) =
Sejam

x1 , x2 , , x7

tan tan
.
1 + tan tan

xy
nos faz pen1+xy

(4.2)

os sete nmeros selecionados arbitrariamente.

Lembramos que a funo tangente uma bijeo entre o intervalo

( 2 , 2 ) e os nmeros reais R, logo para cada xi , 1 i 7, existe um


i ( 2 , 2 ) tal que tan(i ) = xi . Dividimos o intervalo ( 2 , 2 ) em
seis subintervalos de comprimento

Pelo

PCP

dois dos nmeros

i1

Denotemos os mesmos por

, como mostra o desenho a seguir.


6

pertencem ao mesmo subintervalo.

i2

e suponhamos, sem perda de

Exerccios

4.6

157

i1
2
generalidade, que

i2

i1 i2 .

Ento vale

0 i2 i1

.
6

Usando o fato de que a tangente uma funo crescente e a frmula


(4.2) temos que

tan(0) tan(i2 i1 ) tan( ).


6
Equivalentemente,

4.6

xi 2 xi 1
1
.
1 + xi 2 xi 1
3

Exerccios

1. Seja

um conjunto formado por cinco pontos de coordenadas

inteiras no plano. Prove que o ponto mdio de algum dos segmentos com extremos em

tem tambm coordenadas inteiras.

2. O conjunto dos dgitos 1, 2, ..., 9 dividido em trs grupos.


Prove que o produto dos nmeros de algum dos grupos deve ser
maior que 71.
3. Prove que se

mpar ento para qualquer bijeo

p : IN IN

158

O Princpio da Casa dos Pombos

IN = {1, 2, . . . , N } o produto P (p) = (1 p(1))(2


p(2)) (N p(N )) necessariamente par.

do conjunto

(Dica: O produto de vrios fatores par se, e somente se, um dos


fatores par.)
4. Dado um conjunto de 25 pontos no plano tais que entre quaisquer
3 deles existe um par com distncia menor que 1.

Prove que

existe um crculo de raio 1 que contm pelo menos 13 dos 25


pontos dados.
5. Prove que entre quaisquer 5 pontos escolhidos dentro de um
tringulo equiltero de lado 1 sempre existe um par deles cuja
distncia no maior que 0,5.
6. Marquemos todos os centros dos 64 quadradinhos de um tabuleiro de xadrez de

8 8.

possvel cortar o tabuleiro com 13

linhas retas que no passem pelos pontos marcados e de forma


tal que cada pedao de recorte do tabuleiro tenha no mximo
um ponto marcado?
7. Prove que existem duas potncias de 3 cuja diferena divisvel
por 1.997.
8. So escolhidos 6 nmeros quaisquer pertencentes ao conjunto

A = {1, 2, 3, . . . , 10}.
Prove que existem dois desses seis nmeros cuja soma mpar.
9. Seja

um nmero real arbitrrio. Prove que entre os nmeros

x, 2x, 3x, . . . , 101x

4.6

Exerccios

159

existe um tal que sua diferena com certo nmero inteiro menor
0,011.
10. Mostre que entre nove nmeros que no possuem divisores primos maiores que cinco, existem dois cujo produto um quadrado.
11. Um disco fechado de raio um contm sete pontos, cujas distncias entre quaisquer dois deles maior ou igual a um. Prove que
o centro do disco um destes pontos.
12. Na regio delimitada por um retngulo de largura quatro e altura
trs so marcados seis pontos.

Prove que existe ao menos um

par destes pontos cuja distncia entre eles no maior que


13. Seja

5.

um nmero irracional. Prove que existem innitos nme-

ros racionais

r = p/q

tais que

|a r| < 1/q 2 .

14. Suponha que cada ponto do reticulado plano pintado de vermelho


ou azul. Mostre que existe algum retngulo com vrtices no reticulado
e todos da mesma cor.
15. Um certo livreiro vende pelo menos um livro por dia. Sabendo que o
livreiro vendeu 463 livros durante 305 dias consecutivos, mostre que
em algum perodo de dias consecutivos o livreiro vendeu exatamente
144 livros.

Referncias Bibliogrcas
[1] AIGNER, M. e ZIEGLER, G. (2002).

no Livro.

Edgard Blcher.

[2] GARCIA, A. e LEQUAIN, I. (2003).

gebra.

As Provas esto
Elementos de l-

Projeto Euclides, IMPA.

[3] LIMA, E. L.; CARVALHO, P. C. P.; WAGNER, E. e


MORGADO, A.C. (2004).

dio. Volume 1.

A Matemtica do Ensino M-

Sociedade Brasileira de Matemtica.

[4] LIMA, E.L.; CARVALHO, P. C. P.; WAGNER, E. e


MORGADO, A.C. (2004).

dio. Volume 2.
[5] LIMA,E.L.;

A Matemtica do Ensino M-

Sociedade Brasileira de Matemtica.

CARVALHO,P.

MORGADO,A.C. (2004).

dio. Volume 3.

C.

P.;

WAGNER,E.

A Matemtica do Ensino M-

Sociedade Brasileira de Matemtica.

[6] LIMA, E.L.; CARVALHO, P. C. P.; WAGNER,E. e


MORGADO, A.C. (2001).

Temas e Problemas.

Socie-

dade Brasileira de Matemtica.


[7] LIMA, E.L. (2001).

lgebra Linear. Sociedade Brasileira

de Matemtica.

285

286

REFERNCIAS BIBLIOGRFICAS
[8] MORAIS FILHO, D. C. (2007).

tica.

Um Convite Matem-

EDUFCG.

[9] MORGADO, A.;

CARVALHO, J.;

Anlise Combinatria e Pro-

FERNANDEZ, P. (1991).

babilidade .

Sociedade Brasileira de Matemtica.

[10] RIBENBOIM, P. (2001).

Recordes.

Nmeros Primos: Mistrios e

Sociedade Brasileira de Matemtica.

[11] SANTOS, J. P. O. (1993)

meros.

CARVALHO, P.;

Introduo Teoria dos N-

IMPA.

[12] SANTOS, J. P. O.; MELLO, M. P. e MURARI, I. T.


C. (2006).

Introduo Anlise Combinatria.

Editora

Unicamp.
[13] SOARES, M. G. (2005).

plexa.

Clculo em uma Varivel Com-

Sociedade Brasileira de Matemtica.

Mestrado Profissional

em Matemtica em Rede Nacional

Iniciao Matemtica

Autores:
Krerley Oliveira

Adn J. Corcho

Unidade III:
Captulos V e VI

160

5
Contagem
Toda vez que puder, onte.
Fran is Galton

Neste captulo discutiremos problemas envolvendo a contagem de


elementos de um conjunto nito dado. Por exemplo, responderemos
perguntas do tipo: de quantos modos podemos distribuir 32 selees
nacionais de futebol em seis grupos de quatro times cada?
Para solucionar questes como esta, utilizaremos como ferramentas
bsicas os princpios aditivo e multiplicativo da contagem. Veremos
tambm que o uso simultneo destes princpios ser muito til para
resolver problemas com certos nveis de complexidade. Alm disso,
sero abordados os conceitos de permutaes, arranjos e combinaes,
sendo estes de muita importncia por serem os alicerces de um ramo
da matemtica denominado combinatria.
Antes de prosseguirmos daremos algumas denies e notaes que
sero teis ao longo de todo o captulo. Dado um conjunto A denotamos por |A| a quantidade de elementos que este possui. O produto
cartesiano de n conjuntos A1 , A2 , . . . , An1 e An o conjunto denido
161

162

Contagem

por


A1 A2 An := (a1 , a2 , . . . , an ); ai Ai , i = 1, 2, . . . , n ,

onde cada elemento (a1 , a2 , . . . , an ) chamado de n-upla ordenada.


Denotaremos o conjunto vazio com o smbolo . O leitor que deseja
rever os conceitos bsicos da teoria de conjuntos, pode ach-los muito
bem expostos em [3].
5.1

Princpio Aditivo da Contagem

O princpio aditivo
nitos que no tm
unio exatamente
seja, se A1 e A2 so

da contagem garante que dados dois conjuntos


elemento em comum, o nmero de elementos da
a soma do nmero de elementos de cada um, ou
disjuntos (isto , A1 A2 = ), ento
|A1 A2 | = |A1 | + |A2 |.

Apesar de sua simplicidade, muitos problemas podem ser resolvidos utilizando esse simples princpio. A seguir enunciamos uma extenso deste princpio para um nmero nito qualquer de conjuntos.

Princpio Aditivo da contagem: Dados os conjuntos nitos A1 ,


A2 , . . . , An dois a dois disjuntos (isto , Ai Aj = , i 6= j ),

temos que

|A1 A2 An | = |A1 | + |A2 | + + |An |.

Exemplo 5.1. Em Macei entraram em cartaz 4 lmes distintos e


2 peas de teatro. Se Pedro Vtor s tem dinheiro para assistir a um

lme ou a uma pea de teatro, diga quantos so os possveis programas


de Pedro Vtor.

5.1

Princpio Aditivo da Contagem

163

Soluo. Denotemos por f1 , f2 , f3 e f4 os quatro lmes que esto em


cartaz e por t1 e t2 as duas peas de teatro. Agora, representemos pelo
par (i, j), com 0 i 4 e 0 j 2, o programa que consiste em assistir ao lme fi e pea tj (caso i = 0 ou j = 0 isso signica que no
ser assistido a nenhum lme ou a nenhuma pea, respectivamente).
Pelas limitaes econmicas do Pedro Vtor temos que ele s pode
escolher um programa dentro dos seguintes conjuntos disjuntos:




A1 = (1, 0), (2, 0), (3, 0), (4, 0) e A2 = (0, 1), (0, 2) .

Logo, no total so |A1 A2 | = |A1 | + |A2 | = 6 programas distintos,


entre os quais Pedro Vtor ter que escolher um.

Exemplo 5.2. Numa reunio havia um certo nmero de pessoas e

todos os presentes apertaram as mos entre si. Sabendo-se que ao todo


foram feitos 66 cumprimentos, calcule o nmero de pessoas presentes
reunio.

Soluo. Vamos enumerar as pessoas com os nmeros do conjunto


P = {1, 2, . . . , n}. A cada aperto de mo associaremos um par (i, j),
signicando que a pessoa i apertou a mo da pessoa j . Assim, os
apertos de mo envolvendo a pessoa 1 foram:
A1 = {(1, 2), (1, 3), . . . , (1, n)}.

Do mesmo modo, denimos os apertos de mo envolvendo a pessoa 2


que no envolvem a pessoa 1, como:
A2 = {(2, 3), (2, 4), . . . , (2, n)}.

Note que o aperto (2, 1) o mesmo que o aperto (1, 2), j que se 1
aperta a mo de 2, ento 2 aperta a mo de 1. Analogamente,
Ai = {(i, i + 1), (i, i + 2), . . . , (i, n)}, para 1 i n.

164

Contagem

Note que Ai Aj = para i 6= j . Observe tambm que todos os


apertos aparecem em um dos conjuntos Ai . Assim, A1 An
contm todos os apertos de mo. Logo, pelo princpio aditivo:
|A1 A2 An | = |A1 | + |A2 | + . . . |An |

= (n 1) + (n 2) + + 2 + 1

(n 1)n
= 66.
2
Resolvendo em n, temos que n = 12.
=

Vimos que o princpio aditivo nos fornece o nmero de elementos


de qualquer unio de conjuntos dois a dois disjuntos. Discutiremos
agora uma extenso do princpio para qualquer unio de conjuntos,
no necessariamente dois a dois disjuntos.

Proposio 5.3. Sejam A1 e A2 dois conjuntos nitos quaisquer.

Ento,

|A1 A2 | = |A1 | + |A2 | |A1 A2 |.

Demonstrao. Observe que

A1 A2 = (A1 A2 ) A2

onde a unio dois a dois disjunta. Pelo princpio aditivo, temos que
|A1 A2 | = |A1 A2 | + |A2 |.

(5.1)

Analogamente, aplicando novamente este princpio, temos que


|A1 | = |A1 A2 | + |A1 A2 |;

(5.2)

A proposio segue imediatamente combinando as igualdades (5.1) e


(5.2).

5.1

Princpio Aditivo da Contagem

165

Para chegar a uma expresso anloga do princpio aditivo, vamos


fazer mais um caso, considerando agora trs conjuntos.

Corolrio 5.4. Sejam A1 , A2 e A3 trs conjuntos nitos quaisquer.

Ento,

|A1 A2 A3 | =|A1 | + |A2 | + |A3 |


|A1 A2 | + |A1 A3 | + |A2 A3 |
+ |A1 A2 A3 |.

Demonstrao. Pela Proposio 5.3 temos que,


|A1 (A2 A3 )| = |A1 | + |A2 A3 | |A1 (A2 A3 )|,

de onde,
|A1 A2 A3 | = |A1 | + |A2 A3 | |(A1 A2 ) (A1 A3 )|.

Novamente, pela Proposio 5.3 temos que,


|A1 A2 A3 | = |A1 | + |A2 | + |A3 | |A2 A3 | |(A1 A2 ) (A1 A3 )|.

Aplicando mais uma vez a Proposio 5.3 temos que,


|(A1 A2 ) (A1 A3 )| = |A1 A2 | + |A1 A3 | |(A1 A2 ) (A1 A3 ).

Combinando as duas ltimas igualdades obtemos


|A1 A2 A3 | =|A1 | + |A2 | + |A3 |


|A1 A2 | + |A1 A3 | + |A2 A3 |
+ |A1 A2 A3 | ,

como desejvamos.

166

Contagem

Para facilitar nossa escrita, vamos denotar por A1 A2 . . . Ak o conjunto A1 A2 Ak . Assim, outra forma de enunciar o Corolrio
5.4 a seguinte:


3
3
[
X


=
|Ai |
A

i


i=1

i=1

1i1 <i2 3

|Ai1 Ai2 | +

1i1 <i2 <i3 3

|Ai1 Ai2 Ai3 |.

De forma geral, dados os conjuntos nitos A1 , A2 , . . . , An , as expresses anteriores nos levam a denir os nmeros:
S1 =

n
X
i=1

S2 =

..
.
Sk =

..
.

|Ai |

1i1 <i2 n

|Ai1 Ai2 |,

1i1 <i2 <<ik n

|Ai1 Ai2 . . . Aik |,

Sn = |A1 A2 . . . An |.

Assim, a verso mais geral do princpio aditivo, tambm conhecida


como princpio de incluso e excluso , :

Princpio Aditivo - Verso Geral: Sejam A1 , A2 . . . , An


conjuntos nitos quaisquer. Ento,



n
[



Ai = S1 S2 + S3 S4 + + (1)n1 Sn .


i=1

No iremos provar essa verso, mas o leitor pode (e deve!) mostr-la


como exerccio, repetindo os argumentos anteriores.

5.1

Princpio Aditivo da Contagem

167

Exemplo 5.5. No Colgio Fantstico foram entrevistados 78 estudan-

tes. Destes, 32 estavam fazendo um curso de francs; 40 um curso de


fsica; 30 um curso de matemtica; 23 um curso de histria; 19 francs
e fsica; 13 francs e matemtica; 15 fsica e matemtica; 2 francs e
histria; 15 fsica e histria; 14 matemtica e histria; 8 francs, fsica e matemtica; 8 francs, fsica e histria; 2 francs, matemtica
e histria; 6 fsica, matemtica e histria e 2 estavam fazendo todos
os quatro cursos. Quantos estudantes estavam fazendo pelo menos 1
curso nas 4 reas mencionadas?
Soluo. Denotemos por A1 , A2 , A3 , e A4 os conjuntos dos estudantes que fazem francs, fsica, matemtica e histria, respectivamente.
Observemos que as igualdades
|A1 | = 32,
|A2 | = 40,
|A3 | = 30,
|A4 | = 23,

nos do que S1 =

4
X
i=1

|Ai | = 125; as igualdades


|A1 A2 | = 19,
|A1 A3 | = 13,
|A1 A4 | = 2,

|A2 A3 | = 15,
|A2 A4 | = 15,
|A3 A4 | = 14,

168

nos do que S2 =

1i1 <i2 4

Contagem

|Ai1 Ai2 | = 78; as igualdades


|A1 A2 A3 | = 8,
|A1 A2 A4 | = 8,
|A1 A3 A4 | = 2,
|A2 A3 A4 | = 6,

nos do que S3 =
|A1 A2 A3 A4 | = 2.

1i1 <i2 <i3 4

|Ai1 Ai2 Ai3 | = 24; assim como que S4 =



4
[



Segue-se ento, do princpio aditivo, que Ai = 125 78 + 24


i=1
2 = 69.

Denio 5.6. Denimos o complementar do conjunto A em relao

ao conjunto U como sendo um subconjunto de U dado por




Ac = x U; x
/A .

U
A

Figura 5.1: A rea branca corresponde a Ac e o conjunto U representado


por todo o retngulo

5.1

Princpio Aditivo da Contagem

169

Neste caso fcil vericar que os conjuntos A e Ac so disjuntos e


que U = A Ac . Segue-se do princpio aditivo que |U| = |A| + |Ac |;
portanto,
|Ac | = |U| |A|.

Analogamente, dados dois conjuntos A1 U e A2 U , temos que


A1 A2 e (A1 A2 )c so disjuntos e, alis, U = (A1 A2 ) (A1 A2 )c .
Novamente, pelo princpio aditivo, vale que
|U| = |A1 A2 | + |(A1 A2 )c |;

e consequentemente temos que


|(A1 A2 )c | = |U| (|A1 | + |A2 |) + |A1 A2 |.

Similarmente, dados trs conjuntos A1 U, A2 U e A3 U


podemos demonstrar que
|(A1 A2 A3 )c | = |U| (|A1 | + |A2 | + |A3 |)

+ (|A1 A2 | + |A1 A3 | + |A2 A3 |)


|A1 A2 A3 |.

Ento, usando a notao S0 = |U|, temos a seguinte proposio:

Proposio 5.7. Para toda famlia de subconjuntos Ai U , i =


1, 2, . . . , n, vale a relao:

!c
n
[




Ai = S0 S1 S2 + S3 S4 + (1)n1 Sn



i=1

= S0 S1 + S2 S3 + S4 + (1)n Sn ,

ou resumidamente,

n
!c
n
[

X


c
c c
Ai = |A1 A2 An | =
(1)j Sj .



i=1

j=0

170

Contagem

Observao 5.8. Observemos que na ltima relao da proposio

usamos a conhecida Lei de DeMorgan: o complementar da unio de


uma famlia nita de conjuntos, em relao a um conjunto U , a
interseco dos complementares de cada um deles.
5.2

Princpio Multiplicativo de Contagem

Comeamos esta seo discutindo um problema relacionado com o


apaixonante jogo de xadrez. Ele consiste no seguinte: queremos saber
de quantas maneiras diferentes podemos colocar duas torres num tabuleiro de xadrez de forma tal que nenhuma ataque a outra. Uma situao como a que procuramos mostrada na Figura 5.2, pois lembramos
que torres s se movimentam na direo horizontal ou na direo vertical do tabuleiro. Antes de prosseguir deixamos claro o seguinte: se na
Figura 5.2 trocamos a posio da torre a com a torre b consideraremos
isto como uma situao diferente.

Figura 5.2: Torres que no se atacam


Notemos o seguinte: uma vez que coloquemos uma das torres numa

5.2

Princpio Multiplicativo de Contagem

171

casa do tabuleiro no podemos colocar a segunda torre na mesma


linha ou coluna em que esta se encontra, pois ela seria ameaada.
Como cada linha e cada coluna contm 8 casas do tabuleiro, sendo
uma delas comum a ambas, ento temos 15 posies proibidas para
colocar a segunda torre, ou seja, ela s pode ser colocada em 6415 =
49 posies diferentes. Resumindo, por cada uma das 64 possveis
posies para a torre a temos 49 possibilidades diferentes para colocar
a torre b, totalizando 6449 = 3.136 formas diferentes de colocar ambas
as torres no tabuleiro sem que elas se ataquem.
O exemplo acima traz a essncia do que chamado princpio multiplicativo da contagem : se um evento A1 pode ocorrer de m maneiras
distintas e, se para cada uma dessas m maneiras possveis de A1 ocorrer, um outro evento A2 pode ocorrer de n maneiras distintas, ento o
nmero de maneiras de ocorrerem sucessivamente os eventos A1 e A2
m n.
Na linguagem matemtica: relembramos que dados dois conjuntos
A1 e A2 , podemos construir um par ordenado (a1 , a2 ) tomando um
elemento a1 A1 , denominado o primeiro elemento do par, e um
elemento a2 A2 , denominado o segundo elemento do par. O conjunto
A1 A2 constituido por todos os pares ordenados construdos dessa
forma. Assim sendo, a verso mais simples do princpio multiplicativo
nos garante que
|A1 A2 | = |A1 | |A2 |.

Uma extenso deste princpio para um nmero nito qualquer de


conjuntos a seguinte:

princpio multiplicativo da contagem: Dados os conjuntos

172

Contagem

nitos A1 , A2 , . . . , An temos que


|A1 A2 An | = |A1 | |A2 | |An |.

Note que neste princpio, no necessria nenhuma hiptese adicional sobre os conjuntos Ai . Vamos agora dar alguns exemplos de
como aplicar esse princpio.

Exemplo 5.9. Em Macei entraram em cartaz 4 lmes distintos e


2 peas de teatro. Se agora o Pedro Vtor tem dinheiro para assistir

exatamente a um lme e a uma pea de teatro, diga quantos so os


possveis programas que Pedro Vtor pode fazer.
Soluo. Denotemos por f1 , f2 , f3 e f4 os quatro lmes que esto em
cartaz e por t1 e t2 as duas peas de teatro. Denamos os conjuntos
A1 = {f1 , f2 , f3 , f4 } e A2 = {t1 , t2 }.

Neste caso, as condies econmicas do Pedro Vtor permitem que


ele escolha um elemento do conjunto A1 e outro elemento do conjunto
A2 . Este tipo de escolha representa-se pelo conjunto


A1 A2 = (fi , tj ); 1 i 4 e 1 j 2 ,

onde cada par (fi , tj ) representa o programa que consiste em assistir


ao lme fi e pea tj . Logo, no total so |A1 A2 | = |A1 | |A2 | = 8
programas distintos.

Exemplo 5.10. Se numa loja de doces existem 9 tipos distintos de

balas e 5 tipos distintos de chiclete, diga quantas escolhas podemos


fazer para comprar somente uma bala e um chiclete.

5.2

Princpio Multiplicativo de Contagem

173

Soluo. Denotemos por b1 , b2 , b3 , b4 , b5 , b6 , b7 , b8 e b9 os nove tipos


distintos de balas e por c1 , c2 , c3 , c4 e c5 os cinco tipos distintos de
chicletes. Denamos os conjuntos
B = {b1 , b2 , b3 , b4 , b5 , b6 , b7 , b8 , b9 } e C = {c1 , c2 , c3 , c4 , c5 }.

Como precisamos comprar simultaneamente um elemento do conjunto


B e um elemento do conjunto C , ento o conjunto B C me d o
conjunto de todas as escolhas possveis. Logo, o nmero de escolhas
possveis para comprar simultaneamente um tipo de bala e um tipo
de chiclete |B C| = 9 5 = 45.

Exemplo 5.11. De quantas maneiras 2 pessoas podem estacionar seus

carros numa garagem com 10 vagas?

Soluo. Observando que a primeira pessoa pode estacionar seu carro


de 10 formas distintas e que a segunda pessoa pode estacionar seu
carro de 9 formas distintas, temos pelo princpio multiplicativo que
existem 9 10 = 90 formas possveis nas quais duas pessoas podem
estacionar seus carros numa garagem com 10 vagas.

Exemplo 5.12. Dado o nmero 720, diga


(a) quantos divisores inteiros e positivos ele possui;
(b) entre seus divisores inteiros e positivos, quantos so pares;
(c) entre seus divisores inteiros e positivos, quantos so mpares;
(d) dos divisores acima, quantos so quadrados perfeitos.

174

Contagem

Soluo. Pelo teorema fundamental da aritmtica, todo nmero inteiro positivo primo ou produto de primos. Observe que a decomposio de 720 em fatores primos vem dada por:
720 = 24 32 51 .

(5.3)

Agora denamos os seguintes conjuntos:


A ={todos os divisores de 720 que so da forma 2k , onde k Z+ },

B ={todos os divisores de 720 que so da forma 3m , onde m Z+ },


C ={todos os divisores de 720 que so da forma 5n , onde n Z+ }.

Observemos que 0 k 4, pois se k > 4 ento pelo menos a potncia


25 deveria estar presente em (5.3); como isto no acontece segue-se
que 0 k 4, de modo que


A = 20 , 21 , 22 , 23 , 24 ,

seguindo o mesmo raciocnio, podemos demonstrar que 0 m 2 e


que 0 n 1. Assim,




B = 30 , 31 , 32
e C = 50 , 51 .

(a) O conjunto de todos os possveis divisores de 720 pode ser identicado com o conjunto AB C . De onde o nmero de divisores
inteiros e positivos de 720 |AB C|. Porm, o princpio multiplicativo nos garante que |AB C| = |A||B||C|. Portanto,
o nmero de divisores inteiros e positivos de 720 5 3 2 = 30,
pois |A| = 5, |B| = 3 e |C| = 2.
(b) Para obter o conjunto de todos os divisores pares de 720 devemos remover o elemento 20 do conjunto A. Assim, o conjunto de

5.2

Princpio Multiplicativo de Contagem

175

todos os divisores pares e positivos de 720 vem dado pelo conjunto A {20 } B C . O princpio multiplicativo nos garante




que A {20 } B C = A {20 } |B| |C|. Portanto, o
nmero de divisores pares e positivos de 720 4 3 2 = 24,


pois A {20 } = 4, |B| = 3 e |C| = 2.

(c) Para obter o conjunto de todos os divisores mpares de 720 devemos remover os elementos 21 , 22 , 23 e 24 do conjunto A. Assim,
o conjunto de todos os divisores mpares e positivos de 720 vem
dado pelo conjunto

A {21 , 22 , 23 , 24 } B C.

O princpio multiplicativo nos garante que





A {21 , 22 , 23 , 24 } B C = A {21 , 22 , 23 , 24 } |B| |C|.

Portanto, o nmero de divisores mpares e positivos de 720




1 3 2 = 6; pois A {21 , 22 , 23 , 24 } = 1, |B| = 3 e |C| = 2.

(d) Para obter o conjunto de todos os divisores de 720 que so quadrados perfeitos devemos car com as potncias pares nos conjuntos A, B e C , respectivamente. Portanto, devemos remover
os elementos 21 , 23 do conjunto A. Tambm devemos remover o
elemento 31 do conjunto B . Finalmente do conjunto C devemos
remover o elemento 51 . Logo, o conjunto de todos os divisores
quadrados perfeitos e positivos de 720 vem dado pelo conjunto



D := A {21 , 23 } B {31 } C {51 } .

O princpio multiplicativo nos garante que





D = A {21 , 23 } B {31 } C {51 } .

176

Contagem

Portanto, o nmero de divisores quadrados perfeitos e positivos






de 720 3 2 1 = 6; pois A {21 , 23 } = 3, B {31 } = 2


e C {31 } = 1. Observe que {1, 4, 9, 16, 36, 144} o conjunto
dos divisores de 720 que so quadrados perfeitos.

Exemplo 5.13. Se um nmero natural n se fatora como


n = pk11 pk22 pkr r ,

(5.4)

onde os pi so nmeros primos distintos e cada ki Z+ , ento o


nmero de divisores positivos de n, denotado por d(n)
d(n) = (k1 + 1)(k2 + 1) . . . (kr + 1).

Soluo. Dena o conjunto


+
1
A1 ={todos os divisores de n que so da forma pm
1 , onde m Z },

e em geral, dena
+
i
Ai ={ todos os divisores de n que so da forma pm
i , onde t Z }.

Observemos que mi pi , pois se mi > pi , ento pelo menos a potncia


pki i +1 deveria estar presente em (5.4);como isto no acontece segue-se
que mi pi , de modo que


Ai = p0i , p1i , p2i , . . . , pki i , para i = 1, 2, 3, . . . , ki .

imediato ver que Ai = ki + 1.

5.2

Princpio Multiplicativo de Contagem

177

O conjunto de todos os possveis divisores de n vem dado pelo


conjunto A1 A2 Ar , de onde se conclui que o nmero de
divisores inteiros e positivos de n
d(n) = |A1 A2 Ar | = |A1 | |A2 | |Ar |,

onde na ltima igualdade usamos o princpio multiplicativo. Portanto,


o nmero de divisores inteiros e positivos de n
d(n) = (k1 + 1)(k2 + 1) (kr + 1).

Exemplo 5.14. De quantas maneiras podemos escolher dois inteiros


de 1 a 20 de forma que a soma seja mpar?

Soluo. Observemos que


a soma de dois nmeros inteiros pares um nmero par. Com
efeito, para quaisquer a, b Z temos que 2a + 2b = 2(a + b);
a soma de dois nmeros inteiros mpares um nmero par. Com
efeito, para quaisquer a, b Z temos que (2a + 1) + (2b + 1) =
2(a + b + 1);
a soma de um nmero inteiro par com qualquer outro inteiro

mpar sempre um inteiro mpar. Com efeito, para quaisquer


a, b Z temos que 2a + (2b + 1) = 2(a + b) + 1.

Isto nos sugere denir os conjuntos

P = {2, 4, 6, 8, 10, 12, 14, 16, 18, 20},


I = {1, 3, 5, 7, 9, 11, 13, 15, 17, 19},

onde P I so todas as formas possveis de somar um nmero inteiro


par com outro mpar. O princpio multiplicativo nos garante que nossa
resposta |P I| = |P | |I| = 100, pois |P | = |I| = 10.

178

5.3

Contagem

Uso Simultneo dos Princpios Aditivo


e Multiplicativo

Aproveitamos esta seo para apresentar problemas um pouco mais


difceis que os tratados nas sees anteriores. Nestes problemas, precisaremos empregar simultaneamente o Princpio Aditivo e o princpio
multiplicativo. Vamos ao primeiro deles:

Exemplo 5.15. Sabemos que no incio da premiao da 1a fase da

Olimpada Alagoana de Matemtica existem 10 livros diferentes de


lgebra, 7 livros diferentes de combinatria e 5 livros diferentes de
geometria para homenagear os vencedores. Danielle a primeira a
pegar o prmio que consiste em 2 livros, com a condio de que estes
no podem ser da mesma matria. Diga quantas escolhas Danielle
pode fazer para pegar seu prmio.
Soluo. Denotemos por
A = {a1 , . . . , a10 }, C = {c1 , . . . , c7 } e G = {g1 , . . . , g5 },

os conjuntos de livros de lgebra, combinatria e geometria, respectivamente. Observemos que |A| = 10, |C| = 7 e |G| = 5 e Danielle tem
as seguintes possibilidades de escolha:
escolher um livro de A e um livro de C . Neste caso, Danielle tem
|A C| = |A| |C| = 70 escolhas possveis (devido ao princpio

multiplicativo).

escolher um livro de A e um livro de G . Neste caso, Danielle tem


|A G| = |A| |G| = 50 escolhas possveis (devido ao princpio

multiplicativo) ou

5.3

Uso Simultneo dos Princpios Aditivo e Multiplicativo

179

escolher um livro de C e um livro de G . Neste caso, Danielle tem


|C G| = |C| |G| = 35 escolhas possveis (devido ao princpio

multiplicativo).

Agora o Princpio Aditivo nos garante que o nmero total de escolhas


que Danielle pode fazer 70 + 50 + 35 = 155.

Exemplo 5.16. H 18 moas e 12 rapazes, onde 5 deles so irmos

(3 moas e 2 rapazes) e os restantes no possuem parentesco. Diga


quantos casamentos so possveis naquela turma (sabendo que irmos
no se casam).

Soluo. Observemos que 15, entre as 18 moas, no tm parentesco


nenhum com os 12 rapazes, logo, pelo princpio multiplicativo temos
que possvel efetuar 15 12 = 180 casamentos diferentes entre eles.
Por outro lado, as 3 moas restantes podem efetuar casamento com 10
dos 12 rapazes, pois 2 deles so seus irmos. Novamente, pelo princpio
multiplicativo possvel realizar 310 = 30 casamentos diferentes neste
caso. Finalmente, o Princpio Aditivo nos d que podem ser realizados
um total de 180 + 30 = 210 casamentos.

Exemplo 5.17. Quantas palavras de 5 caracteres podem ser formadas

com as letras , e de modo que em cada palavra no falte nenhuma


dessas letras?
Soluo. Denamos os seguintes conjuntos,
U ={palavras de 5 caracteres s com as letras , e };

A ={palavras que esto em U e onde no aparece a letra };


A ={palavras que esto em U e onde no aparece a letra };
A ={palavras que esto em U e onde no aparece a letra }.

180

Contagem

Por exemplo,
a palavra A A ;
a palavra A ;
a palavra A .

Primeiramente, notemos que cada caracter de U pode ser escolhido


de 3 formas distintas. Segue-se ento do Princpio Multiplicativo que
existem 35 formas de escrever uma palavra de 5 caracteres usando um
alfabeto de 3 letras, isto ,
S0 = |U| = 35 = 243.

Calculemos agora |A |, isto , o nmero de palavras onde no aparece a letra . Para isto, observemos que cada caractere em A pode
ser escolhido de 2 formas. Logo, o princpio multiplicativo nos garante
que existem 25 palavras em A , ou seja, |A | = 25 . Analogamente,
podemos mostrar que |A | = |A | = 25 . Portanto,
S1 = |A | + |A | + |A | = 25 + 25 + 25 = 96.

Prosseguimos com o clculo de |A A |, isto , do nmero de palavras onde no aparecem as letras e ; portanto, cada caractere em
A A pode ser escolhido de 1 forma. Logo, o princpio multiplicativo
nos garante que existe 15 = 1 palavra em A A , ou seja, |A A | = 1.
Similarmente, podemos mostrar que |A A | = |A A | = 1. Portanto,
S2 = |A | + |A | + |A | = 3.

Por m, achamos |A A A |, que nos d o nmero de palavras onde


no aparecem as letras , e ; mas cada palavra em A A A tem

5.4

Permutaes Simples

181

que usar pelo menos um dos caracteres proibidos. Logo,


S3 = |A A A | = 0.

Finalmente, observamos que o conjunto das palavras de 5 caracteres que podem ser formadas com as letras , e de modo que em
cada palavra no falte nenhuma dessas letras exatamente o conjunto
Ac Ac Ac . Usando a Proposio 5.7, temos:
|Ac Ac Ac | =S0 S1 + S2 S3
=243 96 + 3 0

=150.

5.4

Permutaes Simples

Denimos o fatorial n! de um inteiro positivo n


n! = n (n 1) (n 2) 2 1

se n > 0 e 0! = 1, por conveno. Observe que o fatorial cresce muito


rapidamente quando n cresce. Por exemplo, para os 10 primeiros
valores de n
1!=1
2!=2
3!=6
4!=24
5!=120
6!=720 7!=5.040 8!=40.320 9!=362.880 10!=3.628.800

Denio 5.18. Uma permutao simples de n objetos distintos


qualquer agrupamento ordenado desses n objetos. Denotaremos por
Pn o nmero de todas as permutaes simples de n objetos dados.

182

Contagem

Por exemplo, todas as permutaes dos 3 elementos do conjunto


A = {a1 , a2 , a3 } so:
1 = (a1 , a2 , a3 ),
2 = (a1 , a3 , a2 ),
3 = (a2 , a1 , a3 ),
4 = (a2 , a3 , a1 ),
5 = (a3 , a1 , a2 ),
6 = (a3 , a2 , a1 ).

Proposio 5.19. Seja n 1. O nmero total de permutaes simples de n objetos O = {o1 , o2 , . . . , on } dado por Pn = n!

Demonstrao. claro que a frmula vale para n = 1. Vejamos agora


que existe a seguinte relao entre Pn e Pn1 para n 2:
Pn = nPn1 .

(5.5)

Para comprovar isto, para cada i denamos Ai como sendo as permutaes dos n 1 objetos {o1 , . . . , oi1 , oi+1 , . . . , on }. Note que |Ai | =
Pn1 , para cada i = 1, 2, . . . , n. Assim, para obtermos uma permutao dos n objetos, basta que xemos o objeto inicial oi e tomemos
um elemento do conjunto Ai , que uma permutao dos n 1 objetos
restantes. Pelo princpio aditivo, temos que:
Pn = |A1 | + |A2 | + + |An | = nPn1 .

Como a equao (5.5) vlida para todo n 2, podemos aplic-la


para n 1, obtendo:
Pn1 = (n 1)Pn2 ,

5.4

Permutaes Simples

183

de onde vem que


Pn = n(n 1)Pn2 .

Repetindo este argumento, obtemos que

Pn = n(n 1)(n 2) 3 2 1 = n!,

como queramos demonstrar.

Exemplo 5.20. De quantas maneiras podemos formar uma la com


4 pessoas?

Demonstrao. Observe que se enumeramos os lugares da la e enumeramos as pessoas, pa , pb , pc , pd , cada distribuio vai corresponder a
uma permutao do conjunto {1, 2, 3, 4}. Por exemplo, a distribuio
(pc , pa , pb , pd ) corresponde permutao (3, 1, 2, 4). Assim, o nmero
de distribuies na la 4! = 24.

Exemplo 5.21. De quantas maneiras k moas e k rapazes podem

formar pares para uma dana?

Soluo. Estando as moas em uma la e os rapazes em outra, podemos enumer-los com nmeros de 1, 2, . . . , k . A uma permutao desses nmeros, digamos (a1 , a2 , . . . , ak ) com ai {1, 2, . . . , k} faremos
uma associao da mulher i com o rapaz ai . Por exemplo, a permutao (2, 1, 3, . . . , k) signica que a moa 1 danar com o rapaz 2, a
moa 2 com o rapaz 1, e a moa i com o rapaz i, para i 3.
Observe que toda associao de k moas e k rapazes produz uma
permutao, de modo que o nmero de associaes possveis das moas com os rapazes igual ao nmero de permutaes dos elementos
do conjunto {1, 2, 3, . . . , k}. Pela Proposio 5.19 existem k! modos
diferentes de combinar as moas com os rapazes.

184

5.5

Contagem

Arranjos Simples

Denio 5.22. Consideremos n objetos e p um inteiro positivo tal

que 0 < p n. Um arranjo simples de classe p dos n objetos dados


uma seleo de p objetos distintos dentre estes que diferem entre si
pela ordem de colocao ou pela natureza de cada um, isto , o que
importa quem participa ou o lugar que ocupa. Denotaremos por Apn
o nmero de arranjos simples de classe p de n objetos.
Por exemplo, dados os objetos o1 , o2 e o3 todos os arranjos possveis
de classe 2 so: A1 = (o1 , o2 ), A2 = (o2 , o1 ), A3 = (o1 , o3 ), A4 =
(o3 , o1 ), A5 = (o2 , o3 ) e A6 = (o3 , o2 ).

Observao 5.23. Notemos que um arranjo simples de classe n de n

objetos dados no mais que uma permutao desses n objetos. Logo,


Pn = Ann = n!.

Proposio 5.24. Seja n 1. O nmero total de arranjos simples


de classe p de n objetos O = {o1 , o2 , . . . , on } dado por Apn =

n!
.
(np)!

Demonstrao. Para n = 1 a frmula obviamente vlida. Similarmente ao caso das permutaes, primeiramente provaremos que para
n 2 vale a seguinte igualdade:
Apn = nAp1
n1 .

(5.6)

Agora denimos os conjuntos Ai como sendo os arranjos simples de


classe p 1 dos n 1 objetos {o1 , . . . , oi1 , oi+1 , . . . , on }. Note que
|Ai | = Ap1
n1 , para cada i = 1, 2, . . . , n. Assim, para obtermos um
arranjo simples de classe p dos n objetos, basta que xemos o objeto
inicial oi e tomemos um elemento do conjunto Ai , que uma arranjo

5.5

Arranjos Simples

185

de classe p 1 dos n 1 objetos restantes. Pelo princpio aditivo,


temos que:
Apn = |A1 | + |A2 | + + |An | = nAp1
n1 .

Como nossa equao (5.6) vlida para todo n 2, podemos aplic-la


para n 1, obtendo:
p2
Ap1
n1 = (n 1)An2 ,

de onde vem que


Apn = n(n 1)Ap2
n2 .

Repetindo este argumento sucessivamente, obtemos que


p(p1)

Apn = n(n 1)(n 2) (n (p 2))An(p1)


= n(n 1)(n 2) (n p + 2)A1np+1 .

Notemos agora que A1np+1 = n p + 1; logo, da igualdade anterior


segue-se que
Apn = n(n 1)(n 2) (n p + 2)(n p + 1)

n(n 1)(n 2) (n p + 2)(n p + 1) (n p) 1


(n p) 1
n!
=
,
(n p)!

como desejvamos.
Agora vamos dar alguns exemplos de como aparecem problemas
prticos que requerem fazer este tipo de clculo. O primeiro dele tem

186

Contagem

a ver com a formao de palavras diferentes com um conjunto dado


de letras.
Um anagrama de uma palavra uma permutao de letras dessa
palavra para formar outra, a qual pode carecer de signicado. Por
exemplo:
um anagrama de amor roma;
um anagrama de celia alice;
um anagrama de caterina natercia;
um anagrama de elvis lives.

Exemplo 5.25. Quantos anagramas de p letras distintas podemos

formar com um alfabeto de 23 letras, sendo p < 23?

Soluo. Como as letras so diferentes, nosso problema consiste em


achar todos os arranjos de classe p de 23 objetos dados, que neste caso
so as 23 letras do alfabeto. Logo, este nmero
Ak23 =

23!
.
(23 k)!

Exemplo 5.26. De quantos modos 2 pessoas podem se sentar em 5


cadeiras que esto em la?

Soluo. Este problema equivalente a achar o nmero total de arranjos de classe 2 de 5 objetos, correspondendo as 5 cadeiras aos 5
objetos e as duas pessoas indicando a ordem do arranjo. Logo, este
nmero dado por
A25 =

5!
= 20.
3!

5.5

Arranjos Simples

187

Exemplo 5.27. Considere os dgitos 2, 3, 4, 5, 7 e 9. Supondo que a


repetio de dgitos no seja permitida, responda s seguintes perguntas:
(a) Quantos nmeros de trs dgitos podem ser formados?
(b) Entre os achados em (a) quantos so pares?
(c) Entre os achados em (a) quantos so mpares?
(d) Entre os achados em (a) quantos so mltiplos de 5?
(e) Entre os achados em (a) quantos so menores do que 400?
Soluo. Seja O = {2, 3, 4, 5, 7, 9} nosso conjunto de objetos.
(a) A quantidade de nmeros de trs dgitos que podemos formar
sem repetio de algum deles claramente o nmero de arranjos
de classe 3 dos 6 dgitos de O, isto ,
A36 =

6!
= 120.
3!

(b) Sabemos que em todo nmero par o ltimo dgito um mltiplo


de 2, isto , ele acaba em 0, 2, 4, 6 ou 8. Ento, em nosso caso
as nicas possibilidades so que o nmero termine em 2 ou 4.
Supondo que o ltimo dgito seja 2, temos que preencher as duas
casas restantes com os dgitos pertencentes ao conjunto O {2}.
= 20 nmeros dos achados
Assim, existem A2|O{2}| = A25 = 5!
3!
em (a) que nalizam em 2. De forma anloga, existem A2|O{4}| =
5!
A25 = 3!
= 20 nmeros dos achados em (a) que nalizam em 4.
Logo, entre os nmeros achados em (a) existem 20 + 20 = 40
nmeros pares.

188

Contagem

(c) Todo conjunto de nmeros pode ser dividido em duas classes


disjuntas: a classe dos nmeros pares e a classe dos nmeros mpares que pertencem ao mesmo. Segue-se que dentre os nmeros
achados em (a) existem 120 40 = 80 nmeros mpares.
(d) Todo nmero mltiplo de 5 acaba em 0 ou 5; no nosso caso temos que a nica possibilidade para o ltimo dgito 5. Assim
o problema consiste em preencher as duas casas restantes com
dgitos do conjunto O {5}. De onde se segue que a quantidade de nmeros mltiplos de 5 existentes em (a) vem dada por
5!
A2|O{5}| = A25 = 3!
= 20.
(e) Para obter os nmeros menores do que 400 a casa das centenas
s poder ser ocupada pelos dgitos 1, 2 ou 3. Como 1
/ O,
temos que as nicas possibilidades em nosso caso so 2 ou 3.
Ento, supondo que o primeiro dgito do nmero seja 2, devemos
preencher duas casas restantes com os dgitos pertencentes a
= 20
O {2}. De forma anloga, existem A2|O{3}| = A25 = 5!
3!
nmeros dos achados em (a) e que comeam com 3. Logo, dentre
os nmeros achados em (a) existem 20 + 20 = 40 menores do que
400.

5.6

Combinaes Simples

O conceito de combinao simples surge naturalmente quando tentamos responder seguinte pergunta: de quantas formas diferentes podemos selecionar p objetos dentro de n objetos dados?

5.6

Combinaes Simples

189

Por exemplo, suponha que queremos enfeitar uma festa de aniversrio com bolas de dois tipos de cores e na loja onde as compraremos
existem bolas nas cores azul, verde e vermelha. De quantas formas
distintas podemos enfeitar nossa festa? claro que podemos enfeitar
a festa de 3 formas diferentes: com bolas em azul e verde; com bolas
em azul e vermelho ou com bolas em verde e vermelho.
Notemos que, ao contrrio do caso em que trabalhamos com arranjos, quando fazemos uma seleo de duas cores no estamos interessados na ordem em que elas foram escolhidas.

Denio 5.28. Consideremos n objetos e p um inteiro positivo tal

que 0 < p n. Uma combinao simples de classe p dos n objetos


dados uma seleo de p objetos distintos entre estes que diferem
entre si apenas pela natureza de cada um, isto , o que importa
simplesmente quem participa no grupo selecionado. Denotaremos por

n
o nmero de combinaes simples de classe p de n objetos.
p

Proposio 5.29. Seja n 1. O nmero total de combinaes

simples de classe p de n objetos O = {o1 , o2 , . . . , on } dado por


n
p

n!
.
p!(np)!

Demonstrao. Veremos a seguir que arranjos simples e combinaes


simples de classe p esto estreitamente relacionados. Com efeito, para
cada combinao simples formada por p objetos distintos de O podemos gerar todos os arranjos simples de classe p formados por estes p
objetos. Basta para isto fazer todas as suas permutaes possveis.
Obtm-se assim p ! arranjos simples diferentes com esses p objetos.
Resumindo, para cada combinao simples de classe p formada com
p objetos diferentes de O podemos fazer p ! arranjos simples diferentes de classe p com estes mesmos objetos; logo, no total, teremos a

190

seguinte relao:
de onde segue-se que

Contagem

 
n
n!
p!
= Apn =
,
p
(n p)!
 
n
n!
=
.
p
p!(n p)!

Exemplo 5.30. De quantas formas diferentes podemos construir uma


palavra de tamanho n com i letras a e n i letras b?

Soluo. A soluo do problema equivale em escolher a posio das


i letras a em questo, uma vez que a posio das (n i) letras b
restantes estar determinada. Se enumeramos as posies das letras
de 1 a n, uma palavra ser formada ao xarmos a posio das i letras

a. Isso exatamente ni , j que corresponde ao nmero de grupos
com i elementos (posies com letra a) tomados em um conjunto de n
elementos (todas as posies), que diferem somente por sua natureza.

Exemplo 5.31. De quantas formas podemos dividir um grupo 5 pes-

soas em um grupo de duas e outro de trs?




Soluo. Temos 52 = 25!3! ! = 10 formas diferentes de escolher duas


pessoas do grupo. Por cada uma dessas escolhas o outro grupo de trs
pessoas automaticamente determinado; logo, temos 10 possibilidades
diferentes de fazer a diviso.

Exemplo 5.32. De quantos modos podemos dividir 6 pessoas em:


(a) Dois grupos de 3 pessoas cada?

5.6

Combinaes Simples

191

(b) Trs grupos de 2 pessoas cada?


Soluo. Comeamos por (a). primeira vista, parece que a resposta

deve ser n3 = 3!6!3! = 20, similarmente ao exemplo anterior. Porm,
aqui h um problema devido ao fato de estarmos dividindo em grupos
que tm a mesma quantidade de pessoas e, portanto, as permutaes
de cada dois grupos formados so consideradas divises iguais; logo,
devemos dividir o resultado por 2 !, obtendo assim 10 formas diferentes
de obter dois grupos com 3 pessoas cada.
Para resolver o item (b) seguimos os seguintes passos:
Primeiramente calcularemos o nmero de formas possveis para

dividir 6 pessoas em um grupo de 2 e outro grupo de 4; esta



quantidade vem dada por 62 = 4!6!2! .

Agora dividiremos as 4 pessoas restantes em um grupo de 2 e



outro grupo de 2; esta quantidade vem dada por 42 = 2!4!2! .
 
6!
Pelo princpio multiplicativo temos que existem 62 42 = (2!)
3 possi-

bilidades de dividir 6 pessoas em 3 grupos com duas pessoas cada.


Igualmente ao caso anterior, aqui as permutaes possveis de cada 3
grupos formados so consideradas iguais; logo, devemos dividir este
ltimo resultado por 3 !. Portanto, existem 15 formas diferentes de
dividir 6 pessoas em trs grupos de 2 pessoas cada.

Exemplo 5.33. Se voc possui 10 amigos, de quantas maneiras voc

pode escolher dois ou mais deles para jantar?

Soluo. Esquematizamos a soluo da seguinte maneira:


Primeiramente, vamos encontrar a quantidade de maneiras pelas

10

quais voc pode jantar com 2 amigos; isto feito de


diferentes.

formas

192

Contagem

Depois, vamos encontrar a quantidade de maneiras pelas quais



10

voc pode jantar com 3 amigos; isto feito de


rentes.

formas dife-

Em seguida, encontramos a quantidade de maneiras pelas quais



10

voc pode jantar com 4 amigos; isto feito de


rentes.

formas dife-

Em geral, o nmero de maneiras diferentes que voc tem de



jantar com p amigos dado por 10p .

Pelo princpio aditivo, temos que a quantidade de formas diferentes


que voc tem de jantar com 2 ou mais de seus amigos, dada por
   
   
10
10
10
10
+
+ +
+
= 1013,
2
3
9
10

sendo este o nmero procurado.

Exemplo 5.34. De um grupo de 10 pessoas das quais 4 so mulheres,

quantas comisses de 5 pessoas podem ser formadas de modo que pelo


menos uma mulher faa parte?

Soluo. Sendo que o grupo tem 10 pessoas e 4 destas so mulheres,


segue-se que no grupo temos 6 homens. Para formar um grupo de 5
pessoas com pelo menos uma mulher, temos as seguintes alternativas:
Nosso grupo composto por uma mulher e 4 homens; neste caso
 
poderemos formar 41 64 = 60 comisses de 5 pessoas.

Nosso grupo composto por 2 mulheres e 3 homens; neste caso


 
poderemos formar 42 63 = 120 comisses de 5 pessoas.

5.7

O Binmio de Newton

193

Nosso grupo composto por 3 mulheres e 2 homens; neste caso


 
poderemos formar 43 62 = 60 comisses de 5 pessoas.
Nosso grupo composto por 4 mulheres e um homem; neste caso
 
poderemos formar 44 61 = 6 comisses de 5 pessoas.

Pelo princpio aditivo temos que possvel formar 246 comisses de 5


pessoas de modo que pelo menos uma mulher faa parte.

5.7

O Binmio de Newton

Nesta seo, estudaremos uma frmula que generaliza a conhecida


expresso
(a + b)2 = a2 + 2ab + b2 .

Essa frmula conhecida como o binmio de Newton ou frmula binomial de Newton, devido ao Matemtico Isaac Newton (1642-1727). A
frmula binomial de Newton pode ser motivada pelas seguintes igualdades que so fceis de vericar:
 
 
1
1
a+
b,
(a + b) = a + b =
0
1
 
 
 
2 2
2
2 2
2
2
2
(a + b) = a + 2ab + c =
a +
ab +
b,
0
1
2
 
 
 
 
3 3
3 2
3
3 3
3
3
2
2
3
2
(a + b) = a + 3a b + 3ab + c =
a +
a b+
ab +
b.
0
1
2
3
1

Os casos particulares acima podem ser estendidos para qualquer potncia inteira positiva de a + b, ou seja, vale o seguinte resultado:

194

Contagem

Teorema 5.35 (Frmula Binomial de Newton) . Sejam a e b nmeros


reais e n N, ento

 
 
 


 
n n
n n1 1
n ni i
n
n n
1 n1
(a+b) =
a +
a
b + +
a b + +
a b
+
b .
0
1
i
n1
n

Os nmeros ni , 0 i n, so chamados tambm de coecientes binon

miais.

Demonstrao. Expandimos o binmio no produto de seus n fatores,


isto ,
(a + b)n = (a + b)(a + b) (a + b) .
{z
}
|

(5.7)

nfatores

Se desenvolveremos o produto destes n fatores iguais acima obtemos


uma soma nita de termos da forma a1 a2 an , onde cada aj , 1
j n, toma valor a ou b. Notemos que em cada termo se o nmero
b aparece i vezes, ento o nmero a aparecer (n i) vezes, ou seja,
quando cada termo for multiplicado dever tomar valor igual a ani bi ,
para algum 1 i n. Por exemplo, os n termos
abb b = abn ,

bab b = abn ,

...,

bbb ba = abn

tm o mesmo valor . Assim, para calcular o coeciente do termo ai bni


que aparece na equao (5.7), basta responder seguinte pergunta: de
quantos modos podemos formar uma palavra com i letras a e (n i)
letras b? A resposta dessa pergunta foi estudada no Exemplo 5.30 e

simplesmente ni . Logo, a expresso na equao (5.7)
 
 


 
n n
n n
n n1 1
n
1 n1
(a + b) =
a +
a b + +
ab
+
b ,
0
1
n1
n
n

o que prova o teorema.

5.8

Contagem e Probabilidades

195

A frmula binomial de Newton nos d algumas propriedades interessantes dos coecientes binomiais que resumimos na prxima proposio.

Proposio 5.36. Seja n N. As seguintes igualdades so vlidas:






n
+ + n1
+ nn = 2n ;




(b) n0 n1 + + (1)i ni + + (1)n nn = 0.

(a)

n
0

n
1

+ +

n
i

Demonstrao. Para a letra (a), basta tomar a = b = 1 e expanda


2n = (1 + 1)n no Binmio de Newton. Para a letra (b), tome a = 1
e b = 1 e expanda 0 = (1 1)n no binmio de Newton, observando
que (1)n igual a 1 se n par, e igual a 1 se n mpar.
5.8

Contagem e Probabilidades

Uma das aplicaes interessantes da contagem de elementos de um


conjunto quando desejamos estudar a probabilidade de eventos aleatrios. Por exemplo, se lanarmos um dado de seis faces, temos os
seguintes resultados possveis:
= {1, 2, . . . , 6}.

Se desejamos saber qual a chance de que ocorra um nmero


primo no lanamento, devemos contar quantos primos aparecem em
{1, 2, 3, 4, 5, 6} e dividir por 6. Ou seja, a chance de ocorrer um nmero
primo num lanamento de um dado de seis faces 3/6 = 0, 5.
Denimos a probabilidade de um subconjunto A como o nmero
p(A) =

|A|
.
||

196

Contagem

Tambm chamamos o subconjunto de todos os resultados possveis


de espao amostral e um subconjunto A de de evento. Por exemplo,
podemos calcular a probabilidade de escolhermos um nmero par no
conjunto 1, 2, 3, . . . , 15. Neste caso, o conjunto est claro e igual a
= {1, 2, 3, . . . , 15}. O conjunto A A = {2, 4, 6, . . . , 14}. Logo,
p(A) =

7
|A|
= .
||
15

Assim, ca claro que a maior diculdade para calcular a probabilidade de um evento contar quantos elementos pertencem a este
evento e quantos elementos pertencem ao espao amostral. A seguir,
veremos um exemplo mais elaborado onde aplicamos a noo de arranjo simples.

Exemplo 5.37. Calcular a probabilidade de que escolhendo um grupo

de 44 pessoas, existam pelo menos duas que fazem aniversrio no


mesmo dia do ano.

Soluo. Podemos reescrever isso do seguinte modo: num saco existem


bolas enumeradas com os nmeros 1, 2, . . . , 365 (correspondentes aos
dias do ano). Retiramos a bola b1 e anotamos o nmero que apareceu.
Devolvemos a bola ao saco e efetuamos uma nova retirada, anotando
novamente o nmero que aparece. Repetindo este processo 44 vezes,
obtemos uma lista com 44 nmeros. Assim, a pergunta se transforma
em: de quantos modos diferentes podemos escolher 44 bolas enumeradas com os nmeros 1, 2, 3, . . . , 365 com reposio, tal que existam
pelo menos duas bolas com o mesmo nmero?
A primeira coisa que devemos fazer calcular o espao amostral,
de todas as possibilidades possveis de resultado. Como escolhemos
44 bolas enumeradas num saco, cada resultado possvel uma lista

5.9

Exerccios Propostos

197

(n1 , n2 , . . . , n44 ) com 44 nmeros. Observe que, pelo princpio multiplicativo, || = 36544 , pois temos 365 opes para escolher n1 , 365
opes para escolher n2 , etc.

A segunda pergunta trata-se de saber quantos resultados so favorveis, ou seja, quantas so as escolhas tais que existam pelo menos
duas bolas com o mesmo nmero. Para isso mais fcil contar quantas
escolhas existem tais que os 44 nmeros so diferentes. Neste caso,
devemos escolher uma ordenao de 44 nmeros distintos entre 365.
Isso corresponde quantidade de arranjos de classe 44 num grupo de
365 elementos. Assim, conclumos que a probabilidade de que este
evento ocorra
36544 A44
365
=1
p=
36544

365!
(365!44!)
.
36544

Obter um valor aproximado para o nmero acima com o computador


uma tarefa fcil nos dias atuais. Porm, aproximar expresses envolvendo
fatoriais (sem o uso do computador) um fato conhecido h muito tempo
pela humanidade, atravs da famosa frmula de Stirling.1 Com a ajuda
desta frmula, obtemos que p aproximadamente p
= 0.93, como havamos
prometido no Captulo 1.

Alm dos exerccios abaixo, recomendamos a leitura de [9]. L, o


leitor encontrar material adicional sobre anlise combinatria, bem
como uma ampla variedade de problemas.
5.9

Exerccios Propostos

1. De quantas maneiras podemos escolher trs nmeros distintos do


conjunto I50 = {1, 2, 3, . . . , 49, 50} de modo que sua soma seja

1 Grosseiramente,
n n

2nn e

a frmula de Stirling diz que o quociente entre n! e


est prximo de 1, para valores de n grandes.

198

Contagem

a) um mltiplo de 3?
b) um nmero par?
2. Considere o conjunto In = {1, 2, 3, . . . , n1, n}. Diga de quantos
modos possvel formar subconjuntos de k elementos nos quais
no haja nmeros consecutivos?
3. Considere as letras da palavra PERMUTA. Quantos anagramas
de 4 letras podem ser formados, onde:
a) no h restries quanto ao nmero de consoantes ou vogais?
b) o anagrama comea e termina por vogal?
c) a letra R aparece?
d) a letra T aparece e o anagrama termina por vogal?
4. Calcular a soma de todos os nmeros de 5 algarismos distintos
formados com os algarismos 1, 3, 5, 7 e 9.
5. Quantos nmeros podem ser formados pela multiplicao de alguns ou de todos os nmeros 2, 2, 3, 3, 3, 5, 5, 6, 8, 9, 9?
6. Entre todos os nmeros de sete dgitos, diga quantos possuem
exatamente trs dgitos 9 e os quatro dgitos restantes todos
diferentes?
7. De quantas maneiras podemos distribuir 22 livros diferentes entre 5 alunos se 2 deles recebem 5 livros cada e os outros 3 recebem
4 livros cada?
8. Quantos so os nmeros naturais de sete dgitos nos quais o
dgito 4 gura exatamente 3 vezes e o dgito 8 gura exatamente
2 vezes?

5.9

Exerccios Propostos

199

9. De quantas maneiras uma comisso de 4 pessoas pode ser formada, de um grupo de 6 homens e 6 mulheres, se a mesma
composta de um nmero maior de homens do que de mulheres?
10. O comprimento de uma palavra a quantidade de caracteres que
ela possui. Encontre a quantidade de palavras de comprimento
5 que podemos formar fazendo uso de 10 caracteres distintos, de
forma que no existam trs caracteres consecutivos idnticos em
cada palavra.
11. Quantos nmeros inteiros existem entre 1 e 10.000 que no so
divisveis por 3, 5 e 7?
12. Quantas so as permutaes da palavra PROPOR nas quais no
existem letras consecutivas iguais?
13. De quantos modos 6 casais podem sentar-se ao redor de uma
mesa circular de tal forma que marido e mulher no quem juntos?
14. Quantas so as permutaes das letras da palavra BRASIL em
que o B ocupa o primeiro lugar, ou o R ocupa o segundo lugar,
ou o L o sexto lugar?
15. De quantas formas podemos representar o nmero 15 como soma
de vrios nmeros naturais?
16. Quantos quadrados perfeitos existem entre 40.000 e 640.000 que
so mltiplos simultaneamente de 3, 4 e 5?
17. Oito amigos vo ao cinema assistir a um lme que custa um real.
Quatro deles possuem uma nota de um real e quatro possuem

200

Contagem

uma nota de dois reais. Sabendo-se que o caixa do cinema no


possui nenhum dinheiro, como eles podem organizar uma la
para pagar o lme permitindo o troco pelo caixa?
18. Se considerarmos todas as conguraes do tabuleiro com duas
torres que no se atacam, como no Exemplo 5.2, sem distinguir
as torres, quantas conguraes obteremos?
19. Continuando o problema anterior, generalize-o para 3, 4, 5, . . .
torres que no se atacam, encontrando tambm o nmero mximo de torres que podem ser colocadas no tabuleiro de modo que
duas delas no se ataquem.
20. Tente fazer o problema anterior para cavalos de xadrez.
21. Mostre que em toda sequncia de n2 + 1 inteiros distintos possui
uma subsequncia crescente de n + 1 elementos ou uma subsequncia decrescente de n + 1 elementos.
22. Encontre o nmero de zeros que termina o nmero 2010!.
23. O jogo do 7 consiste em lanar dois dados e somar o nmero
obtido nas suas faces. Caso a soma seja 7, o jogador A ganha o
dois reais do jogador B . Caso a soma no seja 7, o jogador B
ganha um real de A. Pergunta-se: quem leva vantagem?
24. A funo de Euler associa a cada nmero natural n o valor
(n) igual ao nmero de inteiros positivos menores ou iguais a
n relativamente primos com n. Ou seja,


(n) = {1 m n; (m, n) = 1} .

5.9

Exerccios Propostos

201

Usando os princpios estudados, mostre que se n se decompe


em fatores primos como n = p1 1 p2 2 . . . pk k , ento


 

1
1
1
(n) = n 1
1
... 1
.
p1
p2
pk

O leitor pode achar mais informaes sobre a funo de Euler


nos livros [11] ou ainda [10].

202

Contagem

6
Induo Matemtica
Se as pessoas no a ham a Matemti a simples s por que ainda
no per eberam o quanto a vida ompli ada.
John von Neumann

Imagine uma la com innitos domins, um atrs do outro. Suponha que eles estejam de tal modo distribudos que, uma vez que um
domin caia, o seu sucessor na la tambm cai. O que acontece quando
derrubamos o primeiro domin?
Apesar da simplicidade da pergunta acima ela traz em sua essncia
toda a ideia usada no mtodo da induo nita . Muitas descobertas
em Matemtica so feitas baseadas na realizao de testes que nos
fornecem evidncias empricas. Tais evidncias so estudadas para
efetivamente vericarmos se os resultados que elas insinuam so verdadeiros. O mtodo da induo nita constitui uma ferramenta muito
til na hora de desvendar a veracidade de resultados provenientes deste
tipo de estudo. Esse mtodo uma das grandes armas do matemtico
moderno e tem utilidade na soluo de vrios problemas, como iremos
ver ao longo deste captulo.
203

204

6.1

Induo Matemtica

Formulao Matemtica

No incio do sculo XX, o matemtico Giuseppe Peano (1858-1932)


estabeleceu os axiomas necessrios que nos permitem hoje descrever
com preciso o conjunto dos nmeros naturais. O ltimo dos seus
axiomas diz o seguinte: seja A um subconjunto de N (A N). Se 1
A e se, alm disso, A contm todos os sucessores dos seus elementos,
ento A = N.
Este axioma conhecido como axioma de induo e serve como
base do mtodo de demonstrao por induo, o qual de grande
utilidade para estabelecer provas rigorosas em Matemtica.
O princpio da boa ordenao dos naturais, enunciado no Captulo 3, e o axioma de induo no so independentes e sem nenhuma
conexo. De fato, eles so equivalentes, ou seja, se consideramos o
princpio da boa ordenao como sendo um postulado podemos deduzir o axioma de induo e, reciprocamente, se consideramos o axioma
de induo como sendo um postulado podemos deduzir o princpio da
boa ordenao.
No resto do captulo, p(n) representa uma armao em relao ao
natural n, podendo esta ser verdadeira ou falsa.

Teorema 6.1 (Princpio da Induo Finita) . Considere n0 um in-

teiro no negativo. Suponhamos que, para cada inteiro n n0 , seja


dada uma proposio p(n). Suponha que se pode vericar as seguintes
propriedades:
(a) p(n0 ) verdadeira;
(b) se p(n) verdadeira ento p(n + 1) tambm verdadeira, para
todo n n0 .

6.1

Formulao Matemtica

205

Ento, p(n) verdadeira para qualquer n n0 .


A armao (a) chamada de base da induo e a (b) de passo
indutivo. O fato de que p(n) verdadeira no item (b) chamado de
hiptese da induo.

Demonstrao. Denamos o conjunto


V = {m inteiros no negativos ; m n0 e p(m) verdadeira} .

Notemos que V no vazio, pois a condio (a) nos assegura que


n0 V . A prova do teorema equivalente a mostrarmos que
V = {n0 , n0 + 1, n0 + 2, n0 + 3, },

ou equivalentemente, a provarmos que o conjunto


F = {m inteiros no negativos ; m n0 e p(m) falsa}

vazio. Suponhamos que F no vazio. Pelo principio da boa ordenao existe um menor elemento m0 F , onde p(m0 ) falso. Observemos que,
m0 n0 + 1. De fato, m0 n0 , porm a possibilidade m0 = n0

contradiz a condio (a);

m0 1 V . Com efeito, p(m0 1) verdadeira pois, caso


contrrio, m0 1 F e, alm disso, m0 1 < m0 , contradizendo
isto a minimalidade de m0 .

Finalmente, como p(m0 1) verdadeira, segue da condio (b) que


p(m0 ) tambm verdadeira, o que impossvel pela denio de m0 .
Portanto, o conjunto F vazio, concluindo-se assim a prova.

206

Induo Matemtica

Para um pouco mais sobre a relao entre os princpios de induo


e da boa ordenao, recomendamos o Apndice A da referncia [11].

Observao 6.2. Uma grande vantagem do princpio da induo nita poder provar que uma quantidade innita de armaes so
verdadeiras, simplesmente vericando que uma quantidade nita destas armaes so verdadeiras. Deixaremos clara a utilidade deste
mtodo resolvendo alguns problemas na prxima seo.
6.2

Aplicaes

Dentro da grande gama de problemas que podem ser abordados aplicando o mtodo de induo podemos distinguir trs importantes grupos:
demonstrao de identidades;
demonstrao de desigualdades;
demonstrao de problemas de divisibilidade.

A seguir damos vrios exemplos de como aplicar o mtodo em


problemas referentes a cada um destes grupos.
6.2.1

Demonstrando Identidades

Comeamos com os seguintes problemas clssicos:

(P1) Determinar uma frmula para a soma dos n primeiros nmeros


pares, isto ,

sp (n) := 2 + 4 + 6 + + 2n.

6.2

Aplicaes

207

(P2) Determinar uma frmula para a soma dos n primeiros nmeros


mpares, isto ,

si (n) := 1 + 3 + 5 + + 2n 1.

Para induzir ambas as frmulas, primeiro fazemos os clculos para


vrios valores de n, os quais apresentamos na seguinte tabela.
n
1
sp (n) 2 = 1 2
si (n) 1 = 12

6=23

12 = 3 4

20 = 4 5

4 = 22

9 = 32

16 = 42

30 = 5 6
25 = 52

Os resultados na tabela sugerem que sp (n) = n(n + 1) e que


si (n) = n2 . Entretanto, isto no constitui por si s uma prova rigorosa destas frmulas, pois para poder garantir a veracidade delas
utilizando a tabela teramos que vericar cada valor de n natural,
sendo isto impossvel. Provaremos agora que, de fato, as frmulas
induzidas so vlidas usando o mtodo de induo nita.

Exemplo 6.3. Demonstre que para qualquer n N vlida a igualdade:

2 + + 2n = n(n + 1).

Soluo. Denamos a proposio


p(n) : 2 + + 2n = n(n + 1)

e observemos que a mesma vale para n = 1 (base da induo); de fato


p(1) : 2 = 1(1 + 1).

Agora partimos para a prova do passo indutivo:

208

Induo Matemtica

Hiptese: suponhamos que p(k) verdadeira para um certo k >


1, k N.
Tese: devemos mostrar que p(k + 1) tambm verdadeira.

Com efeito, como


2 + + 2k = k(k + 1),

somando 2(k + 1) a ambos os lados desta igualdade, temos que


2 + + 2k + 2(k + 1) = k(k + 1) + 2(k + 1)
= (k + 2)(k + 1).

Esta ltima igualdade arma que p(k + 1) tambm verdadeira. O


Princpio de Induo nos garante que p(n) verdadeira para qualquer
n N.

Exemplo 6.4. Demonstre que para qualquer n N vlida a igualdade:

1 + 3 + 5 + + 2n 1 = n2 .

Soluo. Aqui denimos a proposio:


p(n) : 1 + 3 + 5 + + 2n 1 = n2

e notamos que a mesma vlida se tomarmos, por exemplo, n = 1.


De fato,
p(1) : 1 = 2 1 1.

Agora s resta provar o passo indutivo:

Hiptese: suponhamos que p(k) seja verdadeira para um certo


k > 1, k N.

6.2

Aplicaes

209

Tese: devemos mostrar que p(k + 1) tambm verdadeira.

Com efeito, como


1 + 3 + 5 + + 2k 1 = k 2 ,

somando 2k + 1 a ambos os lados desta igualdade, temos que


1 + 3 + 5 + + 2k 1 + 2k + 1 = k 2 + 2k + 1
= (k + 1)2 .

O princpio de induo nos garante que p(n) verdadeira para


qualquer n N.
Uma consequncia imediata do Exemplo 6.3 a frmula para a
soma dos n primeiros nmeros naturais, dada por
sn = 1 + 2 + 3 + + n =

n(n + 1)
.
2

(6.1)

Com efeito, como


2 + 4 + + 2n = n(n + 1),

ento dividindo por 2 ambos os membros da igualdade acima, obtemos


a equao (6.1).
Continuando com o mesmo raciocnio, natural nos perguntarmos
se possvel obter uma frmula para a soma dos n primeiros quadrados
perfeitos, ou seja, determinar qn onde:
qn = 12 + 22 + 32 + + n2 .

Para induzir a frmula, consideramos os valores de sn e qn numa tabela:

210

Induo Matemtica

1 2 3 4 5 6
1 3 6 10 15 21
1 5 14 30 55 91

n
sn
qn

Aparentemente no existe nenhuma relao entre sn e qn . Mas, se


considerarmos o quociente qn /sn , vejamos o que acontece:
n

qn /sn

3/3

5/3

7/3

9/3

11/3

13/3

Isso nos sugere que vale a relao


qn
2n + 1
=
,
sn
3

logo nosso candidato para valor de qn


qn =

sn (2n + 1)
n(n + 1)(2n + 1)
=
.
3
6

Convidamos o leitor a provar a veracidade da equao acima utilizando o


Mtodo da Induo no Exerccio 1 no nal do captulo.
6.2.2

Demonstrando Desigualdades

Apresentamos agora alguns exemplos de como usar induo para provar


desigualdades.

Exemplo 6.5. Prove que 3n1 < 2n2 para todo n N.


Soluo. Denotamos por p(n) a propriedade: 3n1 < 2n . claro que p(1)
vlida, pois 1 < 2. Agora supondo que P (n) verdadeira temos que
2

3n = 3n1 3 < 2n 22n+1 = 2(n+1) ,

logo p(n + 1) tambm vale. Observamos que na desigualdade acima usamos


o fato de que 3 < 22n+1 para qualquer n N.

6.2

Aplicaes

211

Exemplo 6.6. Mostre que para todo nmero n N, n > 3, vale que 2n < n!
Demonstrao. Para n = 4 a desigualdade vericada, pois 24 = 16 < 4! =
24. Vamos assumir como hiptese de induo que a desigualdade vlida
para n 4. Ento, precisamos mostrar que a mesma vale tambm para
n + 1. De fato, por hiptese de induo:
(6.2)

2n < n!

Como 2 < n + 1, podemos multiplicar o lado esquerdo da desigualdade


em (6.2) por 2 e o lado direito por n+1, sem alterar o sinal de desigualdade.
Logo, temos que:
2n .2 = 2n+1 < n!(n + 1) = (n + 1)!,

concluindo-se a demonstrao.

Exemplo 6.7. Prove que, para todo n N,


s

2+

2+

2 + + 2 < 2.
{z
}

nradicais

Demonstrao. Claramente a desigualdade vale para n = 1, pois 2 < 2.


Suponhamos que para certo n N a desigualdade acontece, ento
s

2+

2 + 2 + + 2 < 2.
{z
}
nradicais

Logo, adicionando 2 em ambos os lados desta desigualdade tem-se


2+

2+

2 + 2 + + 2 < 2 + 2.
{z
}
nradicais

212

Induo Matemtica

Tomando raiz quadrada em ambos os lados desta ltima desigualdade obtemos


v
s
u
r
u
q

t
2 + 2 + 2 + 2 + + 2 < 2,
{z
}
|
n+1radicais

como desejvamos.

6.2.3

Induo e Problemas de Divisibilidade

Agora damos alguns exemplos de problemas de divisibilidade que podem


ser mostrados utilizando o mtodo da induo:

Exemplo 6.8. Mostre que para qualquer n N,

n3 + 2n sempre divisvel

por 3.

Soluo. Para n = 1 a armao vlida, pois 13 +21 = 3, que obviamente


divisvel por 3.
Assumamos como hiptese indutiva que a armao vale para algum
k N, isto ,
Hiptese: k3 + 2k divisvel por 3.

Devemos mostrar que a armao tambm verdadeira para k + 1, ou


seja, temos que provar que
Tese: (k + 1)3 + 2(k + 1) divisvel por 3.
Para provar isto ltimo, usamos o fato de que
(k + 1)3 + 2(k + 1) = (k 3 + 3k 2 + 3k + 1) + (2k + 2);

6.2

Aplicaes

213

agrupando adequadamente,
(k + 1)3 + 2(k + 1) = (k 3 + 2k) + (3k 2 + 3k + 3)
= (k 3 + 2k) + 3(k 2 + k + 1)
| {z } |
{z
}
mltiplo de 3

mltiplo de 3

= mltiplo de 3,

concluindo assim a prova.

Exemplo 6.9. Mostre que a soma dos cubos de trs nmeros naturais consecutivos divisvel por 9.

Soluo. Denamos a seguinte proposio:


p(n) : n3 + (n + 1)3 + (n + 2)3 um mltiplo de nove.

Notemos que P (1) vlida, pois


13 + 23 + 33 = 1 + 8 + 27 = 36 = 9 4.

Precisamos provar agora o passo indutivo, isto ,


Hiptese: P (k) verdadeira para algum k N.
Tese: P (k + 1) tambm verdadeira.

Para provar isto, observamos que

(k + 1)3 + (k + 2)3 + (k + 3)3 = (k + 1)3 + (k + 2)3 + (k 3 + 9k 2 + 27k + 27).

Ordenando adequadamente, temos que o lado direito da ltima igualdade


se escreve como
k 3 + (k + 1)3 + (k + 2)3 + (9k 2 + 27k + 27)
= k 3 + (k + 1)3 + (k + 2)3 + 9(k 2 + 3k + 3)
{z
} |
{z
}
|
mltiplo de 9

= mltiplo de 9,

completando assim nossa demonstrao.

mltiplo de 9

214

Induo Matemtica

Muitas vezes, para conseguir mostrar que a hiptese p(n + 1) verdadeira, precisamos supor que p(k) verdadeira para todo n0 k n. Isto
a base do princpio forte da induo nita que enunciamos a seguir:

Teorema 6.10 (Princpio Forte da Induo Finita). Considere n0 um inteiro no negativo. Suponhamos que, para cada inteiro n n0 seja dada
uma proposio p(n) e que valem as propriedades
(a) p(n0 ) verdadeira;
(b) se para cada inteiro no negativo k, com n0 k n, temos que p(k)
verdadeira, ento p(n + 1) tambm verdadeira.
Ento, a proposio p(n) verdadeira para qualquer n n0 .
Utilizando o princpio forte da induo, vamos dar uma prova diferente
do teorema fundamental da aritmtica da apresentada no Captulo 3.

Exemplo 6.11 (Teorema Fundamental da Aritmtica). Todo nmero natural N maior que 1 pode ser escrito como um produto
N = p1 p2 p3 pm ,

(6.3)

onde m 1 um nmero natural e os pi , 1 i m so nmeros primos.


Alm disso, a fatorao em (6.3) nica se exigirmos que p1 p2
pm .
Soluo. Para cada n N, n 2, denamos a proposio
p(n) : n escrito de modo nico como um produto de nmeros primos.

Notemos que p(2) verdadeira, pois 2 um nmero primo.


Agora enunciemos o passo indutivo:
Hiptese indutiva: p(k) verdade para cada inteiro k tal que 2 k
n.

6.3

Induo na Geometria

215

Tese: p(n + 1) verdade. Em outras palavras, temos que mostrar que


n + 1 escrito de modo nico como um produto de nmeros primos.

Faremos a prova dividindo em dois casos:


(a) Se n + 1 um nmero primo, ento p(n + 1) verdade e isto acaba
nossa demonstrao.
(b) Se n + 1 no um nmero primo, ento existem , N com 2
n e 2 n tais que n + 1 = .
Nossa hiptese indutiva vlida para e . Isto signica que se
escreve de modo nico como um produto de nmeros primos e que
se escreve de modo nico um produto de nmeros primos. Portanto,
n + 1 = se escreve como um produto de nmeros primos.

Agora mostraremos que n + 1 se escreve de modo nico como produto


de primos. Assuma que
p1 p2 . . . pk = q1 q2 . . . qm = n + 1,

(6.4)

com p1 p2 pk e q1 q2 qm todos primos. Vamos


mostrar que necessariamente k = m e pi = qi .
De fato, como p1 primo, ele divide algum qi . Logo, como qi primo,
p1 = qi q1 . Analogamente, existe um j tal que q1 = pj p1 . Logo,
p1 = q1 . Cancelando p1 em ambos os lados da equao (6.4), temos
que (n + 1)/p1 = p2 . . . pk = q2 . . . qm n. Logo, por hiptese de
induo, k = m e p2 = q2 , . . . , pm = qm , encerrando a demonstrao.

6.3

Induo na Geometria

Tratamos aqui alguns exemplos que mostram a utilidade do mtodo de


induo na resoluo de problemas geomtricos. Vamos comear estudando

216

Induo Matemtica

duas propriedades importantes dos polgonos. A primeira delas trata da


soma dos ngulos internos de um polgono convexo de n lados (n-gono).
Um polgono convexo um polgono tal que qualquer segmento de reta
que liga dois de seus pontos est contido no interior dele. No caso de
polgonos, isto equivalente ao fato de que todo segmento que liga dois
vrtices ou uma aresta ou est contido no interior do polgono.

Exemplo 6.12. Mostre que a soma dos ngulos internos de um polgono

convexo de n lados (n 3) igual a (n 2) radianos.

Soluo. No caso de n = 3 a propriedade acima muito bem conhecida.


Desde Tales de Mileto e Euclides se conhecia que a soma dos ngulos internos
de um tringulo radianos. Faamos mais um caso, tomando n = 4. Neste
caso, podemos dividir um quadriltero em dois tringulos, como mostra a
Figura 6.1 (a). Assim, a soma dos ngulos internos de um quadriltero
2 radianos.
A4

A4

A3
A5

A1

A2
(a)

A3

A1

A2
(b)

Figura 6.1: Dividindo polgonos


Para elucidar o processo de induo e no deixar dvidas sobre o que
iremos fazer, vamos considerar mais um polgono, o pentgono (n = 5).
Neste caso, para mostrar que a soma dos seus ngulos internos (5 2) =
3 radianos, iremos dividir o pentgono A1 A2 A3 A4 A5 em um quadriltero
A1 A2 A3 A4 e um tringulo A1 A4 A5 , como mostra a Figura 6.1 (b). Assim,

6.3

Induo na Geometria

217

a soma dos ngulos internos do pentgono A1 A2 A3 A4 A5 igual soma dos


ngulos internos do tringulo A1 A4 A5 (igual a ) mais a soma dos ngulos
internos do quadriltero A1 A2 A3 A4 (igual a 2 ), ou seja, igual a 3 .
Finalmente, vamos assumir como hiptese de induo que para um certo
n 3 mostramos que a soma dos ngulos internos do n-gono dada pela
expresso (n 2) . Precisamos mostrar que a soma dos ngulos internos
de um n + 1-gono [(n + 1) 2] = (n 1) . De fato, podemos repetir
o processo anterior. Vamos denominar de A1 , A2 , . . . , An , An+1 os vrtices
consecutivos do (n + 1)-gono. Podemos dividi-lo no n-gono A1 A2 . . . An e
no tringulo A1 An+1 An . Logo, a soma dos ngulos internos do (n+1)-gono
(n 2) + = (n 1) .

Exemplo 6.13. Mostre que o nmero de diagonais de um polgono convexo

de n-lados igual a

n(n3)

Soluo. Observe que para n = 3 temos que existem 0 = 3.(3 3)/2 diagonais num tringulo. Para n = 4, temos 2 = 4(4 3)/2 diagonais num
quadriltero convexo (veja a Figura 6.2).
Vamos agora assumir como hiptese de induo que se n um ngono convexo ento o seu nmero de diagonais n(n 3)/2 e vamos provar que a frmula vale para um (n + 1)-gono convexo. De fato, denote
por A1 , A2 , . . . , An , An+1 os vrtices consecutivos do n + 1-gono. Podemos decomp-lo como a unio do n-gono A1 , A2 , . . . , An e do tringulo
A1 , An , An+1 . Neste caso, para contarmos as diagonais do (n + 1)-gono
devemos considerar os seguintes casos:
Diagonais do n-gono A1 , A2 , . . . , An ; por hiptese de induo, o nmero dessas diagonais n(n 3)/2.
n 2 diagonais que partem do vrtice An+1 mais a diagonal A1 An .

Assim, o nmero total de diagonais do (n + 1)-gono

n2 3n + 2n 2
n2 n 2
(n + 1)(n 2)
n(n 3)
+ (n 2) + 1 =
=
=
,
2
2
2
2

218

Induo Matemtica

como queramos demonstrar.


A4

A4

A3
A5

A1

A2

A3

A1

A2

(a)

(b)

Figura 6.2: Diagonais de polgonos

Exemplo 6.14. Mostre que podemos cobrir os

n2 pontos no reticulado a

seguir traando 2n 2 segmentos de reta sem tirar o lpis do papel.

{z

nnpontos

Figura 6.3: O problema de bar n n

6.3

Induo na Geometria

219

Soluo. O caso n = 3 j foi enunciado no Problema 1.12 do Captulo 1. A


gura a seguir mostra a soluo, onde o caminho realizado com as 4 linhas
o seguinte: A B C D B .
C
A

Figura 6.4: Soluo do problema de bar 3 3


Daremos a prova do problema acima por induo. Para isso, veja que
podemos resolver o caso n = 4 continuando a soluo do caso n = 3. Como
paramos num dos vrtices do quadrado 33, acrescentamos mais uma linha
e uma coluna para obter um reticulado 4 4. Assim, conseguimos cobrir os
16 pontos utilizando 4 + 2 = 6 linhas, sem tirar o lpis do papel e cobrindo
dois lados do quadrado, como mostram as linhas descontnuas na Figura
6.5.

Figura 6.5: Completando o reticulado

220

Induo Matemtica

Finalmente, vamos assumir como hiptese de induo que podemos cobrir n 2 um reticulado n n com 2n 2 linhas, sendo que a ltima delas
cobre um dos lados do reticulado. Acrescentando 2n+1 pontos como mostra
a Figura 6.5, obtemos um reticulado (n + 1) (n + 1) que pode ser coberto
com 2n 2 + 2 = 2(n + 1) 2 pontos, como queramos demonstrar.
6.4

Miscelnea

Nesta seo discutiremos alguns exemplos interessantes de como podemos


aplicar o mtodo da induo aos mais variados tipos de problemas. O
primeiro deles uma generalizao do Problema 1.8.

Exemplo 6.15 (A Moeda Falsa). Um rei muito rico possui 3n moedas de

ouro. Porm, uma destas moedas falsa e seu peso menor que o peso das
demais. Com uma balana de 2 pratos e sem nenhum peso, mostre que
possvel encontrar a moeda falsa com apenas n pesagens.

Soluo. Para resolver este problema, vamos utilizar o Mtodo da Induo.


De fato, se n = 1, procederemos da seguinte forma: pegamos duas moedas
quaisquer e colocamos na balana, deixando uma do lado de fora. Caso a
balana se equilibre, a moeda que est do lado de fora necessariamente a
que tem menor peso. Caso a balana se desequilibre, a que tem menor peso
est na balana, no prato mais alto. O caso n = 2 foi feito no Problema
1.8.
Vamos agora assumir como hiptese de induo que dadas 3n moedas,
podemos achar a moeda mais leve com n pesagens. Vamos mostrar que
para 3n+1 moedas, suciente n + 1 pesagens. De fato, dividiremos as
3n+1 moedas em 3 grupos, A, B e C com 3n moedas cada. Colocamos na
balana os grupos A e B . Caso os dois grupos se equilibrem, a moeda mais
leve est no grupo C . Caso o grupo A esteja mais leve, a moeda mais leve se
encontra no grupo A. De qualquer modo, com uma pesagem conseguimos

6.4

Miscelnea

221

determinar em qual grupo de 3n elementos a moeda mais leve se encontra.


Por hiptese de induo, precisamos de mais n pesagens para encontrar a
moeda mais leve, totalizando n + 1 pesagens. Desaamos o leitor a mostrar
que no possvel realizar tal tarefa com menos de n pesagens.

Exemplo 6.16. Mostre que utilizando um balde com 5 litros de capacidade


e outro com 7 litros, possvel separar qualquer quantidade superior ou igual
a 24 litros.

Soluo. Novamente, faremos a prova utilizando o Mtodo da Induo.


Neste caso, comearemos o processo de induo a partir de 24. De fato,
podemos separar 24 litros utilizando duas vezes o balde de 7 e duas vezes o
balde de 5 litros. Note que o problema acima equivale a mostrar que
Todo nmero maior ou igual a 24 pode ser escrito da forma
7x + 5y , onde x e y so nmeros inteiros maiores ou iguais a
zero.
Neste caso, escrevemos 24 como 24 = 2 7 + 2 5. Por hiptese de
induo, vamos supor que conseguimos escrever um nmero n 24 como
n = 7x + 5y , com x e y nmeros inteiros maiores ou iguais a zero. Devemos
mostrar que n + 1 se escreve deste modo tambm. Para isso, vamos dividir
a anlise em dois casos:

Caso 1:

y3

Logo, x 2 pois se isso no ocorresse, teramos 7x + 5y 22 < 24, o


que impossvel. Assim, podemos escrever:
n + 1 = 7x + 5y + 1 = 7(x 2) + 5(y + 3),

pois x 2 0.

Caso 2:

y4

222

Induo Matemtica

Neste caso, y 4 0. Logo, podemos escrever:


n + 1 = 7x + 5y + 1 = 7(x + 3) + 5(y 4),

nalizando a nossa prova por induo.


6.4.1

Cuidados ao Usar o Princpio da Induo

Observao 6.17. Quando aplicamos o princpio da induo devemos tomar certos cuidados. A seguir damos um exemplo de como o mtodo pode
ser aplicado de forma errada. Vamos mostrar a seguinte armao:

Armao: Num conjunto qualquer de n bolas, todas as bolas

possuem a mesma cor.

Observe que nossa proposio claramente falsa. Mas, mesmo assim, vamos
dar uma prova por induo.
Para n = 1, nossa proposio verdadeira pois em qualquer conjunto
com uma bola, todas as bolas tm a mesma cor, pois s existe uma bola. Assuma por hiptese de induo que a proposio verdadeira para n e provemos que a proposio verdadeira para n+1. Ora, seja A = {b1 , . . . , bn , bn+1 }
o conjunto com n + 1 bolas referido. Considere os subconjuntos de B e C
de A com n elementos, construdos como:
B = {b1 , b2 , . . . , bn } e C = {b2 , . . . , bn+1 }

Observe que ambos os conjuntos tm n elementos. Assim, as bolas


b1 , b2 , . . . , bn do conjunto B tm a mesma cor. Do mesmo modo, as bolas do conjunto C tm a mesma cor. Em particular, a bola bn tem a mesma
cor da bola bn+1 . Assim, todas as bolas tm a mesma cor. Ache o erro no
argumento! Se voc no conseguir, leia a nota de rodap. 1
1 Uma

dica da soluo encontra-se no nal do captulo.

6.5

Induo e Recorrncias

6.5

223

Induo e Recorrncias

Vamos comear esta seo discutindo um problema muito conhecido e interessante.

Exemplo 6.18 (As Torres de Hani2 ). Diz uma antiga lenda que na origem

dos tempos, em um templo de Hani, foram colocados 64 discos perfurados


de ouro puro e de dimetros diferentes ao redor de uma de trs hastes de
diamante. Muitos sacerdotes moviam os discos, respeitando as seguintes
regras: eles comeam empilhados em ordem crescente de acordo com seu
tamanho (ver Figura 6.6). Os discos podem ser deslocados de uma coluna
para qualquer outra, sendo que nunca pode ser colocado um disco maior em
cima de um menor e a cada segundo os sacerdotes movem um disco.
Quando os sacerdotes transportassem todos os discos de uma coluna para
outra, o mundo se acabaria. Suponha que eles comearam esse processo no
ano 2000 e que a lenda verdadeira, quanto tempo ainda resta para a Terra?

Figura 6.6: Torre de Hani


Para responder esse problema, consideraremos o problema geral de descobrir quantos movimentos so necessrios para mover n anis de uma haste
para outra. Argumentaremos do seguinte modo: observe que podemos mover
os discos para outra haste se n = 1 ou 2. Com efeito, se temos somente um
anel basta mover este para qualquer outra haste com um nico movimento.
2 Este

jogo foi inventado, em 1882, pelo matemtico Francs douard Lucas.

224

Induo Matemtica

Se temos 2 anis ento movemos o menor deles para a segunda haste, o


maior para a terceira haste e, nalmente, o menor para a terceira haste,
realizando um total de 3 movimentos. Para calcular o caso geral, vamos
empregar um mtodo chamado de mtodo recursivo: o nmero ak+1 de movimentos necessrios para mover k + 1 anis ser expresso como uma funo
de ak .
De fato, se temos k + 1 anis na primeira haste e sabemos mover k anis
de uma haste para outra utilizando ak movimentos, ento podemos mover
todos os k + 1 anis para a segunda haste usando 2ak + 1 movimentos.
De fato, movemos todos eles, exceto o maior, para a terceira haste usando
ak movimentos. A seguir, colocamos o maior na segunda haste usando 1
movimento. Imediatamente, deslocamos todos os anis da terceira haste
para a segunda haste usando mais ak movimentos. Logo, movemos todos os
k + 1 anis utilizando 2ak + 1 movimentos. Em resumo:
ak+1 = 2ak + 1,

(6.5)

onde ak o nmero de movimentos necessrios para mover k discos de uma


haste para outra. Vamos agora usar induo para provar que ak = 2k 1.
Uma vez constatada a veracidade da armao para k = 1, 2, para
calcular ak , por hiptese de induo, vamos assumir que ak = 2k 1. Temos
pela equao (6.5):
ak+1 = 2ak + 1 = 2(2k 1) 1 = 2k+1 1.

como queramos demonstrar.


Vamos aproveitar o Exemplo 6.18 para discutir algumas equaes que
aparecem em muitas situaes em Matemtica: as equaes de recorrncia.
Em geral, uma equao de recorrncia uma equao envolvendo uma
certa quantidade de termos de sequncia xn . Para ilustrar isso, observe
a equao (6.5). Aqui, estaremos interessados em um tipo particular de
equao de recorrncia, as equaes de recorrncia lineares.

6.5

Induo e Recorrncias

225

Denio 6.19. Uma equao de recorrncia linear de grau

k uma ex-

presso da forma:

xn+1 =rk1 xn + rk2 xn1 + + r0 xnk+1


x1 = a1 , x2 = a2 , . . . , xk = ak ,

(6.6)

onde r0 , r1 , . . . , rk1 so nmeros reais e r0 6= 0.


Por exemplo, so equaes de recorrncia lineares as seguintes equaes
2xn 3xn+1 = 0

2
3xn + xn+1 = 5xn+2
3

e no so equaes de recorrncia lineares as equaes


2(xn )3 5xn+1 = 0

2
3xn + xn+1 = 5xn+2 + 3.
3

Exemplo 6.20 (Sequncia de Fibonacci). Um exemplo muito interessante

de equao de recorrncia a sequncia conhecida por sequncia de Fibonacci, devido ao matemtico italiano Leonardo di Pisa (1170-1250). Esta
sequncia adquiriu muita fama devido a suas conexes com reas das mais
variadas na cultura humana. Ela aparece em problemas de Biologia, Arquitetura, Engenharia, Fsica, Qumica e muitos outras reas da cincia e
arte.
Denimos a sequncia de Fibonacci como sendo a sequncia Fn que
satisfaz a seguinte equao de recorrncia:
F1 = 1;
F2 = 1;
Fn = Fn1 + Fn2 , se n 3.

Agora vamos utilizar induo para mostrar algumas de suas propriedades.

226

Induo Matemtica

Exemplo 6.21. Considere Fn a sequncia de Fibonacci. Mostre que


 n
7
.
Fn <
4

n

Soluo. Denamos a proposio p(n) := Fn < 47 . Para n = 1 temos


que F1 = 1 < 74 , de modo que p(1) verdadeira. Suponhamos que
p(1), p(2), . . . , p(n), n 2,

sejam todas verdadeiras. Mostraremos que Fn+1 <

n1
+ 47
n1
n1
< 74 74
+ 74
 n1
.
< 1 + 74 74

Fn+1 = Fn + Fn1 <

Como 1 +

7
4

<


7 2
4 ,


7 n
4


7 n+1
.
4

segue-se que Fn+1 <


Fn+1 <


7 n+1
.
4


7 2
4


7 n1
.
4

Com efeito,

Portanto,

Exemplo 6.22. Dada a seguinte relao de recorrncia


a0 = 8;
a1 = 10;
an = 4an1 3an2 ,

n 2.

Mostre que an = 7 + 3n , para todo n Z+ .


Soluo. Denamos a proposio P (n) : an = 7 + 3n . P (0) verdadeira,
pois P (0) = 7 + 30 = 7 + 1 = 8. Suponhamos que P (k) verdadeiro para
cada inteiro k tal que 1 k n. Vamos mostrar que P (k) verdade para

6.5

Induo e Recorrncias

227

k = n + 1. Com efeito,
an+1 = 4an 3an1

= 4(7 + 3n ) 3(7 + 3n1 )

= 7 + 4 3n 3 3n1

= 7 + 3n1 4 3 3

= 7 + 3n1 9 = 7 + 3n1 32
= 7 + 3n+1 .

Vamos agora discutir o caso geral da equao de recorrncia linear (6.6).


Para isso, vamos fazer algumas observaes preliminares que deixaremos a
cargo do leitor:
se an e bn so solues da equao (6.6), ento an + bn tambm

soluo;

se an soluo da equao (6.6) e um nmero real, ento an

tambm soluo.

Com isto em mente, vamos descrever agora como obter todas as solues
xn da equao (6.6) em funo de n. Observe que dados os termos iniciais
a1 , a2 , . . . , ak a sequencia xn ca inteiramente determinada pela equao de
recorrncia. O interessante aqui determinar o termo xn+1 sem que seja
preciso o clculo dos termos xn , xn1 , . . . , xnk+1 .
Vamos primeiro procurar o que se chama de soluo particular da equao (6.6). Particular porque ela assume uma forma caracterstica e porque
no assumiremos que as condies x1 = a1 , . . . , xk = ak valham.
Vamos procurar solues do tipo xn = n , onde um nmero real
positivo. Neste caso, temos que:
n+1 = xn+1 =rk1 xn + rk2 xn1 + + r0 xnk+1

= rk1 n + rk2 n1 + + r0 nk+1 .

228

Induo Matemtica

Passando os termos do lado direito da igualdade e colocando em evidncia o termo nk+1 temos:

nk+1 k rk1 k1 rk2 k2 r0 = 0.

(6.7)

k rk1 k1 rk2 k2 r0 = 0.

(6.8)

Assim, como k 6= 0, pois > 0, temos que

O polinmio
p() = k rk1 k1 rk2 k2 r0

recebe o nome especial de polinmio caracterstico da equao de recorrncia (6.6). Acabamos de mostrar que qualquer raiz do polinmio caracterstico gera uma soluo particular da equao (6.6).
Vamos assumir que a equao (6.8) possui k razes diferentes, digamos
1 > 2 > > k . Ento vale o seguinte teorema:

Teorema 6.23. Se escolhermos nmeros reais c1 , c2 , . . . , ck , ento


xn = c1 n1 + c2 n2 + + ck nk

(6.9)

uma soluo da equao de recorrncia, onde os termos iniciais ai para


i = 1, 2, . . . , k so:
ai = c1 i1 + c2 i2 + + ck ik .

Demonstrao. Para mostrar o teorema, como x1 = a1 , . . . xk = ak pela


denio dos ai 's, basta mostrar que xn uma soluo.
Ora, o produto de uma soluo por um nmero real tambm uma
soluo. Assim, como ni uma soluo para i = 1, 2, . . . , k e ci um

6.6

Exerccios

229

nmero real, temos que ci ni soluo para i = 1, 2, . . . , k. Como j vimos


acima, a soma de solues tambm uma soluo. Logo,
xn = c1 n1 + c2 n2 + + ck nk

uma soluo.
Neste ponto, voltamos a equao (6.6). Desde o princpio, dados os
nmeros ai buscvamos a soluo xn tal que x1 = a1 , . . . , xk = ak . A
Equao (6.9) nos d uma variedade de solues, onde podemos escolher
os nmeros ci como bem entendermos. Usando equaes lineares, podemos
mostrar que sempre possvel escolher os nmeros ci de modo que x1 =
a1 , . . . , xk = ak . Isso encerra nossa busca. Para complementar esta seo,
recomendamos a leitura do Captulo 3 de [4].

6.6

Exerccios

1. Se qn denota a soma qn = 12 + 22 + + n2 , prove que para todo


nN

qn =

n(n + 1)(2n + 1)
.
6

2. Use o princpio da induo para provar as seguintes armaes:


(a) 3n+1 + 2n+2 divisvel por 7 para todo n N;
(b) a soma dos cubos de trs nmeros naturais consecutivos divisvel por 9;
(c) 7 + 77 + 777 + + 777
. . . 7} = 7(10n+1 9n 10)/81;
| {z
nvezes

(d) (n + 1)(n + 2) . . . (n + n) = 2n 1 3 5 (2n 1).

3. Use o princpio da induo para provar as seguintes desigualdades:

230

(a) 2n1 (an + bn ) > (a + b)n ,


a 6= b;

Induo Matemtica

n N, com a, b R, a + b > 0 e

1
1
1
1
n, para todo n N;
n
1
2
3
1
1
1
1
13
(c)
+
+
+ +
> , para todo n N.
n+1 n+2 n+3
2n
24

(b) + + + + >

4. Mostre a seguinte identidade trigonomtrica


cos x + 2 cos 2x + + n cos nx =

(n + 1) cos nx n cos(n + 1)x 1


.
4 sin2 x2

5. Um torneio de xadrez tem n jogadores. Cada jogador joga uma nica


partida com cada um dos outros jogadores. Calcule o nmero total
de partidas realizadas no torneio.
6. Demonstre que para qualquer n N vlida a igualdade


n(n + 1)
1 + 2 + 3 + + n =
2
3

2

7. Demonstre que para qualquer n N valida desigualdade




1 n
an = 1 +
< 3.
n

8. Prove que, para todo n N e a > 0,


s

a+

1 + 4a + 1
a + a + + a <
.
2
{z
}
nradicais

9. Mostre que para qualquer nmero natural n 0, 11n+2 + 122n+1


sempre divisvel por 133.
10. Mostre que para todo n Z+ temos que 32n+1 + 2n+2 um mltiplo
de 7.

6.6

Exerccios

231

11. Mostre que para todo n Z+ temos que 32n+2 + 26n+1 um mltiplo
de 11.
12. Considere Fn a sequncia de Fibonacci . Mostre que
1
Fn =
5

!n
1
1+ 5

2
5

!n
1 5
.
2

13. Mostre as seguintes propriedades a respeito da sequncia de Fibonacci


Fn :
(a)
(c)

n
X

i=1
n
X
i=1

Fi = Fn+2 1;
F2i = F2n+1 1;

(b)

n
X

F2i1 = F2n ;

i=1

(d) Fn1 Fn+1 Fn2 = (1)n .

14. De quantas formas diferentes podemos cobrir um tabuleiro de 2


n com peas de domins que cobrem exatamente duas celas do tabuleiro?
15. Calcular o nmero de regies em que o plano dividido por n retas
distintas em cada uma das seguintes situaes:
(a) as n retas so concorrentes;
(b) no existem duas retas paralelas nem trs retas concorrentes.3
16. Dizemos que uma gura enquadrvel com rgua e compasso, se a
partir dela possvel, utilizando apenas rgua e compasso, construir
um quadrado de mesma rea. Prove que:
3 At

onde sabemos, este problema conhecido como a pizza de Steiner, o qual


foi resolvido, em 1826, pelo notvel gemetra Jacob Steiner (1796-1863).

232

Induo Matemtica

(a) um tringulo sempre enquadrvel;


(b) um polgono qualquer enquadrvel.

Sugesto para o item (b): Utilize induo dividindo a gura em tringulos.


17. D uma resposta situao Observao 6.17.

Sugesto: Observe a validade do argumento quando o conjunto A tem


2 elementos. Veja que B e C no se intersectam. Ou seja, o passo
indutivo falha de n = 1 para n = 2.

Referncias Bibliogrcas
[1] AIGNER, M. e ZIEGLER, G. (2002). As Provas esto no
Livro. Edgard Blcher.
[2] GARCIA, A. e LEQUAIN, I. (2003). Elementos de lgebra.
Projeto Euclides, IMPA.
[3] LIMA, E. L.; CARVALHO, P. C. P.; WAGNER, E. e MORGADO, A.C. (2004). A Matemtica do Ensino Mdio. Volume
1. Sociedade Brasileira de Matemtica.
[4] LIMA, E.L.; CARVALHO, P. C. P.; WAGNER, E. e MORGADO, A.C. (2004). A Matemtica do Ensino Mdio. Volume
2. Sociedade Brasileira de Matemtica.
[5] LIMA,E.L.; CARVALHO,P. C. P.; WAGNER,E. e MORGADO,A.C. (2004). A Matemtica do Ensino Mdio. Volume
3. Sociedade Brasileira de Matemtica.
[6] LIMA, E.L.; CARVALHO, P. C. P.; WAGNER,E. e MORGADO, A.C. (2001). Temas e Problemas. Sociedade Brasileira
de Matemtica.
[7] LIMA, E.L. (2001). lgebra Linear. Sociedade Brasileira de
Matemtica.

285

286

REFERNCIAS BIBLIOGRFICAS

[8] MORAIS FILHO, D. C. (2007). Um Convite Matemtica.


EDUFCG.
[9] MORGADO, A.; CARVALHO, J.; CARVALHO, P.; FERNANDEZ, P. (1991). Anlise Combinatria e Probabilidade .
Sociedade Brasileira de Matemtica.
[10] RIBENBOIM, P. (2001). Nmeros Primos: Mistrios e Recordes. Sociedade Brasileira de Matemtica.
[11] SANTOS, J. P. O. (1993) Introduo Teoria dos Nmeros.
IMPA.
[12] SANTOS, J. P. O.; MELLO, M. P. e MURARI, I. T. C.
(2006). Introduo Anlise Combinatria. Editora Unicamp.
[13] SOARES, M. G. (2005). Clculo em uma Varivel Complexa.
Sociedade Brasileira de Matemtica.

Mestrado Profissional

em Matemtica em Rede Nacional

Iniciao Matemtica

Autores:
Krerley Oliveira

Adn J. Corcho

Unidade IV:
Captulos VII e VIII

7
Desigualdades
Existem duas formas de fazer tima Matemti a. A primeira ser
mais esperto que todo mundo. A segunda ser mais estpido que
todo mundo  mas persistente.
Raoul Bott

Neste captulo estudaremos algumas desigualdades clssicas que


so usadas frequentemente na resoluo de problemas matemticos,
sendo estas aplicadas em contextos que variam desde o nvel mais
simples at o mais complexo.
Uma vez que uma inequao em uma ou mais variveis resolvida,
o resultado d lugar a uma desigualdade que vlida para um certo
conjunto de valores. Alguns exemplos simples de desigualdades so os
seguintes:
(a)

x |x|,

para qualquer

(b)

x2 < x,

se

(c)

(x y)2 0,

(d)

x
y

y
x

2,

1 < x < 1;

x < 1;
para quaisquer

para quaisquer

reais;

x, y > 0.
233

234

7.1

Desigualdades

Desigualdade Triangular

A desigualdade triangular arma o seguinte

Teorema 7.1 (Desigualdade Triangular) . Dado um tringulo ABC o

comprimento de um dos lados sempre inferior soma dos comprimentos dos outros dois lados, ou seja,
AB < AC + CB,

AC < AB + BC

e BC < BA + AC.

Figura 7.1:

Desigualdade Triangular

Em outras palavras, a desigualdade triangular a formulao matemtica da ideia intuitiva de que o caminho reto mais curto entre
os pontos A e B.
Em analogia com a geometria plana temos uma verso da desigualdade triangular para nmeros reais, que provamos a seguir.

Proposio 7.2. Sejam a e b nmeros reais quaisquer, ento


|a + b| |a| + |b|.

Demonstrao.
contrrio, se

a + b 0, ento |a + b| = a + b |a| + |b|.


a + b < 0, ento |a + b| = a b |a| + |b|.
Se

Caso

7.1

Desigualdade Triangular

235

Corolrio 7.3. As seguintes desigualdades valem


|a b| |a| + |b|

(7.1)

|a b| |a| |b|,


|a b| |a| |b|

Demonstrao.

(7.2)
(7.3)

|a b| = |a + (b)|
desigualdade decorre de |a| =

Para a primeira, escrevemos

|a| + | b| = |a| + |b|. A segunda


|b + (a b)| |b| + |a b|. A ltima desigualdade
segunda, trocando os papis de a e b.

consequncia da

D
C
A

O
P
B

Figura 7.2:

Problema da central de energia

Exemplo 7.4. Quatro cidades rurais, A, B , C e D, esto situadas


geogracamente formando um quadriltero convexo. Deseja-se construir uma central de distribuio de energia para as quatro cidades de
modo que a soma total das distncias da central a cada uma das quatro
cidades seja a mnima possvel. Onde dever ser construda a central?
Soluo.
no ponto

Mostraremos que a central de energia dever ser colocada

de interseco das diagonais do polgono

ABCD.

Com

236

efeito, considerando um ponto

P,

diferente de

O,

Desigualdades

(veja Figura 7.2) a

desigualdade triangular nos garante que

OA + OC = AC < P A + P C
e

OB + OD = BP < P B + P D,
de onde se segue que

OA + OC + OB + OD < P A + P C + P B + P D,
como espervamos.

Exemplo 7.5. Duas torres de alturas h1 e h2 , respectivamente, esto

separadas a uma distncia d. As torres so amarradas por uma corda


AP B que vai do topo A da primeira torre para um ponto P no cho,
entre as torres, e ento at o topo B da segunda torre, como na Figura
7.3. Qual a posio do ponto P que nos d o comprimento mnimo da
corda a ser utilizada?
A
B

P
Figura 7.3:

Problema das Torres

Desigualdade Triangular

7.1

Soluo.

237

Imaginemos que a superfcie do cho um espelho e que re-

etimos o ponto atravs deste, obtendo assim o ponto

B0

como mostra

a Figura 7.4.

A
B

C
P

B
Figura 7.4:

Soluo geomtrica do problema das torres

Consideremos o segmento

AB 0

que intercepta o cho no ponto

e para nossa surpresa vericaremos que este o ponto que nos d o


comprimento mnimo das cordas. Com efeito, suponhamos que existe
outro

P0

situado entre as torres que nos d um comprimento menor

BP D
B0P 0D

para a corda, ento da Figura 7.4 fcil ver que os tringulos


e

B PD

so congruentes, assim como os tringulos

BP D

tambm so congruentes. Logo, as seguintes igualdades seguem diretamente das congruncias:

BP = B 0 P

BP 0 = B 0 P 0 .

Agora, usando a desigualdade triangular no tringulo

AB 0 P 0 e as igual-

dades acima, temos que

AP 0 + P 0 B = AP 0 + P 0 B 0 AB 0 = AP + P B 0 = AP + P B,

238

chegando assim concluso de que

AP + P B

Desigualdades

nos oferece o compri-

mento mnimo desejado.


Agora calcularemos a que distncia est
que

AC = h1 , BD = h2

CD = d

tang(]BP D)

Da tem-se

7.2

PD =

dh2
.
h1 + h2

da base

D.

Lembremos

e observamos que

h1
h2
=
.
PD
d PD

Desigualdade das Mdias

Denio 7.6. Sejam a1 , a2 , . . . , an1 e an nmeros reais positivos.

As quantidades

mh (a1 , a2 , . . . , an ) =

n
,
1/a1 + 1/a2 + + 1/an

mg (a1 , a2 , . . . , an ) =

ma (a1 , a2 , . . . , an ) =

a1 + a2 + + an
,
n

a1 a2 an ,

r
mq (a1 , a2 , . . . , an ) =

a21 + a22 + + a2n


n

(7.4)

(7.5)

(7.6)

(7.7)

so chamadas, respectivamente, de mdia harmnica, mdia geomtrica, mdia aritmtica e mdia quadrtica dos nmeros ai , i = 1, 2, . . . , n.
A seguir provaremos alguns resultados que estabelecem relaes de
desigualdades entre as mdias denidas acima.

7.2

Desigualdade das Mdias

239

Proposio 7.7 (Desigualdade das Mdias Aritmtica e Quadrtica) .


Dados a1 , a2 , . . . , an nmeros reais positivos tem-se
a1 + a2 + + an

a21 + a22 + + a2n


,
n

ou seja, ma (a1 , a2 , . . . , an ) mq (a1 , a2 , . . . , an ). Alm disso, a igualdade vale se, e somente se, a1 = a2 = = an .
Demonstrao.

Usando a igualdade

X
1i<jn

(ai aj ) = (n 1)

n
X
i=1

a2i 2

ai aj

(7.8)

1i<jn

conclumos que,

X
1i<jn

ai aj (n 1)

n
X

a2i ,

(7.9)

i=1

dado que o termo da esquerda em (7.8) no negativo. Somando em


n
P
ambos os membros de (7.9) a quantidade
a2i obtemos
i=1

n
n
X
2
X
ai n
a2i ,
i=1

donde, dividindo por

n2

i=1

e tomando a raiz quadrada, segue-se a desi-

gualdade desejada. Por ltimo, observamos que a igualdade em (7.9)


atingida se, e somente se,

(ai aj )2 = 0,

o que verdade se,

1i<jn
e somente se,

a1 = a2 = = an .

Proposio 7.8 (Desigualdade das mdias Geomtrica e Aritmtica) .


Dados a1 , a2 , . . . , an nmeros reais positivos tem-se

a1 + a2 + + an
n
a1 a2 an
,
n

240

Desigualdades

ou seja, mg (a1 , a2 , . . . , an ) ma (a1 , a2 , . . . , an ). Alm disso, a igualdade vale se, e somente se, a1 = a2 = = an .
Demonstrao.

A prova desta desigualdade mais tcnica e exige um

pouco mais de esforo. Dividiremos a mesma em dois passos.

Passo 1.

n = 2m .
Para n = 2

A desigualdade vale para

Procederemos por induo.

a desigualdade vale. De

fato,

( a1 a2 )2 = a1 + a2 2 a1 a2 0.

a +a
Assim, a1 + a2 2 a1 a2 e conseqentemente 1 2
a1 a2 .
2
Agora provamos que se a desigualdade vale para n = k ,
tambm vale para n = 2k . Com efeito,

ento

a1 ++ak
2k
+ ak+1 ++a
a1 + + a2k
k
k
=
2k
2

(1) k a1 ak + k ak+1 a2k

2
(2) q

k
a1 ak k ak+1 a2k

= 2k a1 a2k ,
onde em (1) e (2) usamos a validade da desigualdade em para

n=k

n = 2, respectivamente. Logo, como j provamos a validade


n = 2, claro que vale tambm para n = 4, 8, . . . , 2m , . . . , como

e para
para

espervamos.

Passo 2.
todo

Dado

n<2

inteiro positivo, ento a desigualdade vale para

Para vericar isto, denimos o nmero

L=

n
a1 an ,

Desigualdade das Mdias

7.2

e como a desigualdade vale para

241

n = 2m ,

a1 + + an + L
+ L}
| + {z
2m n vezes

2m
Portanto,

temos ento que

2m

p
a1 an L2m n

2m
=
Ln L2m n = L.

a1 + + an + (2m n)L
L,
2m

logo

a1 + + an 2m L (2m n)L = nL,


obtendo assim que

a1 + + an nL = n n a1 an ,
o que nos d a desigualdade desejada.
Como para qualquer inteiro positivo
positivo

tal que

n<2

sempre existe um inteiro

n.
a1 = a2 = = an

, a desigualdade ca provada para todo

A prova de que a igualdade s ocorre quando

pode tambm ser feita por induo e deixamos a cargo do leitor.

Proposio 7.9 (Desigualdade das Mdias Harmnica e Geomtrica) .


Dados a1 , a2 , . . . , an nmeros reais positivos tem-se

n
n a1 a2 an ,
1/a1 + 1/a2 + + 1/an

ou seja, mh (a1 , a2 , . . . , an ) mg (a1 , a2 , . . . , an ). Alm disso, a igualdade vale se, e somente se, a1 = a2 = = an .
Demonstrao.

Usando a Proposio 7.8 com os nmeros

ai

substitu-

1/ai (i = 1, 2 . . . , n) vale que


n
n
Y
1 1/n
1X 1
= mg (1/a1 , . . . , 1/an ) ma (1/a1 , . . . , 1/an ) =
.
a
n i=1 ai
i=1 i

dos por

242

Desigualdades

Invertendo esta ltima desigualdade, obtemos ento

mh (a1 , a2 , . . . , an ) mg (a1 , a2 , . . . , an ),
concluindo-se assim a prova. Notemos que as igualdades s ocorrem

1/a1 = 1/a2 = = 1/an


an .
se

equivalem as igualdades

a1 = a2 = =

O prximo resultado resume as relaes provadas, nas proposies


7.7, 7.8 e 7.9, para as mdias

Teorema 7.10

mh , mg , ma

(Desigualdade das Mdias)

mq .

. Para toda coleo de n-

meros reais positivos a1 , a2 , . . . , an1 e an se vericam as seguintes


desigualdades:
min(a1 , . . . , an ) mh (a1 , a2 , . . . , an )
mg (a1 , a2 , . . . , an )

(7.10)

ma (a1 , a2 , . . . , an )

mq (a1 , a2 , . . . , an ) max(a1 , . . . , an ).

Alm disso, em cada caso a igualdade ocorre se, e somente se, a1 =


a2 = = an .

Exemplo 7.11. Num tringulo retngulo a altura relativa hipote-

nusa sempre menor ou igual que a metade da hipotenusa. Alm


disso, a igualdade s ocorre quando o tringulo retngulo issceles
(ou seja, seus catetos so iguais).
Soluo.

Usando a Figura 7.5, temos que a hipotenusa

c = x+y

e usando o teorema das alturas para um tringulo retngulo

dada por

Desigualdade das Mdias

7.2

temos que

h2 = xy ,

logo

243

h=

xy.

A desigualdade entre as mdias

geomtrica e aritmtica nos d que

h=

xy

x+y
c
= ,
2
2

como queramos. Alm disso, a altura a metade da hipotenusa se, e

h
y

Interpretao geomtrica da desigualdade das mdias geomtrica e aritmtica


Figura 7.5:

somente se, a igualdade entre as mdias ocorre, ou seja, quando


Ento, os catetos

x = y.

a e b do tringulo so iguais, sendo este issceles.

Exemplo 7.12 (Desigualdade Isoperimtrica para Tringulos) . O pe-

rmetro de um tringulo de lados a, b e c a soma p = a + b + c.


Entre todos os tringulos com permetro xado p o de maior rea o
tringulo equiltero.
Soluo.

Usando a Frmula de Hern temos que a rea de um trin-

gulo com permetro

dada pela expresso

A=
onde

a, b

p p
(
2 2

a)( p2 b)( p2 c),

so os lados do tringulo.

Usando agora a desigualdade

s 
p
A

p
2

mg ma

3
p
p
a+ b+ c
2
2
3

temos que,

p2
.
12 3

244

Logo a maior rea possvel

p
2

a=

p
2

2
p
, a qual atingida quando
12 3

b=

p
2

c a = b = c,

ou seja, quando o tringulo equiltero.

2
p
a2 3
=
.
4
12 3

Exemplo 7.13

Desigualdades

Notemos que neste caso,

(Desigualdade Isoperimtrica para Paraleleppedos)

Entre todos os paraleleppedos com rea lateral xada A o de maior


volume o cubo (ou seja, o paraleleppedo com todos seus lados iguais).
c

b
a

A rea lateral de um paraleleppedo de lados a, b e c dada


por AL = 2(ab + bc + ac).

Figura 7.6:

Soluo.

Denotando por

a, b

as medidas das arestas do paralele-

ppedo sabemos que a soma das reas de todas as faces do paraleleppedo, ou seja,

AL = 2(ab + ac + bc).
Sendo

o volume do paraleleppedo e usando a desigualdade entre as

mdias aritmtica e geomtrica temos que

3  3
AL
ab + ac + bc
=
.
V = ab ac bc
3
6
q

AL 3
Assim, o maior volume possvel V =
, obtido
6
ac = bc, consequentemente a = b = c.
2

(7.11)

quando

ab =

Desigualdade de Cauchy-Schwarz

7.3

7.3

245

Desigualdade de Cauchy-Schwarz

Teorema 7.14

(Desigualdade de Cauchy-Schwarz)

. Dados a1 , . . . ,

an e b1 , . . . , bn nmeros reais tem-se


q
q
2
2
|a1 b1 + + an bn | x1 + + an b21 + + b2n

(7.12)

Alm disso, a igualdade s ocorre se existir um nmero real , tal que


a1 = b1 , . . . , an = bn ou b1 = a1 , . . . , bn = an .
Demonstrao.
n
X
i=1

Usando a identidade de Lagrange:

a2i

n
X

b2i

n
X

i=1

temos que

ai b i

2

i=1

n
X

ai b i

(ai bj aj bi )2

1i<jn

2

i=1

n
X
i=1

a2i

n
X

b2i ,

i=1

de onde se obtm diretamente a desigualdade de Cauchy-Schwarz.


Alm disso, a igualdade ocorre se, e somente se,

(ai bj aj bi )2 = 0 ai bj aj bi = 0, 1 i < j n,

1i<jn

o que verdade se, e somente se, existe


com

tal que ai = bi

ou

bi = ai ,

i = 1, 2, . . . , n.

Exemplo 7.15. Entre todos os tringulos retngulos de catetos a e b

e hipotenusa c xada, o que tem maior soma dos catetos s = a + b


o tringulo issceles.
Soluo.

Usando a desigualdade de Cauchy-Schwarz temos que

a+b=a1+b1

a2 + b2 12 + 12 = c 2

246

Desigualdades

a = 1 e b = 1
devemos ter a = b.

e este mximo atingido quando

1 = b.

Em qualquer caso

Exemplo 7.16

(Desigualdade de Minkowski)

ou

1 = a

. Dados ai , bi com 1

i n, nmeros reais, tem-se


v
v
v
u n
u n
u n
X
X
u
u
uX
2
t (ai + bi )2 t
ai + t
b2i .
i=1

Soluo.

i=1

i=1

Partimos da seguinte igualdade:

n
X

(ai + bi )2 =

i=1

n
X

a2i +

i=1

n
X

b2i + 2

i=1

n
X

ai b i .

(7.13)

i=1

Aplicando a desigualdade de Cauchy-Schwarz no lado direito de (7.13)


temos que

n
X
i=1

(ai + bi )2

n
X

a2i +

i=1

n
X

v
v
u n
u n
uX uX
2
2
at
b2
b + 2t
i

i=1

i=1

v
v
2
u n
u n
uX
uX
= t
a2 + t
b2 .
i

i=1

i=1
(7.14)

i=1

Tomando raiz quadrada em ambos os membros de (7.14) obtemos a


desigualdade de Minkowski.

7.4

Desigualdade de Jensen

A Desigualdade de Jensen est estreitamente relacionada com o conceito de convexidade, o qual explicamos a seguir.

Desigualdade de Jensen

7.4

247

Denio 7.17. Uma funo f : [, ] R dita convexa se para


quaisquer a, b [, ] e para todo [0, 1] satisfaz


f a + (1 )b f (a) + (1 )f (b).

y
y = f (x)
(b, f (b))
(a, f (a))

a
Figura 7.7:

Grco de uma funo convexa

Geometricamente, a denio de convexidade signica que para


cada par de pontos

escolhidos no intervalo

[, ]

o grco da

funo encontra-se abaixo do segmento de reta secante que junta os


pontos

(a, f (a))

(b, f (b)),

como mostra a Figura 7.7.

Exemplo 7.18. A funo f (x) = x2 convexa em qualquer intervalo

[, ].

Soluo.

Sejam

a, b [, ] e suponhamos, sem perda de generalidade,

248

que

a < b.

Ento, para todo

[0, 1]

Desigualdades

valem as desigualdades:

(a + (1 )b)2 = 2 a2 + (1 )2 b2 + 2(1 )ab


(1)

2 a2 + (1 )2 b2 + (1 )(a2 + b2 )

(7.15)

= a2 [2 + (1 )] + b2 [(1 )2 + (1 )]
= a2 + (1 )b2 ,

onde na passagem (1) usamos a desigualdade

ab

a2 +b2
.
2

Exemplo 7.19. A funo f (x) = 1/x convexa em qualquer intervalo

[, ] com positivo.

Soluo.

Sendo

a, b [, ]

com

a < b,

para todo

[0, 1]

tem-se

1 = ( + (1 ))2

= 2 + 2(1 ) + (1 )2
a b 
(1)
2
+
(1 ) + (1 )2
+
b a
a
b
= 2 + (1 ) + (1 ) + (1 )2
b
a
 (1 ) 
 (1 ) 
= a
+
+(1 )b
+
a
b
a
b

 (1 ) 
= a + (1 )b
+
a
b

onde na passagem (1) usamos que


meros positivos

b.

a/b + b/a 2

para quaisquer n-

De (7.16) segue-se que

1
1
1
+ (1 ) ,
a + (1 )b
a
b
mostrando isto a convexidade da funo

1
.
x

(7.16)

7.4

Desigualdade de Jensen

249

Observemos que, usando a desigualdade entre as mdias aritmtica


e quadrtica obtemos

a1 + a2 + + an
n

2

a21 + a22 + + a2n


,
n

em outras palavras

(ma (a1 , a2 , . . . , an ))2 ma (a21 , a22 , . . . , a2n ).

(7.17)

Por outro lado, a desigualdade entre as mdias harmnica e aritmtica


nos garantem que

1
ma (1/a1 , 1/a2 . . . , 1/an ).
ma (a1 , a2 , . . . , an )

(7.18)

O seguinte resultado garante que as propriedades (7.17) e (7.18), satisfeitas pelas funes convexas

x2

1
, so vlidas para qualquer funo
x

convexa.

Teorema 7.20

(Desigualdade de Jensen)

. Seja f : [, ] R uma

funo convexa e sejam i [0, 1] (i = 1, . . . , n) tais que


Ento, para quaisquer ai [, ] (i = 1, . . . , n) vale

n
X

i = 1.

i=0

f (1 a1 + + n an ) 1 f (a1 ) + + n f (an ).

(7.19)

Observao 7.21. Observemos que, quando 1 = 2 = = n =


1/n, a desigualdade de Jensen nos diz que

 a + a + + a  f (a ) + f (a ) + + f (a )
1
2
n
1
2
n
f

,
n
n

ou seja, f (ma (a1 , . . . , an )) ma (f (a1 ), . . . , f (an )).

250

Demonstrao.

Faremos a prova por induo. Para

decorre diretamente da denio.

Desigualdades

n=2

Suponhamos que dado

a validade

natural

(7.19) vale, ento temos que provar a validade de

n+1
X
j=1

 n+1
X
j aj
j f (aj ).

(7.20)

j=i

Notemos que

n+1
X

j aj =

n
X

j=1

j aj + 1

j=1
n
X

j=1
onde

n
P

n
X

j an+1

j=1

j
aj + (1 )an+1 ,

j . Assim, usando que

j=1

n
P
j
j=i

(7.21)

= 1 e a hiptese de induo,

obtemos

n+1
X
j=1

j aj f

n
X

j=1

n
X
j
j=1

n+1
X


aj +(1 )f (an+1 )

f (aj ) + (1 )f (an+1 )

(7.22)

j f (aj ),

j=1
como queramos provar.

7.5

Exerccios

1. Provar que em todo tringulo a soma dos comprimentos das


medianas menor que o permetro do tringulo e maior que o
semipermetro deste.

7.5

Exerccios

251

2. Os centros de trs crculos que no se intersectam esto sobre


uma reta. Prove que se um quarto crculo toca de forma tangente
os trs crculos, ento o raio deste maior que pelo menos um
dos raios dos trs crculos dados.

3. Dado

inteiro positivo, provar que

n
X
1
j=1

4. A soma de trs nmeros positivos


seus quadrados no menor que

6.

2n
.
n+1

Provar que a soma de

12.

5. Determinar as dimenses do paraleleppedo de menor diagonal


possvel, sabendo que a soma dos comprimentos de todas suas
arestas

12.

6. Encontrar todas as solues positivas do sistema de equaes


no lineares

x 2 + + x 2 = 1
1
10
1
1
2 + + 2 = 100.
x1

7. Demonstrar que, se

x10

a1 , a2 , . . . , an

so nmeros positivos tais que

a1 a2 an = 1
ento

(1 + a1 )(1 + a2 ) (1 + an ) 2n .
8. Prove que a mdia geomtrica super-aditiva, isto , para n-

bi , 1 i n, tem-se
v
v
v
u n
u n
u n
uY
uY
uY
n
n
n
t
ai + t
bi t
(ai + bi ).

meros no negativos

i=1

ai

i=1

i=1

252

Desigualdades

Alm disso, estude em que condies ocorre a igualdade.


Sugesto: Use a desigualdade entre as mdias geomtrica e aritmtica.

9. Usar o mtodo de induo para provar a desigualdade de CauchySchwarz.


10. Para todo

real

n
P

(ai + bi )2 0.

Use este fato para dar outra


i=1
prova da desigualdade de Cauchy-Schwarz.

11. Use a desigualdade de Cauchy-Schwarz para dar uma prova alternativa da desigualdade entre as mdias aritmtica e quadrtica
(ma

mq ).

12. Prove que

n
X
i=1
13. Prove que

1
ai b i
2

( n
X

a2i +

i=1

n
X

)
b2i

i=1

a4 + b4 + c4 abc(a + b + c).

14. Prove que se

a 0, b 0

c 0,

ento

(a + b)(a + c)(b + c) 8abc.


15. Prove a desigualdade de Bernoulli:
qualquer

positivo e

16. Prove que se

a, b, c

n
d

(1 + x)n > 1 + nx,

inteiro positivo.

so inteiros positivos, ento:


(a + b + c + d)


16.

c 0, ento

(ab + bc + ca) a bc + b ac + c ab.

17. Prove que se

a 0, b 0

1 1 1 1
+ + +
a b c d

para

7.5

Exerccios

253

18. Prove que se

x 0,

ento

3x3 6x2 + 4 0.

Sugesto: Use a desigualdade entre as mdias aritmtica e geomtrica.

19. Prove que se

C1

20. Sejam

x 0,

C2

ento

2x + 3/8 4 x.

dois crculos concntricos de raios

pectivamente, com

r1 < r2 .

Sobre o crculo

d(P ) = P P1 + P P2 .

r2 ,

res-

se marcam dois

P1 e P2 diametralmente opostos. Deseja-se


P sobre o crculo C2 que maximiza a soma

pontos
ponto

C1

r1

encontrar o

254

Desigualdades

8
Polinmios
A oisa mais bela que podemos ontemplar o mistrio. Isto a
fonte da verdadeira arte e in ia.
Albert Einstein

8.1

Operaes com Polinmios

A necessidade de estudar equaes polinomiais aparece em problemas


prticos da humanidade desde pocas muito remotas. Indcios arqueolgicos indicam que os babilnicos j tinha o domnio de tcnicas de
resoluo de algumas equaes do primeiro grau e do segundo grau,
apresentadas em forma de problemas cotidianos. Contudo, o grande
avano terico no estudo das equaes polinomiais s se iniciou com o
Renascimento na Europa. No incio do sculo XVI, Viti introduziu o
uso de letras para representar quantidades desconhecidas.
Na mesma poca, um outro grande desao estava perturbando as
mentes matemticas de toda a Europa, em especial as da Itlia.

soluo explcita utilizando as operaes elementares (soma, subtrao, multiplicao, diviso, radiciao e potenciao) da equao do

255

256

Polinmios

terceiro grau no era conhecida e muitos dos melhores matemticos


da poca trabalharam neste problema, destacando-se entre eles Nicolo Fontana, o

Tartaglia

(gago, em italiano). A histria da soluo

desta equao est repleta de intrigas, disputas e acusaes, envolvendo Tartaglia e Cardano. Hoje os historiadores atribuem a Tartaglia a primazia na descoberta da soluo da equao do terceiro grau
como conhecemos.

desta poca tambm a soluo da equao do

quarto grau, atribuda a Ludovico Ferrari.


Entretanto, apesar dos muitos esforos empreendidos na direo de
encontrar a soluo geral da equao do quinto grau, mais de 200 anos
se passaram sem nenhum sucesso.

At que em 1824, o matemtico

noruegus Niels Abel mostrou que

impossvel

resolver as equaes de

grau cinco em sua forma geral. Ou seja, nem todas as equaes de grau
cinco podem ser resolvidas com as operaes elementares. Mais ainda,
em 1830 o matemtico francs Evariste Galois descobriu um mtodo
que determina quando uma equao de grau

qualquer

resolvel com

as operaes elementares, encerrando um belssimo captulo do estudo


das equaes polinomiais e da Matemtica.
Neste captulo iremos estudar um pouco mais formalmente os polinmios e suas propriedades.

Denio 8.1. Um polinmio na varivel x uma expresso do tipo


p(x) = an xn + an1 xn1 + + a1 x + a0

onde a0 , a1 , . . . , an so nmeros. Se an 6= 0, dizemos que n o grau


do polinmio e a0 , a1 , . . . , an so seus coecientes. O coeciente an
chamado de coeciente lder do polinmio.

Observao 8.2. No se dene o grau do polinmio nulo, que tem

todos os coecientes iguais a zero.

Operaes com Polinmios

8.1

257

Por exemplo,

p(x) = 3x 1

um polinmio de grau 1;

q(x) = 4x3 + 7x + 1

um polinmio de grau 3;

4
x um monmio de grau 4;
2

v(x) = x4 + 5x2 + 1 um polinmio


2
t(x) =

u(x) = 7

um polinmio de grau

de grau 4;

0.

n, ou simplesmente uma equao


de grau n, uma sentena p(x) = 0, onde p(x) um polinmio de
grau n com coecientes reais. Por exemplo, 2x 1 = 0 uma equao
5
3
do primeiro grau, enquanto x + 4x + 5x 1 = 0 uma equao de
grau 5. Note que nem todos os coecientes precisam ser diferentes de
Uma equao polinomial de grau

zero.
Para obtermos o

+ a1 x + a0

valor

p(x) = an xn + an1 xn1 +


r, devemos substituir x por r para obter

do polinmio

no nmero real

o nmero real

p(r) = an rn + an1 rn1 + + a1 r + a0 .


Por exemplo, o valor do polinmio

4 23 7 2 + 1 = 19.

Dizemos que um nmero real

p(x) = 4x3 7x + 1

em

p(2) =

uma raiz para a equao

an xn + an1 xn1 + + a1 x + a0 = 0
se o valor de
seja, se

p(x) = an xn + an1 xn1 + + a1 x + a0

verica

an rn + an1 rn1 + + a1 r + a0 = 0.

em

zero, ou

258

Por exemplo,

Polinmios

raiz da equao:

2x 10 = 0.
Uma das vantagens dos polinmios sobre outros objetos matemticos que podemos denir as operaes de soma de polinmios e
multiplicao de polinmios. Com estas operaes, o conjunto dos polinmios possui muitas propriedades similares dos nmeros inteiros,
tornando prtico o seu uso.
Vamos denir agora o que signica a

soma

de dois polinmios.

Para isso, vamos comear somando dois monmios e depois estender


nossa denio para polinmios em geral.

p(x) = ak xk e q(x) =
bk xk somamos seus coecientes, obtendo o polinmio t(x) = p(x) +
q(x) = (ak +bk )xk . Em geral, para somar o polinmio p(x) = a0 +a1 x+
a2 x2 + + an xn com o polinmio q(x) = b0 + b1 x + + bm xm , onde
n m devemos somar todos os monmios de mesmo grau, obtendo o
Para somar dois monmios de mesmo grau

polinmio:

t(x) = p(x) + q(x) = c0 + c1 x + + cm xm


onde,

c i = ai + b i

para

Por exemplo, sendo

0in

p(x) = 3x 1,
q(x) = 4x3 + 7x + 1,
t(x) = 2 x4 ,
v(x) = 2 x4 + 5x2 + 1

ci = b i

para

i > n.

Operaes com Polinmios

8.1

259

temos que

p(x) + q(x) = 4x3 + (3 + 7)x 1 + 1 = 4x3 + 10x,


v(x) + t(x) = ( 2 2 )x4 + 5x2 + 1 = 5x2 + 1.
A seguir, enumeramos algumas propriedades simples e importantes
da soma de polinmios que decorrem da denio dada e das propriedades anlogas vlidas para os nmeros reais.
1.

Associatividade .

Dados polinmios

p(x), q(x)

t(x),

vale

(p(x) + q(x)) + t(x) = p(x) + (q(x) + t(x))


2.

Elemento neutro .

Se 0 denota o polinmio nulo e

p(x)

um

polinmio qualquer, ento

0 + p(x) = p(x).
3.

Elemento simtrico .
polinmio, ento o
satisfaz:

p(x) = a0 + a1 x + + an xn um
n
polinmio q(x) = a0 a1 x an x
Se

p(x) + q(x) = 0.
4.

Comutatividade .

Se

p(x)

q(x)

so polinmios, ento

p(x) + q(x) = q(x) + p(x).


Note que os nmeros inteiros possuem propriedades similares para
a operao de soma de nmeros inteiros. Vamos agora denir o

produto

de dois polinmios. Para isso, vamos primeiramente denir o produto


de dois monmios, como j zemos no caso de soma de polinmios.

260

n, m so nmeros naturais, denimos


p(x) = an xn e q(x) = bm xm como:
Se

Polinmios

o produto dos monmios

p(x)q(x) = an bm xn+m .
Tendo isto em mente, para efetuarmos o produto do polinmio de

n, p(x) = a0 + a1 x + a2 x2 + + an xn pelo polinmio q(x) =


b0 + b1 x + + bm xm de grau m, com n m, devemos:

grau

p(x) e de q(x) at o termo n + m


k > n e bk = 0 para k > m;

Completamos a escrita de
colocando

ak = 0

para

Denimos

t(x) = p(x)q(x) = c0 + c1 x + + cn+m xn+m


onde,

ci = a0 bi + a1 bi1 + + ai1 b1 + ai b0

para

0 i n + m.

Apesar de parecer complicada, a denio no to difcil de ser


aplicada. Para tentar visualizar o processo de multiplicao de dois
polinmios vamos pensar que os monmios so seres aliengenas vindos do distante planeta de Algebrum e possuam mos. Quando dois
monmios se encontram, invariavelmente eles apertam as mos e desse
aperto aparece o produto desses monmios.
Assim, para multiplicar os polinmios

p(x)

q(x),

que so forma-

dos por dois grupos de monmios, devemos escolher o primeiro monmio de

q(x),

p(x)

e faz-lo apertar a mo de cada um dos monmios de

somando os monmios obtidos. Aps isso, tomamos o segundo

monmio de

p(x)

e fazemos ele apertar a mo de cada um dos mon-

q(x), somando os monmios obtidos aos monmios anteriores.


Repetimos o processo at o ltimo monmio de p(x).
mios de

Operaes com Polinmios

8.1

Deste modo, se
obter

p(x)q(x)

261

p(x) = x2 + 2x 3

fazemos:

q(x) = x2 + 5x + 1,

para

p(x)q(x) = x4 + 5x3 + x2 2x3 + 10x2 + 2x + 3x2 15x 3


= x4 + 3x3 + 14x2 13x 3.
Observe que com a denio de multiplicao de polinmios dada
acima, o coeciente

c0

igual a

a0 b 0 .

Do mesmo modo, o coeciente

cn+m = an bm . Como p(x) tem grau n (isto , an 6= 0)


q(x) tem grau m (bm 6= 0), o coeciente cn+m = an bm 6= 0. Logo,
polinmio p(x)q(x) tem grau n + m. Com isso, demonstramos o

do termo
e

n+m

seguinte fato:

Proposio 8.3. Se o polinmio p(x) tem grau n e o polinmio q(x)


tem grau m, ento o polinmio p(x)q(x) tem grau n + m.

Um caso particular interessante quando multiplicamos um nme-

c, que podemos considerar como sendo um polinmio de grau zero


q(x) = c, por um polinmio p(x) = a0 + a1 x + + an xn . Neste caso,
ro

ns obtemos o polinmio

cp(x) = ca0 + ca1 x + + can xn .


Do mesmo modo em que podemos vericar as propriedades da
soma de polinmios a partir das propriedades similares dos nmeros
reais, podemos tambm vericar as propriedades abaixo sobre a multiplicao de polinmios. Deixamos essa vericao como exerccio.

1.

Associatividade .

Dados polinmios

p(x), q(x)

(p(x)q(x))t(x) = p(x)(q(x)t(x))

t(x),

vale

262

2.

Elemento neutro .

Polinmios

Se 1 denota o polinmio constante e

p(x)

um polinmio qualquer, ento

1p(x) = p(x).
3.

Comutatividade .

Se

p(x)

q(x)

so polinmios, ento

p(x)q(x) = q(x)p(x).
4.

Distributividade .

Se

p(x), q(x)

t(x)

so polinmios, ento

(p(x) + q(x))t(x) = q(x)t(x) + p(x)t(x).


Note que, assim como nos inteiros, a propriedade de existncia de
elementos inversos para a multiplicao de polinmios no vale. De

p(x) um polinmio de grau n maior ou


igual a um, ento no existe um polinmio q(x) tal que p(x)q(x) = 1.
De fato, suponha por absurdo, que exista q(x) um polinmio com grau
m 0 tal que
fato, podemos vericar que se

p(x)q(x) = 1.

Ento, utilizando a Proposio 8.3 temos que o grau de

n+m

que maior ou igual que um.

constante

p(x)q(x)

Como o grau do polinmio

zero, temos que a igualdade acima no pode valer, onde

chegamos a um absurdo.
Em resumo, os nicos polinmios que podem ter inversos com respeito operao de multiplicao so os polinmios constantes no
nulos. Esta mais uma das semelhanas entre os inteiros e os polinmios.

Algoritmo de Euclides

8.2

8.2

263

Algoritmo de Euclides

Diremos que um polinmio


tal que

a(x) divide o polinmio b(x) se existir q(x)

b(x) = q(x)a(x).

Por exemplo, o polinmio

x3 1

a(x) = x2 + x + 1

divide o polinmio

pois

(x 1)(x2 + x + 1) = x3 1.
a(x)

Devido Proposio 8.3, se o polinmio


no nulo

b(x),

ento o grau de

a(x)

divide o polinmio

menor ou igual ao grau de

b(x).

Agora, vamos enunciar um fato que vale para os inteiros e que vale
tambm para os polinmios e que ser de grande utilidade. Pedimos
que o leitor releia o

algoritmo de Euclides ,

estudado no Captulo 3.

No conjunto dos polinmios, ainda vale

Teorema 8.4 (Algoritmo de Euclides) . Sejam a(x) e b(x) dois polin-

mios com coecientes reais, b(x) 6= 0. Ento, existem polinmios com


coecientes reais q(x) e r(x), com r(x) = 0 ou grau de r(x) menor
que o grau de b(x) tais que:
a(x) = b(x)q(x) + r(x).

Alm disso, q(x) e r(x) esto determinados de modo nico.


Demonstrao.

Vamos mostrar primeiro a unicidade. De fato, assuma

que

a(x) = b(x)q1 (x) + r1 (x) = b(x)q2 (x) + r2 (x),


com

r1

r2

de graus menores que o grau de

b.

b(q1 q2 ) = r2 r1 .

Assim,

264

Consequentemente,

q2 )

q1 = q2 ,

b(q1
r2 r1 tem

j que caso contrrio, o polinmio

teria grau pelo menos igual ao grau de

grau menor que o grau de

Polinmios

e o polinmio

b.

Vamos agora mostrar a existncia. Os passos da prova so idnticos


a prova do algoritmo de Euclides para nmeros inteiros, demonstrado
no Captulo 3.

De fato, a ideia reduzir o grau do dividendo at

que ele se torne menor que o do divisor e a diviso se torne imediata.

a tem grau menor que b, ento tomamos o resto com


sendo r = a e o quociente como sendo q = 0. Suponhamos que
a(x) = an xn + +a1 x+a0 tenha grau n e b(x) = bm xm + +b1 x+b0
tenha grau m e que n > m. Dena
Note que se

c1 (x) = a(x)

an nm
x
b(x).
bm

c1 no mximo n 1. Se c1 puder se dividido


c1 (x) = b(x)q(x) + r(x), com grau de r(x) menor

Observe que o grau de


por

b,

digamos com

que o grau de

b(x),

a(x) = b(x)

ento

an nm
an
x
+ c1 (x) = b(x)( xnm + q(x)) + r(x).
bm
bm

a(x) por b(x) pelo


problema de dividir o polinmio c1 (x) por b(x), com c1 (x) de grau
menor que a(x). Repetimos o processo, utilizando c1 no lugar de a(x),
obtendo o polinmio c2 (x) de grau menor que o de c1 (x). Como a cada
Logo, reduzimos o problema de dividir o polinmio

passo reduzimos o grau do dividendo em pelo menos uma unidade, ao

n m passos, obteremos um polinmio com grau


menor que o grau de b(x), que claramente divisvel por b(x). Procedendo como antes, achamos q(x) e r(x) tais que a(x) = b(x)q(x)+r(x)
e r(x) com grau menor que o grau de b(x).
m de no mximo

8.2

Algoritmo de Euclides

265

a(x) = 10x3 3x + 2
r(x) = 13x + 2 temos que

Por exemplo, se

q(x) = 10x

b(x) = x2 + 1,

tomando

10x3 3x + 2 = (x2 + 1)10x + (13x + 2).


Note que o grau de

x2 + 1 .

r(x) = 13x + 2

Se na expresso do polinmio

por um nmero real

s,

p(x)

estaremos

menor que o grau de

b(x) =

decidimos substituir a varivel

avaliando

p(s).
p(x) = x + 3x + 1,

o polinmio

p(x)

em

e denotamos este nmero por


Por exemplo, se

ento substituindo

por

2,

temos que

p(2) = 22 + 3 2 + 1 = 11
e fazendo

x = 3
p(3) = (3)2 + 3 (3) + 1 = 1.

p(s) = 0 dizemos que s anula o polinmio no nulo p(x),


ou ainda, que s uma raiz do polinmio p(x).
3
Por exemplo, para p(x) = x 8, temos que 2 uma raiz de p(x)
3
j que p(2) = 2 8 = 0.
Quando

Um fato muito importante que consequncia do algoritmo de

Euclides o seguinte teorema:

Teorema 8.5. Se s uma raiz do polinmio p(x), ento o polinmio

x s divide p(x). Reciprocamente, se x s divide p(x), ento s raiz


de p(x).

Demonstrao.

x s divida p(x). Neste


p(x) = q(x)(x s). Avaliando

Primeiramente, assuma que

caso, existe um polinmio

q(x)

tal que

266

Polinmios

o polinmio

p(x)

s,

em

temos que:

p(s) = q(s)(s s) = q(s) 0 = 0.


Logo

p(x).
se s uma

uma raiz de

p(x) ento x s divide p(x),


vamos utilizar o algoritmo da diviso, com a(x) = p(x) e b(x) = x s.
Neste caso, temos que existem q(x) e r(x) de modo que r(x) = 0 ou o
grau de r(x) menor que o grau de x s e alm disso vale
Para provar que

raiz de

p(x) = q(x)(x s) + r(x).


r(x), podemos escrever que
r(x) = c R. Ento, p(x) = q(x)(xs)+c e 0 = p(s) = q(s)0+c = c.
Portanto, r(x) = 0 e p(x) = q(x)(x s), isto , x s divide p(x).
Observe que, com as condies do resto

A proposio anterior nos permite determinar o nmero mximo


de razes reais de um polinmio no nulo. De fato, vamos mostrar.

Proposio 8.6. O nmero mximo de razes reais do polinmio no


nulo p(x) = an xn + an1 xn1 + + a1 x + a0 n.
Demonstrao.

Digamos que

distintas do polinmio

p(x).

s0 < s1 < s2 < < sk

sejam razes

Observe que podemos utilizar a Propo-

sio 8.5 para garantir que existe um polinmio no nulo

q1 (x)

tal

que

p(x) = q1 (x)(x s0 ).
Assim, pela Proposio 8.3, o grau de

q1 (x)

deve ser igual a

n 1.

p(si ) = q1 (si )(si s0 ). Como para todo i = 1, 2, . . . , k temos


que si > s0 com p(si ) = 0, temos que, necessariamente, q1 (si ) = 0.
Assim, em particular, temos que q1 (s1 ) = 0. Logo, podemos aplicar
Note que

Algoritmo de Euclides

8.2

267

a proposio novamente para obter que existe um polinmio no-nulo

q2 (x)

tal que

q1 (x) = q2 (x)(x s1 ).
q1 (x) n 1, pela Proposio 8.3, o grau de
q2 (x) deve ser igual a n 2.
Novamente, temos que q1 (si ) = q2 (si )(si s1 ), si > s1 e p(si ) = 0
para todo i = 2, . . . , k . Disto segue que, necessariamente, q2 (si ) = 0,
se i = 2, 3, . . . , k. Assim, temos que q2 (s2 ) = 0.
Assim, como o grau de

Logo, podemos repetir esse argumento para obter um polinmio

q3 (x) de grau n 3, de modo que s3 , s4 , . . . , sk

q3 (x). Repetindo o argumento, encontramos uma sequncia q1 (x), q2 (x), q3 (x), . . .


com graus no mximo n 1, n 2, n 3, . . . o que nos leva a concluir que no podemos repetir esse argumento mais que n vezes, j
que os graus dos polinmios q1 (x), q2 (x), q3 (x), . . . esto diminuindo.
Ou seja, no podemos ter mais que n razes para o polinmio p(x), o
so razes de

que conclui a prova.

Alertamos que, apesar da Proposio 8.6 nos garantir que existem

n razes reais de um polinmio de grau n no nulo, existem


2
polinmios que no possuem razes reais. Por exemplo, p(x) = x + 1
2
no possui razes rais, j que x 0 para todo nmero real x.

no mximo

Uma consequncia da Proposio 8.6 a seguinte:

Proposio 8.7. Se dois polinmios p(x) e q(x) de grau n avaliados

em n + 1 nmeros r1 , r2 , . . . , rn+1 coincidem, isto , p(ri ) = q(ri ) para


i = 1, 2, 3, . . . , n + 1, ento p(x) e q(x) so iguais.

Demonstrao.
que se

t(x)

t(x) = p(x) q(x).


t(x) no mximo n, j

Considere o polinmio

no-nulo, o grau de

Observe
que

p(x)

268

Polinmios

q(x) tm graus
p(ri ) = q(ri ) e
e

iguais a

n.

Observe ainda que

t(ri ) = 0,

j que

t(ri ) = p(ri ) q(ri ) = 0.


Logo,

t(x)

tem grau no mximo

e mais de

razes, contradizendo a

Proposio 8.6.
No Exerccio 23 faremos uma aplicao interessante dessa propo-

a1 , a2 , . . . ,
de grau n tal

sio, propondo que voc prove que dados nmeros reais

an+1 e r1 , r2 , . . . , rn+1 ,
que p(ri ) = ai .

ento existe um nico polinmio

8.3 Sempre Existem Razes de um Polinmio?


Pode parecer frustrante o fato de que um polinmio com coecientes
reais pode no possuir razes reais.

Por exemplo, quando tentamos

aplicar a frmula de Bhaskara equao

= 4

encontramos

e, consequentemente, se fosse possvel escrever as solues,

elas se escreveriam como

x1 =
e

x2 + 1 = 0,

4
2

4
x2 =
2
claro que as expresses acima no tm sentido no conjunto dos

nmeros reais, pois no existe nmero cujo quadrado seja


no possvel extrair a raiz quadrada de

4.

4, ou seja,

Isso tirou o sono de

vrias geraes de matemticos. Desde Hern de Alexandria h dois


mil anos atrs, os matemticos encontram expresses como a do tipo
acima, envolvendo razes de nmeros negativos.

Sempre Existem Razes de um Polinmio?

8.3

269

A primeira reao da comunidade matemtica foi rejeitar esses


nmeros

complexos

e simplesmente desconsiderar razes de nmeros

negativos. Porm, j no sculo XVI, Cardano se deu conta de que os


nmeros complexos surgem naturalmente quando desejamos resolver
uma equao do terceiro ou quarto grau, mas relutava quanto ao seu
uso, dizendo que esses nmeros eram to sutis, quanto inteis.
No sculo seguinte, motivado pela sugesto de Albert Girard que
uma equao de grau

n possui n razes, Rene Descartes observou que

os nmeros reais eram insucientes para representar todas essas razes


e utilizou o termo

imaginrias

A notao tradicional

para as razes que no so reais.

i = 1 s veio a ser introduzida um sculo

mais tarde, com Leonard Euler, que tambm o pai do termo

complexo.

nmero

Euler e o matemtico francs Jean D'Alambert zeram apli-

caes dos nmeros complexos a problemas prticos, como projeo


de mapas e hidrodinmica.

Euler e Lagrange, grandes matemticos

da histria da humanidade, tentaram mostrar a armao de Girard,


de que

uma equao de grau n possui n razes,

mas sem sucesso. A

primeira prova correta de tal teorema s apareceu no nal do sculo


XVIII com os trabalhos de Gauss.

8.3.1 Nmeros Complexos e Razes de Polinmios


O conjunto dos nmeros complexos, denotado pela letra

C,

o con-

junto das expresses

C = {x + iy; x, y R},
onde
que

satisfaz

i2 = 1.

Costuma-se denotar

por

1.

Destacamos

meramente um smbolo que nos ajudar a denir as operaes

de soma e de multiplicao de nmeros complexos. Essas operaes

270

Polinmios

tero as mesmas propriedades que as operaes de nmeros reais, como


associatividade, comutatividade, elemento neutro, etc. Por exemplo,
so nmeros complexos

2 3i, 3 + i

3i.

Vamos denir a soma e multiplicao de nmeros complexos. Dados dois nmeros complexos

a + bi

c + di

denimos a soma como:

(a + bi) + (c + di) = (a + b) + (c + d)i


e denimos a multiplicao como

(a + bi)(c + di) = (ac bd) + (bc + ad)i


Por exemplo, se tomamos os nmeros

2 3i

3 + 4i

ento

(2 3i) + (3 + 4i) = 5 + i
e

(2 3i)(3 + 4i) = (2 3 (3 4)) + (3 3 + 2 4)i = 18 i.


Aqui ns estamos considerando
nos permite

colocar

0 + 3i = 3i

3 + 0 i = 3.

Isso

os nmeros reais dentro do conjunto dos nmeros

complexos, considerando cada nmero real


complexo da forma

como sendo um nmero

r + 0 i.

Fica para o leitor a vericao de que valem as propriedades de


associatividade, comutatividade, etc. O elemento neutro da soma o
elemento

0+0i

que simplesmente denotaremos por

modo, o elemento neutro da multiplicao


por

1.

0.

Do mesmo

1+0i, que ser denotado

O leitor curioso pode achar mais informaes sobre nmeros

complexos e solues de equaes algbricas em [5] ou [13].

8.3

Sempre Existem Razes de um Polinmio?

271

z faz sentido avaliar o polinmio


p(x) = an xn + an1 xn1 + +

Assim, dado um nmero complexo


(de coecientes complexos ou reais)

a1 x + a0

z,

em

obtendo o nmero complexo

p(z) = an z n + an1 z n1 + + a1 z + a0 .
Por exemplo, se
polinmio, j que:

p(x) = x2 + 4,

ento

2i

2i

so razes deste

p(2i) = (2i)2 + 4 = 4 + 4 = 0.
e

p(2i) = (2i)2 + 4 = 4i2 + 4 = 4 + 4 = 0.


Note que

p(x) no possui nenhuma raiz real, mas possui duas razes

complexas. Como j mencionamos, a grande vantagem em utilizar os


nmeros complexos em vez dos nmeros reais que, dado um polinmio qualquer com coecientes complexos, ele sempre tem uma raiz
complexa.

Isso foi o assunto da tese de doutorado do

Prncipe da

Matemtica, Johann Carl Friedrich Gauss (1777-1855).

Teorema 8.8

(Teorema Fundamental da lgebra)

. Todo polinmio

no constante com coecientes complexos de grau n possui exatamente


n razes complexas, contadas com multiplicidade.
Uma demonstrao do Teorema Fundamental da lgebra foge do
objetivo deste livro. Podem ser dadas vrias demonstraes diferentes desse teorema, utilizando diversas teorias matemticas avanadas.
Uma demonstrao desse teorema pode ser achada em [13].

272

8.4

Polinmios

Exerccios

1. Calcule o quociente e o resto da diviso de


os polinmios

p(x)

q(x)

por

q(x)

para

dados:

(a)

p(x) = 3x3 2x + 1

(b)

p(x) = x5 1

(c)

p(x) = 3x5 2x3 + 1

p(x)

q(x) = 7x 1;

q(x) = x 1;

2. Encontre os valores de

q(x) = x2 + x + 1

de forma que

A
B
x+1
= +
.
2
x x
x x1
3. Se os polinmios

a + b.

x2 x+4 e (xa)2 +(x+b) so iguais, encontre

4. Quais os valores de

P1 (x) = x x 6
5. A diviso de

P (x)

a e b que tornam iguais


P2 (x) = (x + a)2 b?
x4 + 1
P (x).

por

Encontre o polinmio

tem quociente

6. Qual o resto da diviso do polinmio

x100

por

os polinmios

x+2

x + 1?

p(x) pelo
p(x) = (x 1)(x 2) . . . (x n) + b .

7. Determine o resto da diviso do polinmio

g(x) = x,

onde

8. Mostre que

xn 1

divisvel por

x1

e resto 1.

para todo

polinmio

n 1.

9. Faa os seguintes itens:


(a) encontre o quociente da diviso de

xn+1 1

por

x 1;

8.4

Exerccios

273

(b) utilize a diviso anterior para calcular a soma

x3 + + xn

dos

geomtrica de razo

1 + x + x2 +

primeiros termos de uma progresso

x.

a para que o polinmio P (x) seja


P (x) = x3 + (1 a)x2 + (1 + a)x 1.

10. Determine o valor de


por

x a,

onde

11. Mostre que o polinmio

x 1.
12. Mostre que o resto

r(x)

P (x) = x100 2x50 + 1


da diviso do polinmio

divisvel

divisvel por

p(x)

por

r(x) = p(s).

xs

p(x) = an xn + an1 xn1 + + a1 x + a0


derivada de p(x) como sendo o polinmio:

Dado o polinmio
denimos a

p0 (x) = nan xn1 + (n 1)an1 xn2 + + 2a2 x + a1 .


5x4 e a
3
2
2
derivada do polinmio x +5x +2x1 o polinmio 3x +10x+2.
Por exemplo, a derivada do polinmio

x5

o polinmio

13. Usando as informaes do Exerccio 12, calcule:


(a) a derivada dos polinmios:
(i)

x + 1;

(ii)

x4 + 3 ;

(ii)

1 + x + x2 + x3 + + xn .

(b) Sabendo que

p(0) = 1,

cuja derivada
(i)
(ii)

x4 .
x2 + 1.

calcule tambm o polinmio

p(x)

274

(ii)

x3 + 2x2 + 3.

(c) Prove que se


(i)
(ii)

Polinmios

p(x)

q(x)

so polinmios, ento

(p + q)0 (x) = p0 (x) + q 0 (x)


(pq)0 (x) = p0 (x)q(x) + p(x)q 0 (x)
Sugesto: Faa primeiro para monmios.

Denimos uma
uma raiz

raiz mltipla

tal que

(x a)

p(x) como sendo


p(x). Caso a seja uma raiz

de um polinmio
divide

que no raiz mltipla, dizemos que ela


14. Mostre que
se,

raiz

raiz simples.

a raiz mltipla de um polinmio p(x) se, e somente


0
de p(x) e de p (x).

Sugesto: Use o exerccio anterior.

15. Para quais valores de

nN

tem-se que

(a)

1 + x2 + x4 + . . . + x2n2

divisvel por

1 + x + . . . + xn1 ?

(b)

1 + x3 + x6 + . . . + x3n3

divisvel por

1 + x + . . . + xn1 ?

(c) Generalize.
16.

(a) Resolva a equao


que

1
2

20x3 30x2 + 12x 1 = 0,

uma de suas razes.

(b) Uma raiz da equao

sabendo-se

a + 1,

x3 (2a+1)x2 +a(a+2)xa(a+1) = 0

ache as outras duas.

17. Ache os possveis valores de

aZ

para que o polinmio

a2 x4 + 4x3 + 4ax + 7
seja divisvel por

x + 1.

8.4

Exerccios

275

Um polinmio com coecientes reais no constante

redutvel

se

constantes.

p(x) dito ir-

p(x) = a(x)b(x), ento a(x) ou b(x) so polinmios


Quando p(x) no for irredutvel, diremos simples-

mente que ele

redutvel.

Os polinmios irredutveis desempe-

nham papel anlogo no conjunto dos polinmios ao dos nmeros


primos em

Z.

18. Prove que todo polinmio de grau

irredutvel.

f (x) um polinmio de grau 2 e possui uma raiz


f (x) redutvel.

19. Prove que se


real, ento

20. Mostre que todo polinmio

f (x)

de grau mpar

redutvel.

Um polinmio com coecientes inteiros no constante

p(x) dito

irredutvel sobre Q se p(x) = a(x)b(x) com a(x) e b(x) polinmios


com coecientes racionais, ento

a(x)

ou

b(x)

so polinmios

constantes.
Um teorema importante que descreve uma condio para um
polinmio ser irredutvel sobre

o conhecido critrio de Ei-

senstein, que diz:

Teorema 8.9

(Critrio de Eisenstein)

. Seja f (x) = a0 + a1 x +

+ an xn um polinmio com coecientes inteiros. Suponha que


exista um primo p tal que:

(a) p - an ;
(b) p | a0 , p | a1 , . . . , p | an1 ;
(c) p2 - a0 .

Ento, f (x) irredutvel sobre Q.

276

Polinmios

Para uma prova desse resultado veja o livro [2]. Faa os seguintes
problemas:
21. Mostre que os seguintes polinmios

f (x)

so irredutveis sobre

Q.
Sugesto: Use o critrio de Eisenstein .

(a)

f (x) = x4 + 2x3 + 2x2 + 2x + 2;

(b)

f (x) = x6 + 15;

(c)

f (x) = x4 + 10x3 + 20x2 + 30x + 22.

22. Determine quais dos polinmios abaixo so irredutveis sobre

Q.

Sugesto: Use o critrio de Eisenstein .

(a)

x3 x + 1

(b)

x3 + 2x + 10

(c)

x4 x + 1

O problema a seguir trata do polinmio de interpolao de Lagrange.


23. Demonstre a proposio a seguir:

Polinmio de Interpolao de Lagrange.


R, i = 1, 2, . . . , n,

com os

ai s

ai , bi em
0
os bi s nem

Sejam

dois a dois distintos e

todos nulos. Considere os polinmios

pi (x) = bi

(x ai ) (x ai1 )(x ai+1 ) (x an )


(ai a1 ) (ai ai1 )(ai ai+1 ) (ai an )

8.4

Exerccios

para

277

i = 1, 2, . . . , n.

Ento, o polinmio

p(x) =

n
X

pi (x)

i=1
o nico polinmio de grau menor que
para todos

n,

tal que

i = 1, 2, . . . , n.

24. Determine o polinmio

p(x)

de grau 7 tal que

p(1) = p(2) = = p(7) = 8

p(0) = 1.

p(ai ) = bi ,

278

Polinmios

A
Apndice: Funes

Estamos acostumados a expresses cotidianas que retratam uma

o quanto Joo ganha


funo do que ele trabalha , ou ainda a distncia que percorremos
uma funo da velocidade e do tempo que viajamos . Essas e outras
relao entre grandezas, como por exemplo,

expresses ilustram a noo de funo como uma relao entre grandezas de dois conjuntos dados. Matematicamente, a noo de funo foi
melhor entendida muito recentemente, com os avanos tericos ocorridos no nal do sculo XIX e incio do sculo XX. Entretanto, o seu uso
como instrumento e os estudos para tornar sua denio um objeto
claro so bem antigos e datam pelo menos desde o incio do clculo
diferencial, onde a noo de funo era por vezes entendida como sua

expresso analtica.

O entendimento dessa noo foi crucial para o

avano da Matemtica e importante que o estudante de Matemtica


tenha claro seu signicado.
Para iniciar a discusso um pouco mais formalmente da noo
de funo, vamos denir intuitivamente uma funo como um objeto
matemtico composto de trs ingredientes: um conjunto no vazio

279

A,

280

Apndice: Funes

domnio da funo, um conjunto no vazio B , chamado


contradomnio da funo e uma correspondncia, que associa a

chamado de
de

cada elemento do primeiro conjunto um nico elemento do segundo


conjunto. O trio domnio, contradomnio e correspondncia damos o
nome de funo. Para simplicar o seu uso, foi criada uma notao
que empacota todos os trs ingredientes. Denotamos uma funo por

f : AB
x f (x)
para indicar que
um elemento de

A o domnio, B o contradomnio e que se x


A ento a ele associaremos o elemento f (x) de B .

importante no confundir uma funo com sua expresso analtica,


quando esta dada.

Para caracterizar uma funo, precisamos dar

seus trs ingredientes: domnio, contradomnio e correspondncia, e


no somente a correspondncia

y = f (x).

Exemplo A.1. Seja a funo f denida de modo que o seu domnio


o conjunto dos nmeros naturais e o contradomnio o conjunto dos
nmeros naturais, e a correspondncia tal que a cada nmero natural
n associamos o seu quadrado n2 . Observe que podemos denotar isso
compactamente por:
f : NN
n n2

Veja tambm que se dermos simplesmente a expresso analtica


x x2 ou y = x2 para nossa funo, ela no estaria caracterizada,
pois no saberamos qual o domnio e o seu contradomnio.
Em alguns casos onde o domnio e o contradomnio esto xados e
claros para o interlocutor, podemos nos referir a uma funo simplesmente invocando sua correspondncia

y = f (x).

281

Exemplo A.2. Considere o domnio como sendo o conjunto P for-

mado pelas pessoas do Brasil e o segundo conjunto como sendo o conjunto L das letras do alfabeto. A correspondncia ser a seguinte: a
cada pessoa do Brasil, associaremos a primeira letra do seu nome.
Assim, uma pessoa chamada Mrio, ser associada letra M. Em
notao de funo:
f : P L
x f (x)

onde f (x) a primeira letra do nome de x.

Exemplo A.3. Considere o domnio S como sendo o conjunto dos

pontos de uma sala de aula e o contradomnio como sendo os nmeros


reais. A cada ponto x da sala de aula associamos sua temperatura t(x)
em um dado momento, medida por um termmetro instalado na sala.
Observe que t : S R assim denida uma funo, pois cada ponto
possui uma nica temperatura bem denida no instante xado, que
um nmero real. Por outro lado, se trocarmos os papis do domnio
e contradomnio e a cada nmero real associamos o ponto da sala que
tem aquela temperatura, no teremos uma funo, pois pode haver
mais de um ponto com a dada temperatura ou ainda uma temperatura
que no atingida por nenhum ponto da sala.

Exemplo A.4. Vamos agora dar outro exemplo em que no temos

uma funo, isto , cuja a nossa aparente correspondncia no de


fato uma correspondncia, pois no associa a cada elemento x do domnio um nico elemento f (x) do contradomnio. Para tanto, xe o
domnio como sendo o conjunto dos nmeros reais no intervalo [0, 1]
e como contradomnio o conjunto denido pelas sequncias de elementos no conjunto {0, 1, 2, . . . , 9}. Ou seja,

282

Apndice: Funes



= (a1 , a2 , a3 , . . . ); ai {0, 1, 2, 3, . . . , 9} .

Cada elemento x [0, 1] possui uma expanso decimal x = 0, x1 x2 x3 . . . .


Dena f : [0, 1] colocando f (x) = (x1 , x2 , x3 , . . . ).
A princpio, parece que f denida desse modo uma funo. Porm, olhando de perto vemos que o nmero 0, 1 possui mais de uma
representao na base decimal, pois 0, 1 = 0, 09999 . . . . Portanto, f
no est bem denida, isto , f no associa a cada elemento de [0, 1]
um nico elemento de .

Denio A.5. Dada uma funo f denida por


f : AB
x f (x)

o conjunto imagem de f o subconjunto f (A) do contradomnio B


formado pelos pontos y do contradomnio tais que existe algum ponto
x no domnio A tal que y = f (x). Ou seja
f (A) = {y B; existe x A tal que y = f (x)}.

imagem de um ponto x A o ponto f (x). Denimos tambm


0
a restrio de f a um subconjunto A de seu domnio a nova funo
A

A0

e os

demais elementos os mesmos. Denotamos essa nova funo por

f |A0

denida considerando-se o domnio como sendo o conjunto

ou ainda

f |A0 : A0 B
x f (x)

No Exemplo A.2 poderamos trocar o domnio por um de seus


subconjuntos no vazios. Por exemplo, poderamos considerar o subconjunto
Alagoas.

de

formado pelas pessoas do Brasil que nasceram em

283

Denio via Relaes


Um modo mais formal de denir funo usar a noo de relao
entre dois conjuntos
um subconjunto
relao

entre

B.

R do produto cartesiano
A e B que satisfaz duas condies:

R unvoca: dados x1 , x2 A
(x2 , y) R ento x1 = x2 ;
R total:
que dado

A e B simplesmente
A B . Uma funo uma

Uma relao entre

yB

tais que

(x1 , y) R

x A existe y B tal que (x, y) R. de modo


x A, existe um nico y B tal que (x, y) R.
dado

Funes Injetoras, Sobrejetoras e Bijetoras


Denio A.6. Uma funo f : A B dita injetora (ou

injetiva

se a seguinte propriedade vale:

Dados x, y A tais que f (x) = f (y), ento x = y.


Outro modo equivalente de formular tal propriedade usando sua
forma contrarrecproca:
Se x, y A so tais que x 6= y, ento f (x) 6= f (y).

Exemplo A.7. Por exemplo, a funo f : R R dada por f (x) = x2

no injetora, pois f (1) = (1)2 = 12 = f (1).


Por outro lado, se g : [0, +) R dada por g(x) = x2 , ento g
injetora, pois dados dois nmeros no negativos a e b tais que g(a) =
g(b), isto , a2 = b2 , ento a2 b2 = 0, de onde (a b)(a + b) = 0,
restando as possibilidades a = b ou a = b. Como a e b so positivos,
temos que a = b.

284

Denio A.8. Uma funo f : A B dita


brejetiva

Apndice: Funes

sobrejetora

(ou

so-

) se a seguinte propriedade vale:

Dado y B existe x A, tal que f (x) = y.

Exemplo A.9. Por exemplo, a funo f : R R dada por f (x) =


x2 do exemplo anterior no sobrejetora, pois no existe nenhum
nmero real x tal que f (x) = 1, por exemplo. Por outro lado, se
considerarmos g(x) : R [0, +) dada por g(x) = x2 , ento g
sobrejetora, pois dado qualquer nmero no negativo b, podemos tomar

a como sendo a = b de modo que g(a) = a2 = b.

Denio A.10. Uma funo dita

bijetora

(ou ainda

bijetiva

) se

ela injetora e sobrejetora.

f : R R dada por f (x) = x3 uma funo


bijetora, pois injetora e sobrejetora, j que dado y R, existe um
3
nico x R tal que y = x .
Quando f : A B bijetora, ento dado qualquer elemento y
B , existe um elemento x A tal que f (x) = y (pois f sobrejetora) e
esse elemento nico (pois f injetora). Em outros termos, podemos
denir uma nova funo: g : B A associando a cada elemento y B
o nico elemento x em A tal que f (x) = y . Em outras palavras,
Por exemplo, a funo

g(y) = x,
g

chamada de

se e somente se,

funo inversa de f .

Quando existe uma bijeo


que

f (x) = y.

entre dois conjuntos

tm a mesma quantidade de elementos ou

B,

dizemos

cardinalidade.

Para mais informaes sobre funes, recomendamos a leitura de [3].

Referncias Bibliogrcas
[1] AIGNER, M. e ZIEGLER, G. (2002).

no Livro.

Edgard Blcher.

[2] GARCIA, A. e LEQUAIN, I. (2003).

gebra.

As Provas esto
Elementos de l-

Projeto Euclides, IMPA.

[3] LIMA, E. L.; CARVALHO, P. C. P.; WAGNER, E. e


MORGADO, A.C. (2004).

dio. Volume 1.

A Matemtica do Ensino M-

Sociedade Brasileira de Matemtica.

[4] LIMA, E.L.; CARVALHO, P. C. P.; WAGNER, E. e


MORGADO, A.C. (2004).

dio. Volume 2.
[5] LIMA,E.L.;

A Matemtica do Ensino M-

Sociedade Brasileira de Matemtica.

CARVALHO,P.

MORGADO,A.C. (2004).

dio. Volume 3.

C.

P.;

WAGNER,E.

A Matemtica do Ensino M-

Sociedade Brasileira de Matemtica.

[6] LIMA, E.L.; CARVALHO, P. C. P.; WAGNER,E. e


MORGADO, A.C. (2001).

Temas e Problemas.

Socie-

dade Brasileira de Matemtica.


[7] LIMA, E.L. (2001).

lgebra Linear. Sociedade Brasileira

de Matemtica.

285

286

REFERNCIAS BIBLIOGRFICAS

[8] MORAIS FILHO, D. C. (2007).

tica.

Um Convite Matem-

EDUFCG.

[9] MORGADO, A.;

CARVALHO, J.;

Anlise Combinatria e Pro-

FERNANDEZ, P. (1991).

babilidade .

Sociedade Brasileira de Matemtica.

[10] RIBENBOIM, P. (2001).

Recordes.

Nmeros Primos: Mistrios e

Sociedade Brasileira de Matemtica.

[11] SANTOS, J. P. O. (1993)

meros.

CARVALHO, P.;

Introduo Teoria dos N-

IMPA.

[12] SANTOS, J. P. O.; MELLO, M. P. e MURARI, I. T.


C. (2006).

Introduo Anlise Combinatria.

Editora

Unicamp.
[13] SOARES, M. G. (2005).

plexa.

Clculo em uma Varivel Com-

Sociedade Brasileira de Matemtica.

Resoluc
ao de Problemas
Lista 01
Relembramos algumas dicas discutidas no livro-texto para ajudar na resoluc
ao de um problema em Matematica.
(D1) Ler bem o enunciado do problema e utilizar todas as informacoes disponveis.
(D2) Resolver casos particulares ou casos mais simples de problemas similares,
para adquirir familiaridade com o problema.
(D3) Mudar a representac
ao do problema, transformando-o em um problema
equivalente.
(D4) Usar a imaginac
ao, pesquisando caminhos alternativos.

Alem dessas dicas, vamos acrescentar mais algumas outras:

(D5) Re
una todos os fatos que voce conhece e que estejam relacionados ao
problema; Em seguida, selecione alguns deles que voce ache que possam
servir para resolver o problema.
(D6) Se o problema permitir, ache um invariante. Isto e, uma quantidade ou
propriedade que nao muda e que possa ser usada para resolver seu problema.
(D7) Se for possvel e conveniente, comece de tras-para-frente, iniciando na
situac
ao final onde voce deseja chegar e, a partir de jogadas validas, tente
chegar na situac
ao inicial.
(D8) Antes de avancar demais, cheque o trabalho ja desenvolvido, verificando
se voce cometeu algum erro. Isso lhe poupara tempo.
Claramente, nenhuma dessas dicas faria sentido se lida isoladamente e desconectada de um problema especfico. Alem disso, nenhuma delas e universal,
permitindo-lhe tornar-se um grande resolvedor de problemas. Na verdade, resolver problemas e algo que so se aprende fazendo. Porem, adotar estrategias
como as que listamos acima pode ajuda-lo a ganhar tempo. A seguir, vamos
listar alguns problemas para que voce tente empregar as estrategias descritas
acima e, eventualmente, outras que voce venha a criar. Boa sorte!

Buscando um Invariante
Em algumas situac
oes, a busca de uma quantidade ou de uma propriedade que
n
ao muda quando um processo ocorre, pode levar `a solucao do problema. Essa
quantidade e chamada de invariante. Isso e particularmente verdadeiro quando
consideramos problemas quem involvem impossibilidades, como os que vamos
descrever abaixo. Aplique a dica (D6) aos problemas 5, 6, 7 e 8.
1. Seis pessoas formando um crculo seguram pequenos quadros em suas
m
aos, nos quais estao escritos n
umeros. A cada rodada, escolhe-se uma
das pessoas e adiciona-se uma unidade ao n
umero escrito no quadro dessa
pessoa, bem como uma unidade aos n
umeros nos quadros de seus vizinhos.
(a) Se os n
umeros iniciais forem 1,0,1,0,0,0 e possvel, apos repetir esse
procedimento um certo n
umero de vezes, fazer com que todos os
quadros tenham os mesmos n
umeros?
(b) Se os n
umeros iniciais forem 5,2,0,3,5,6 mostre que e possvel, apos
repetir esse procedimento um certo n
umero de vezes, fazer com que
todos os quadros tenham os mesmos n
umeros.
(c) Se os n
umeros iniciais forem 5,2,0,3,5,6, qual o menor n
umero de
jogadas de modo que todos os quadros tenham os mesmos n
umeros?
2. Invente e resolva um problema, usando como inspiracao o problema ante apropriado para que
rior. Decida o grau de dificuldade da resolucao. E
ano? Discuta com seus colegas do PROFMAT.
3. Num tabuleiro de xadrez 8 8 e permitido escolher um quadrado 2 2
qualquer e trocar as cores de uma das linhas ou colunas deste quadrado.
possvel que se chegue a uma situacao na qual todos os quadrados do
E
tabuleiro 8 8 sejam brancos, exceto um?
4. Invente e resolva um problema, usando como inspiracao o problema ante apropriado para que
rior. Decida o grau de dificuldade da resolucao. E
ano? Discuta com seus colegas do PROFMAT.
5. Os n
umeros 1, 2, 3, . . . , 99 sao escritos no quadro-negro e e permitido realizar a seguinte operacao: apagar dois deles e substitu-los pela diferenca
do maior com o menor. Fazemos esta operacao sucessivamente ate restar
apenas um u
ltimo n
umero no quadro. Pode o u
ltimo n
umero que restou
ser o zero?

Resolvendo de Tr
as-para-frente
Andar para tras pode ser uma ferramenta importante na solucao de um
problema. A tecnica consiste em supor que chegamos ao objetivo que
pretendemos (posic
ao vitoriosa, por exemplo, ou uma configuracao especial) e, fazendo movimentos para tras, tentamos chegar na situacao inicial.
Resolva o seguinte problema usando a dica (D7):
permitido a cada
6. Joao e Maria brincam com um monte de 30 palitos. E
um deles retirar no seu turno 1, 2 ou 3 palitos. Ganha quem retirar o
u
ltimo palito. Sabendo que Joao comeca e que os dois aprenderam a jogar
com o mestre sabetudo, quem ganha o jogo? E retirando-se 1,2,3 ou 4
palitos, quem ganharia?
O jogo anterior se inclui numa classe chamada de jogos progressivamente
finitos. Esta classe e constituida de jogos que necessariamente acabam
ap
os um n
umero de jogadas. Outro exemplo deste tipo de jogo e o seguinte:
7. Suponha agora que na pilha existam 107 palitos e que a cada rodada, Joao
e Maria se alternam, escolhendo um primo p, n 0 e inteiro nao-negativo
e retirando pn palitos. Ganha quem retirar o u
ltimo palito. Pergunta-se:
se Joao comeca o jogo, quem ganha?
8. Agora, Joao e Maria dispoem de dois montes com 30 palitos cada. Em
cada turno, o jogador escolhe somente um dos montes e retira quantos
palitos quiser, inclusive o monte inteiro. Ganha quem retirar o u
ltimo
palito. Sabendo que Joao comeca, quem ganha o jogo?
9. Joao e Maria se alternam desenhando diagonais de um 2012-agono. Perde
quem desenhar uma diagonal cruzando alguma outra ja desenhada. Qual
e a estrategia vitoriosa para esse jogo?

Usando equac
oes
10. Demonstre que:
(a) n4 + 4 nao e primo se n > 1;
(b) Generalize, mostrando que n4 + 4n nao e primo, para todo n > 1.
11. Nove copias de certas notas custam menos de R$ 10,00 e dez copias das
mesmas notas (com o mesmo preco) custam mais de R$ 11,00. Quanto
custa uma copia das notas?
12. As paginas de um livro sao numeradas de 1 ate n. Ao somarmos estes
n
umeros, por engano um deles e somado duas vezes, obtendo-se o resultado
incorreto: 2.012. Qual e o n
umero da pagina que foi somado duas vezes?

13. Analise variac


oes deste problema. Por exemplo, permita que se some dois
n
umeros consecutivos duas vezes, ou ainda, o mesmo n
umero tres vezes.
Mude o valor da soma. Para cada mudanca que voce fizer, discuta com
seus colegas do PROFMAT a serie onde um desafio desse poderia ser
proposto.
14. Mostre que entre os retangulos com um mesmo permetro, o de maior area
e um quadrado.
15. Mostre que para quaisquer a, b, c reais vale
a2 + b2 + c2 ab + ac + bc.
16. Usando cada dgito 1,2,3,4,5,6,7,8, 9 somente uma vez, decida se e possvel
escrever n
umeros de modo que sua soma seja 100.
17. Invente e resolva um problema, usando como inspiracao o problema ante apropriado para que
rior. Decida o grau de dificuldade da resolucao. E
ano? Discuta com seus colegas do PROFMAT.
18. Joao esta na beira de um rio com dois baldes de 9 e 4 litros, sem marcacoes.
Ele deseja medir exatamente 6 litros de agua para poder levar para Maria
fazer uma deliciosa sopa para sua numerosa famlia. Mostre como Joao
dever
a proceder para obter os 6 litros de agua.
19. Analise variac
oes deste problema. Por exemplo, permita que a quantidade
a ser separada seja diferente de 6, ou ainda, que os baldes tenham outra
capacidade. Por exemplo, discuta o que ocorre quando os baldes tem
MDC diferente de 1. Decida o grau de dificuldade da resolucao de cada
apropriado para que ano? Discuta com seus
problema que voce criar. E
colegas do PROFMAT.
20. Faca mentalmente as seguintes multiplicacoes:
(a) 27 37
(b) 21 23
21. Invente e resolva um problema, usando como inspiracao o problema ante apropriado para que
rior. Decida o grau de dificuldade da resolucao. E
ano? Discuta com seus colegas do PROFMAT.

Sugest
oes de Leitura
Revista Eureka!
Na revista Eureka! e possvel encontrar varios problemas interessantes e
desafiadores, alem de artigos que tratam de assuntos que nao sao rotineiros
na sala de aula. A revista esta disponvel no site www.obm.org.br, onde
podem ser encontradas as provas (muitas resolvidas) da Olimpada Brasileira
de Matematica.
Colec
ao de Iniciac
ao Cientfica J
unior da OBMEP
A colec
ao e destinada aos alunos premiados na Olimpada Brasileira de
Matem
atica das Escolas P
ublicas. Pode ser baixada em http://www.obmep.org.br,
onde e possvel tambem obter um banco de problemas e solucoes.
Tio Petros http://tiopetrus.blogia.com/
Um blog interessante, com varios recursos que podem ser aproveitados.
Cabe ao leitor uma leitura crtica.
Alem da colec
ao Olimpadas de Matem
atica e da colecao Professor de
Matem
atica, que podem ser adquiridos no site www.sbm.org.br, recomendo
a leitura dos seguintes livros:
How to solve it - G. Polya, Princeton Science Library. Princeton University
Press, 1945.
Excelente livro sobre didatica da Matematica e a arte de resolver problemas. O autor apresenta seu ponto de vista sobre as tecnicas de resolucao
de problemas, com alguns exemplos discutidos. A discussao envolve problemas de todos os nveis (Fundamental e Medio). Sem traducao (que eu
conheca) para o portugues.
Mathematical Circles: Russian Experience (Mathematical World, Vol. 7)
Dimitri Fomin, Sergey Genkin, Ilia V. Itenberg.
Excelente livro, com varios problemas interessantes para ampliar a visao
do estudante. Esta dividido em duas partes, sendo a primeira dedicada
aos primeiros anos do Fundamental II. Os problemas sao daqueles que
consideramos de raciocnio, privilegiando a criatividade e ideias novas dos
estudantes. Pode ser usado como livro para um clube ou grupo especial
de treinamento de Matematica na sua escola.
Problem Solving Strategies - A. Engel. Problem Books in Mathematics,
Springer.
Uma coletanea de exerccios, em geral do nvel de olimpadas de matematica.
Est
a separado por temas (geometria, inducao etc) e com dicas e resolucoes
para os exerccios propostos. Pode ser usado numa turma avancada de
preparacao para olimpadas ou exames difceis.

Resoluc
ao de Problemas
Lista 01 com dicas e discuss
ao
Faca mentalmente as seguintes multiplicacoes:
1. 27 37
2. 21 23
Invente e resolva um problema, usando como inspiracao o problema anterior.
apropriado para que ano? Discuta
Decida o grau de dificuldade da resolucao. E
com seus colegas do PROFMAT.
Dica 1. Para o item a), note que 3 37 = 111. Para o item b), lembre-se que
212 = 441.
Discuss
ao 1. A ideia desse problema e simplesmente ilustrar que conhecimento
previo relacionado ao que procuramos pode ajudar na resolucao de um problema.
Creio que e o tipo problema que pode ser usado em qualquer serie, especialmente
as menores do Fundamental II. Discuta com seus colegas a respeito. Comecar
do zero um problema pode tornar mais difcil encontrar a solucao do problema.
Uma avaliac
ao criteriosa de fatos conhecidos ou de problemas semelhantes pode
nos ajudar a encontrar a solucao mais rapidamente.

Buscando um Invariante
Em algumas situac
oes, a busca de uma quantidade ou de uma propriedade que
n
ao muda quando um processo ocorre, pode levar `a solucao do problema. Essa
quantidade e chamada de invariante. Isso e particularmente verdadeiro quando
consideramos problemas quem involvem impossibilidades, como os que vamos
descrever abaixo. Aplique a dica (D6) aos problemas 5, 6, 7 e 8.
1. Seis pessoas formando um crculo seguram pequenos quadros em suas
m
aos, nos quais estao escritos n
umeros. A cada rodada, escolhe-se uma
das pessoas e adiciona-se uma unidade ao n
umero escrito no quadro dessa
pessoa, bem como uma unidade aos n
umeros nos quadros de seus vizinhos.
(a) Se os n
umeros iniciais forem 1,0,1,0,0,0 e possvel, apos repetir esse
procedimento um certo n
umero de vezes, fazer com que todos os
quadros tenham os mesmos n
umeros?
(b) Se os n
umeros iniciais forem 6,3,0,0,3,6 mostre que e possvel, apos
repetir esse procedimento um certo n
umero de vezes, fazer com que
todos os quadros tenham os mesmos n
umeros.
(c) Se os n
umeros iniciais forem 6,3,0,0,3,6, qual o menor n
umero de
jogadas de modo que todos os quadros tenham os mesmos n
umeros?

Dica 2. Observe o resto na divisao por 3 da soma dos n


umeros nos quadros
e um invariante.
imDiscuss
ao 2. A noc
ao-chave aqui e o que vem a ser um invariante. E
portante frisar que algo n
ao muda quando realizamos uma jogada. Neste
caso, para o primeiro item, o invariante em questao e o resto da divisao
da soma total por 3. Como a soma da configuracao inicial deixa resto 2
quando dividida por 3, temos que e impossvel chegar na situacao onde todos os n
umeros dos quadros sao iguais comecando com 1,0,1,0,0,0, ja que
se todos os n
umeros sao iguais, o resto da soma e zero quando dividida
por 3.
Para o segundo item, basta fazer uma construcao. Em geral, o aluno
comeca com o processo tentativa-e-erro, ate pegar o jeito e inferir que jo importante guiar o aluno
gadas sao interessantes para atingir o objetivo. E
neste processo, estimulando a inferencia de propriedades gerais. Isso permite, por exemplo, ter ideias sobre o terceiro item. Por exemplo, ele pode
perceber que os n
umeros so aumentam a cada jogada. Isso permite inferir
que o n
umero mnimo de jogadas deve ser maior igual a (36-18)/3=6. Isso
se deve ao fato que a primeira configuracao que talvez seja possvel que
igual todos os n
umeros e 6,6,6,6,6,6 (soma 36) e que a configuracao inicial
6,3,0,0,3,6 tem soma 18. Assim, como a cada jogada acrescentamos 3, o
mnimo e maior ou igual a 6. De fato, 6 e o mnimo pois uma sequencia
que resolve o problema e
6, 3, 0, 0, 3, 6 6, 4, 1, 1, 3, 6 6, 4, 2, 2, 4, 6
6, 5, 3, 3, 4, 6 6, 5, 4, 4, 5, 6 6, 6, 5, 5, 5, 6 6, 6, 6, 6, 6, 6.
2. Invente e resolva um problema, usando como inspiracao o problema ante apropriado para que
rior. Decida o grau de dificuldade da resolucao. E
ano? Discuta com seus colegas do PROFMAT.
3. Num tabuleiro de xadrez 8 8 e permitido escolher um quadrado 2 2
qualquer e trocar as cores de uma das linhas ou colunas deste quadrado.
possvel que se chegue a uma situacao na qual todos os quadrados do
E
tabuleiro 8 8 sejam brancos, exceto um?
Dica 3. Olhe a paridade da diferenca entre o n
umero de quadrados brancos e quadrados pretos.
Discuss
ao 3. A paridade da diferenca entre o n
umero de quadrados brancos e quadrados pretos nao muda a cada operacao permitida. Como no
incio temos 32 quadrados brancos e 32 quadrados pretos, a cada passo
teremos sempre um n
umero PAR como resultado da diferenca entre os
quadrados brancos e os quadrados pretos, mostrando que e impossvel
atingir a configurac
ao pedida.

4. Invente e resolva um problema, usando como inspiracao o problema ante apropriado para que
rior. Decida o grau de dificuldade da resolucao. E
ano? Discuta com seus colegas do PROFMAT.
5. Os n
umeros 1, 2, 3, . . . , 99 sao escritos no quadro-negro e e permitido realizar a seguinte operacao: apagar dois deles e substitu-los pela diferenca
do maior com o menor. Fazemos esta operacao sucessivamente ate restar
apenas um u
ltimo n
umero no quadro. Pode o u
ltimo n
umero que restou
ser o zero?

Resolvendo de Tr
as-para-frente
Andar para tras pode ser uma ferramenta importante na solucao de um
problema. A tecnica consiste em supor que chegamos ao objetivo que
pretendemos (posic
ao vitoriosa, por exemplo, ou uma configuracao especial) e, fazendo movimentos para tras, tentamos chegar na situacao inicial.
Resolva o seguinte problema usando a dica (D7):
permitido a cada
6. Joao e Maria brincam com um monte de 30 palitos. E
um deles retirar no seu turno 1, 2 ou 3 palitos. Ganha quem retirar o
u
ltimo palito. Sabendo que Joao comeca e que os dois aprenderam a jogar
com o mestre sabetudo, quem ganha o jogo? E retirando-se 1,2,3 ou 4
palitos, quem ganharia?
Dica 4. Para o jogo retirando-se 1,2 ou 3 palitos, tente resolver o problema
fazendo uso do mnimo de itens da lista abaixo.
(a) Reduza o n
umero de palitos no monte.
(b) Tente com 7 e depois passe para 10 palitos.
(c) Analise o que acontece quando seu adversario esta na posicao 4.
(d) Analise o que acontece quando seu adversario esta na posicao 8.
(e) Analise o que acontece quando seu adversario esta em uma posicao
m
ultiplo de 4.
Discuss
ao 4. A estrategia vencedora e deixar seu adversario numa posicao
m
ultiplo de 4. Verifique isso tracando uma arvore de possveis caminhos a
partir de uma dada posicao, onde a posicao 0 significa vitoria. Isso pode
ser feito construindo-se um grafo orientado, isto e, um conjunto de pontos
(posic
oes) e setas indicando as jogadas permitidas.
O jogo anterior se inclui numa classe chamada de jogos progressivamente
finitos. Esta classe e constituida de jogos que necessariamente acabam
ap
os um n
umero de jogadas. Outro exemplo deste tipo de jogo e o seguinte:

7. Suponha agora que na pilha existam 107 palitos e que a cada rodada, Joao
e Maria se alternam, escolhendo um primo p, n 0 e inteiro nao-negativo
e retirando pn palitos. Ganha quem retirar o u
ltimo palito. Pergunta-se:
se Joao comeca o jogo, quem ganha?
Dica 5. Observe que o primeiro n
umero que nao e potencia de primo e o
6. Ou seja, se Joao deixa Maria com 6 palitos na jogada dela, Joao ganha.
Discuss
ao 5. O conjunto de posicoes vitoriosas consiste nos m
ultiplos de
6. De fato, se Joao deixa Maria com um m
ultiplo de 6 palitos na jogada
dela, depois de Maria retirar uma potencia de primo, Joao pode retornar
a uma posic
ao que e m
ultiplo de 6 retirando 1,2,3,4 ou 5 palitos. Assim,
no final Maria estara na posicao 6 e Joao ganhara o jogo.
8. Agora, Joao e Maria dispoem de dois montes com 30 palitos cada. Em
cada turno, o jogador escolhe somente um dos montes e retira quantos
palitos quiser, inclusive o monte inteiro. Ganha quem retirar o u
ltimo
palito. Sabendo que Joao comeca, quem ganha o jogo?
Dica 6. Olhe na expressao dos montes quando escritos na base 2.
Discuss
ao 6. A estrategia vencedora desse jogo e deixar seu oponente
numa situac
ao de modo que quando somamos os dgitos na base dois
dos montes relativos a mesma potencia, o resultado e par. Ou seja, os
dgitos sao iguais. Por exemplo, se os montes tem 15 e 21 palitos, primeiro
escrevemos eles na base dois:
15 = (1111)2

21 = (10101)2 .

Em seguida, escrevemos a soma


10101
+1111

11212
Chamaremos de posicao vencedora, aquela que obtiver como resultado das
somas das colunas apenas os dgitos 2 e 0. As demais sao as perdedoras.
Por exemplo, a posic
ao acima (um monte com 15 e outro com 21) e posicao
perdedora. Para vencer o jogo, basta o jogador transformar este resultado
(11212) numa posic
ao vencedora, retirando palitos. Por exemplo, podemos retirar 6 palitos do monte com 21 para deixa-lo com 15 tambem.
Assim, a soma coluna a coluna dos dgitos sera (2222). Para mostrar que
a estrategia e a vitoriosa, voce deve verificar que
A partir de uma posicao vencedora, sempre podemos ir para uma
posic
ao perdedora.

A partir de uma posicao perdedora, nunca podemos ir para uma


posic
ao vencedora
Como o zero e uma posicao vencedora, essa e a estrategia vitoriosa do
jogo.
9. Joao e Maria se alternam desenhando diagonais de um 2012-agono. Perde
quem desenhar uma diagonal cruzando alguma outra ja desenhada. Qual
e a estrategia vitoriosa para esse jogo?
Dica 7. Observe que o que acontece quando um jogador traca uma diagonal principal, isto e, aquela que divide o polgono em dois polgonos
com a mesma quantidade de vertices.
Discuss
ao 7. Como temos uma quantidade par de vertices, e possvel
tracar alguma diagonal principal. Digamos que Joao tenha tracado uma
diagonal principal, dividindo o 2012-agono em dois 1006-agonos, P1 e P2 .
A seguir, observe que as diagonais que nao cruzam a diagonal principal
corresponde exatamente `as diagonais de um dos dois polgonos obtidos.
Assim, para cada diagonal de Maria que nao cruza a diagonal principal,
Jo
ao podera tracar uma diagonal correspondente no outro polgono. Ao
final, Maria tera que cruzar a diagonal principal tracada.

Usando equac
oes do primeiro e segundo graus
10. Nove copias de certas notas custam menos de R$ 10,00 e dez copias das
mesmas notas (com o mesmo preco) custam mais de R$ 11,00. Quanto
custa uma copia das notas?
11. As paginas de um livro sao numeradas de 1 ate n. Ao somarmos estes
n
umeros, por engano um deles e somado duas vezes, obtendo-se o resultado
incorreto: 2.012. Qual e o n
umero da pagina que foi somado duas vezes?
Discuss
ao 8. Primeiramente, relembre que a soma dos n primeiros n
umeros
e S = n(n + 1)/2. Observe tambem a soma efetuada incorretamente esta
entre S + 1 e S + n. Assim, sabemos que S + 1 2012 S + n, ou seja,
n2 + n + 2 4024 n2 + 3n
Testando os valores de n2 + n + 2 temos que n = 62, ja que 61 e pouco
e 63 e demais e a func
ao e crescente. Portanto, o valor desejado e obtido
fazendo 2012 S = 58.
12. Analise variac
oes deste problema. Por exemplo, permita que se some dois
n
umeros consecutivos duas vezes, ou ainda, o mesmo n
umero tres vezes.
Mude o valor da soma. Para cada mudanca que voce fizer, discuta com
seus colegas do PROFMAT a serie onde um desafio desse poderia ser
proposto.

13. Determine os valores de a para os quais a funcao quadratica ax2 ax + 12


e sempre positiva.
14. Mostre que entre os retangulos com um mesmo permetro, o de maior area
e um quadrado.
15. Mostre que para quaisquer a, b, c reais vale
a2 + b2 + c2 ab + ac + bc.
16. Usando cada dgito 1,2,3,4,5,6,7,8, 9 somente uma vez, decida se e possvel
escrever n
umeros de modo que sua soma seja 100.
Discuss
ao 9. A resposta deste problema e negativa, ou seja, e impossvel
realizar o que se pede. De fato, primeiramente observe que todos os
n
umeros construdos tem que ser menores que 100. Portanto, cada n
umero
tem no maximo dois dgitos. Digamos que os dgitos que forem usados na
casa das unidades dos n
umeros construdos formem um conjunto com soma
t. Naturalmente, o conjunto dos n
umeros que foram usados nos algarismos das dezenas dos n
umeros construdos formara um conjunto com soma
45 t, ja que 1 + 2 + 3 + . . . + 9 = 45. Portanto, teremos que a soma sera
(45 t)10 + t = 100,
e essa equac
ao do primeiro grau nao tem solucoes inteiras.
17. Invente e resolva um problema, usando como inspiracao o problema ante apropriado para que
rior. Decida o grau de dificuldade da resolucao. E
ano? Discuta com seus colegas do PROFMAT.
18. Joao esta na beira de um rio com dois baldes de 9 e 4 litros, sem marcacoes.
Ele deseja medir exatamente 6 litros de agua para poder levar para Maria
fazer uma deliciosa sopa para sua numerosa famlia. Mostre como Joao
dever
a proceder para obter os 6 litros de agua.
19. Analise variac
oes deste problema. Por exemplo, permita que a quantidade
a ser separada seja diferente de 6, ou ainda, que os baldes tenham outra
capacidade. Por exemplo, discuta o que ocorre quando os baldes tem
MDC diferente de 1. Decida o grau de dificuldade da resolucao de cada
apropriado para que ano? Discuta com seus
problema que voce criar. E
colegas do PROFMAT.

Resoluc
ao de Problemas
Lista 02

Divisibilidade
1. Prove que se n e mpar
(a) n2 1 e divisvel por 8;
(b) n3 n e divisvel por 24;
(c) n2 + (n + 2)2 + (n + 4)2 + 1 e divisvel por 12.
2. Tres n
umeros primos p, q e r, maiores que 3, formam uma progressao
aritmetica, ou seja, q = p + d e r = p + 2d. Prove que d e divisvel por 6.1
3. Mostre que 311 3 e divisvel por 112 .
4. Um robo possui dois botoes, permitindo que a cada momento ele suba a
degraus ou desca b degraus de uma escada com infinitos degraus. Sabendo
que o robo esta no incio da escada, pergunta-se:
(a) Se a = 12 e b = 3, e possvel que o robo visite todos os degraus apos
uma sucessao desses movimentos?
(b) Mostre que se a e b sao primos entre si, o robo consegue visitar todos
os degraus.
5. Encontre o resto que deixa
(a) 2001 2002 2003 2004 + 20052 quando e dividido por 7;
(b) 2100 quando e dividido por 3;
28
(c) 1237156 + 34
quando e dividido por 111.
6. Encontrar o u
ltimo dgito dos n
umeros
(a) 19892005 ;
(b) 777777 + 250 ;
(c) 1 + 22 + 32 + + 20052 .
7. Verifique que
. . . 2} +(333
111
. . . 1} = 222
. . . 3})2 .
| {z
| {z
| {z
2012 uns
1 Foi

1006 dois

1006 tr
es

demonstrado em 2004 pelos Matem


aticos B. Green e T. Tao que existem progress
oes
aritm
eticas de tamanho arbitrariamente grande formadas somente por primos. A prova pode
ser encontrada em www.arxiv.org.

8. Demonstre que o n
umero 1 000
. . . 00} 1 e composto.
| {z
2012 zeros

9. Considere o polinomio p(n) = am nm +am1 nm1 + +a0 de grau m 1


com coeficientes inteiros e n N. Prove que p(n) e um n
umero composto
para infinitos valores de n.2
10. Prove que se (x0 , y0 ) e uma solucao da equacao diofantina linear ax by =
1, ent
ao a area do triangulo cujos vertices sao (0, 0), (b, a) e (x0 , y0 ) e
1/2.
11. Ao entrar numa sala, Joao se depara com 100 interruptores e 100 lampadas,
numerados de 1 a 100. O interruptor n acende somente a lampada n, para
cada valor de n = 1, 2, . . . , 100. De incio, todas as 100 lampadas estao
apagadas. Joao aperta todos os interruptores m
ultiplos de 2. A seguir,
aperta todos os m
ultiplos de 3, e assim sucessivamente, ate o u
nico interruptor m
ultiplo de 100. Ao final deste procedimento, pergunta-se:
(a) Quais lampadas estao apagadas?
(b) Quantas lampadas acenderam exatamente 4 vezes?
(c) Qual e o n
umero da lampada que mais acendeu?

2 Sugest
ao: Use o fato de que existe a N tal que = |p(a)| > 1 e mostre que divide a
p(k + a), para todo k Z.

Princpio das Casas dos Pombos


12. Seja C um conjunto formado por cinco pontos de coordenadas inteiras no
plano. Prove que o ponto medio de algum dos segmentos com extremos
em C tem tambem coordenadas inteiras.
13. O conjunto dos dgitos 1, 2, ..., 9 e dividido em tres grupos. Prove que o
produto dos n
umeros de algum dos grupos deve ser maior que 71.
14. Prove que se N e mpar entao para qualquer bijecao
p : IN IN
do conjunto IN = {1, 2, . . . , N } o produto P (p) = (1p(1))(2p(2)) (N
p(N )) e necessariamente par.3
15. Dado um conjunto de 25 pontos no plano tais que entre quaisquer 3 deles
existe um par com distancia menor que 1. Prove que existe um crculo de
raio 1 que contem pelo menos 13 dos 25 pontos dados.
16. Marquemos todos os centros dos 64 quadradinhos de um tabuleiro de
possvel cortar o tabuleiro com 13 linhas retas que nao
xadrez de 8 8. E
passem pelos pontos marcados e de forma tal que cada pedaco de recorte
do tabuleiro tenha no maximo um ponto marcado?
17. Prove que existem duas potencias de 3 cuja diferenca e divisvel por 2012.
18. Mostre que entre nove n
umeros que nao possuem divisores primos maiores
que cinco, existem dois cujo produto e um quadrado.
19. Um disco fechado de raio um contem sete pontos, cujas distancias entre
quaisquer dois deles e maior ou igual a um. Prove que o centro do disco e
um destes pontos.
20. Seja a um n
umero irracional. Prove que existem infinitos n
umeros racionais r = p/q tais que |a r| < 1/q 2 .
21. Suponha que cada ponto do reticulado4 plano e pintado de vermelho ou azul.
Mostre que existe algum ret
angulo com vertices no reticulado e todos da mesma
cor.
22. Um certo livreiro vende pelo menos um livro por dia. Sabendo que o livreiro
vendeu 463 livros durante 305 dias consecutivos, mostre que em algum perodo
de dias consecutivos o livreiro vendeu exatamente 144 livros.

3 Dica:

O produto de v
arios fatores
e par se, e somente se, um dos fatores
e par.
refere-se ao conjunto dos pontos do plano com coordenadas inteiras

4 Reticulado

Resoluc
ao de Problemas
Lista 02

Solu
c
oes dos Exerccios de Divisibilidade
1. Prove que se n e mpar
(a) n2 1 e divisvel por 8;
Demonstrac
ao. Como n e mpar, temos que n = 2q + 1 para algum
q Z. Assim,
n2 1 = (2q + 1)2 1 = 4q 2 + 4q + 1 1 = 4q 2 + 4q = 4q(q + 1).
Como q e q + 1 sao consecutivos, um deles tem que ser par. Logo,
n2 1 e divisvel por 8.

(b) n3 n e divisvel por 24;


Demonstrac
ao. Usaremos o fato que
se a, b s
ao primos entre si, entao c e divisvel por ab se, e somente
se, c e divisvel por a e por b.
Assim, para mostrar que n3 n e m
ultiplo de 24, basta mostrar que
n3 n e m
ultiplo de 3 e de 8, uma vez que 3 e 8 sao primos entre si.
Observe que n3 n = n(n2 1). Como n e mpar, segue do exerccio
anterior que n3 n e divisvel por 8. A divisibilidade por 3 decorre
do lema dos restos. De fato, se n 0 mod 3, n3 n sera m
ultiplo
de 3, claramente. Se n 1 mod 3, temos que n3 n sera m
ultiplo
de 3, j
a que
13 1 0 mod 3.
Se n 2 mod 3, temos que n3 n sera m
ultiplo de 3, ja que
23 2 0

mod 3,

completando assim a prova.

(c) n2 + (n + 2)2 + (n + 4)2 + 1 e divisvel por 12.

Demonstrac
ao. Novamente, usaremos o fato que se a, b sao primos entre
si, ent
ao c e divisvel por ab se, e somente se, c e divisvel por a e por
b. Assim, para mostrar que a expressao e m
ultiplo de 12, basta mostrar
que 3 e de 4. Vamos primeiro `a divisibilidade por 4. Observe que como
n e mpar, basta verificar os casos quando n 1 mod 4 e n 3 mod 4.
Quando n 1 mod 4 temos que
n2 + (n + 2)2 + (n + 4)2 + 1 12 + (1 + 2)2 + (1 + 4)2 + 1 36 0

mod 4.

Do mesmo modo, quando n 3 mod 4 temos que


n2 + (n + 2)2 + (n + 4)2 + 1 32 + (3 + 2)2 + (3 + 4)2 + 1 84 0

mod 4.

Para verificar a divisibilidade por 3, repetimos o argumento usando os


restos da divis
ao por 3. Observe que quando n 0 mod 3 temos que
n2 + (n + 2)2 + (n + 4)2 + 1 02 + (0 + 2)2 + (0 + 4)2 + 1 21 0

mod 3.

Procedendo de modo inteiramente analogo para os casos n 1 mod 3 e


n 2 mod 3, completamos a prova.

2. Tres n
umeros primos p, q e r, maiores que 3, formam uma progressao
aritmetica, ou seja, q = p + d e r = p + 2d. Prove que d e divisvel por 6.1
Demonstrac
ao. Temos que d e par, ja que d = q p, com p e q primos
maiores que dois, logo mpares. Basta mostrar que d e divisvel por 3.
Ora, supondo o contrario, temos que d 1 mod 3 ou n 2 mod 3.
Analisando o resto de q e r modulo 3 nas tabelas a seguir (em funcao dos
restos de d nas colunas e p nas linhas).
Para q temos
mod 3
p1
p2

d1
2
0

d2
0
1.

mod 3
p1
p2

d1
0
1

d2
1
0.

Para r temos:

1 Foi demonstrado em 2004 pelos Matem


aticos B. Green e T. Tao que existem progress
oes
aritm
eticas de tamanho arbitrariamente grande formadas somente por primos. A prova pode
ser encontrada em www.arxiv.org.

Logo, se d n
ao e divisvel por 3, ou q ou r serao divisveis por 3, o que e
impossvel, j
a que s
ao ambos n
umeros primos.

3. Mostre que 311 3 e divisvel por 112 .


4. Um rob
o possui dois botoes, permitindo que a cada momento ele suba a
degraus ou desca b degraus de uma escada com infinitos degraus. Sabendo
que o rob
o est
a no incio da escada, pergunta-se:
(a) Se a = 12 e b = 3, e possvel que o robo visite todos os degraus apos
uma sucess
ao desses movimentos?
Demonstrac
ao. Se apertamos o botao de subir x vezes e o de descer
y vezes, o rob
o ira subir ou descer 12x 3b degraus. Note que esse
n
umero e sempre m
ultiplo de 3, nao sendo possvel para o robo com
uma sequencia desses movimentos atingir um degrau com n
umero
que n
ao e divisvel por 3.
(b) Mostre que se a e b sao primos entre si, o robo consegue visitar todos
os degraus.
Demonstrac
ao. Como a e b sao primos entre si, pelo Teorema de
Bezout-Bachet, e possvel encontrar x e y de modo que ax by = 1.
Assim, o rob
o pode sair do nvel inicial e ir para o degrau 1 apertandose x vezes o bot
ao de subir e em seguida apertando-se y vezes o botao
de descer. Repetindo esse procedimento, o robo pode atingir qualquer
degrau.
5. Encontre o resto que deixa
(a) 2001 2002 2003 2004 + 20052 quando e dividido por 7;
Demonstrac
ao. Vamos usar o lema dos restos. 2001 quando dividido
por 7 deixa resto 6; Assim, 2002 e divisvel por 7 e 2005 deixa resto
3 quando dividido por 7. Logo, 20052 deixa resto 2, ja que 32 deixa
resto 2 quando dividido por 7.
(b) 2100 quando e dividido por 3;
Demonstrac
ao. 22 resto 1 quando dividido por 3. Logo, 2100 = (22 )5 0
deixa resto 1 quando dividido por 3.
(c) 1237156 + 34

28

quando e dividido por 111.

6. Encontrar o u
ltimo dgito dos n
umeros
(a) 19892005 ;

(b) 777777 + 250 ;


(c) 1 + 22 + 32 + + 20052 .
7. Verifique que
111
. . . 1} = |222{z
. . . 2} +(333
. . . 3})2 .
| {z
| {z
2012 uns

1006 dois

1006 tr
es

Demonstrac
ao. Observe que
111
. . . 1} =
| {z
2012 uns

102012 1
9

e que

101006 1
222
. . . 2} = 2
| {z
9

1006 dois

(333
. . . 3})2 = 9
| {z

101006 1 2
.
9

1006 tr
es

Portanto, basta verificar que


102012 1
101006 1
101006 1 2
=2
+9
,
9
9
9
De fato, cancelando 9 em ambos os lados
102012 1 = 2 (101006 1) + (101006 1)2 ,
como queramos demonstrar.

8. Demonstre que o n
umero 1 000
. . . 00} 1 e composto.
| {z
2012 zeros

Demonstrac
ao. Para termos uma ideia da prova deste fato, vamos provar
que 1001 e composto. Ora, temos que
1001 = 11 91.
Do mesmo modo, 100001 = 11 9091 e que 10000001 = 11 909091.
Assim, depois dessa analise inicial, fica bem claro que nosso candidato
para divisor do n
umero pedido e o 11; Podemos verificar sem maiores
dificuldades que
1 |000 {z
. . . 00} 1 = 11 9090
. . . 90} 91.
| {z
2012 zeros

1005 vezes

9. Considere o polin
omio p(n) = am nm +am1 nm1 + +a0 de grau m 1
com coeficientes inteiros e n N. Prove que p(n) e um n
umero composto
para infinitos valores de n.2
10. Prove que se (x0 , y0 ) e uma solucao da equacao diofantina linear ax by =
1, ent
ao a
area do tri
angulo cujos vertices sao (0, 0), (b, a) e (x0 , y0 ) e
1/2.
11. Ao entrar numa sala, Joao se depara com 100 interruptores e 100 lampadas,
numerados de 1 a 100. O interruptor n acende somente a lampada n, para
cada valor de n = 1, 2, . . . , 100. De incio, todas as 100 lampadas estao
apagadas. Jo
ao aperta todos os interruptores m
ultiplos de 2. A seguir,
aperta todos os m
ultiplos de 3, e assim sucessivamente, ate o u
nico interruptor m
ultiplo de 100. Ao final deste procedimento, pergunta-se:
(a) Quais l
ampadas estao apagadas?
k
0
1
Demonstrac
ao. Dado um n
umero natural n = p
1 pk , com pi s
0
primos e i s 0, sabemos que o n
umero de divisores de n e

d(n) = ki=1 (i + 1).


Para responder essa pergunta, basta analisar quantos divisores cada
n
umero de 1 a 100 possui. Na verdade, basta verificar a paridade
da quantidade de divisores de cada n
umero, ja que se d(n) for par,
a l
ampada n ficara acesa. Da expressao acima, e claro que d(n) e
mpar se, e somente se, i e par, para cada valor de i. Assim, d(n)
e mpar se, e somente se, n e um quadrado. Logo, as lampadas que
ficar
ao apagadas sao aquelas que tem n
umero que e um quadrado
perfeito.
(b) Quantas l
ampadas acenderam exatamente 4 vezes?
Demonstrac
ao. Basta contar quantas lampadas tem 8 ou 9 divisores. Fazendo a contagem usando a expressao em produto de primos,
obtemos a lista: 24, 30, 36, 40, 42, 54, 56, 66, 70, 78, 88 e 100.
(c) Qual e o n
umero da lampada que mais acendeu?
Demonstrac
ao. Essa lampada corresponde aos n
umeros que tem mais
divisores. O problema esta mal-formulado, ja que temos cinco n
umeros
com exatamente 12 divisores: 60, 72, 84, 90 e 96.

2 Sugest
ao: Use o fato de que existe a N tal que = |p(a)| > 1 e mostre que divide a
p(k + a), para todo k Z.

Resoluc
ao de Problemas
Lista 03

1. De quantas maneiras podemos escolher tres n


umeros distintos do conjunto
I50 = {1, 2, 3, . . . , 49, 50} de modo que sua soma seja
a) um m
ultiplo de 3?
b) um n
umero par?
2. Considere o conjunto In = {1, 2, 3, . . . , n 1, n}. Diga de quantos modos e
possvel formar subconjuntos de k elementos nos quais nao haja n
umeros
consecutivos?
3. Considere as letras da palavra PERMUTA. Quantos anagramas de 4 letras
podem ser formados, onde:
a) n
ao h
a restric
oes quanto ao n
umero de consoantes ou vogais?
b) o anagrama comeca e termina por vogal?
c) a letra R aparece?
d) a letra T aparece e o anagrama termina por vogal?
4. Calcular a soma de todos os n
umeros de 5 algarismos distintos formados
com os algarismos 1, 3, 5, 7 e 9.
5. Quantos n
umeros podem ser formados pela multiplicacao de alguns ou de
todos os n
umeros 2, 2, 3, 3, 3, 5, 5, 6, 8, 9, 9?
6. Quantos s
ao os n
umeros naturais de sete dgitos nos quais o dgito 4 figura
exatamente 3 vezes e o dgito 8 figura exatamente 2 vezes?
7. De quantas maneiras uma comissao de 4 pessoas pode ser formada, de um
grupo de 6 homens e 6 mulheres, se a mesma e composta de um n
umero
maior de homens do que de mulheres?
8. O comprimento de uma palavra e a quantidade de caracteres que ela possui. Encontre a quantidade de palavras de comprimento 5 que podemos
formar fazendo uso de 10 caracteres distintos, de forma que nao existam
tres caracteres consecutivos identicos em cada palavra.
9. De quantos modos 6 casais podem sentar-se ao redor de uma mesa circular
de tal forma que marido e mulher nao fiquem juntos?
10. Quantas s
ao as permutacoes das letras da palavra PROFMAT em que o
P ocupa o primeiro lugar, ou o T ocupa o segundo lugar, ou o A o sexto
lugar?

11. De quantas formas podemos representar o n


umero 15 como soma de varios
n
umeros naturais?
12. Quantos quadrados perfeitos existem entre 40.000 e 640.000 que sao m
ultiplos
simultaneamente de 3, 4 e 5?
13. Oito amigos v
ao ao cinema assistir a um filme que custa um real. Quatro
deles possuem uma nota de um real e quatro possuem uma nota de dois
reais. Sabendo-se que o caixa do cinema nao possui nenhum dinheiro, de
quantas formas eles podem organizar uma fila para pagar o filme permitindo o troco pelo caixa?
14. Encontre o n
umero de zeros que termina o n
umero 2010!.
15. A func
ao de Euler associa a cada n
umero natural n o valor (n) igual ao
n
umero de inteiros positivos menores ou iguais a n relativamente primos
com n. Ou seja,


(n) = {1 m n; (m, n) = 1} .
Usando os princpios estudados, mostre que se n se decompoe em fatores
k
1 2
primos como n = p
ao
1 p2 . . . pk , ent

 


1
1
1
(n) = n 1
1
... 1
.
p1
p2
pk

Você também pode gostar